Sie sind auf Seite 1von 120

Wills

& Trusts
Chapter 1 Introduction
1. The Power to Transmit Property at Death: Justifications and Limitations a. The Right to Inherit and the Right to Convey i. Blackstone Commentaries (CB p. 1) 1. Wills & testaments are creatures of the civil or municipal laws govt should be able to regulate wills & trusts ii. Locke Two Treatises of Govt (CB p. 2) 1. Wills & testaments are a personal right, to be governed by people, not governments iii. Hodel v. Irving (p. 3) iv. F/P 1. In 1889, pursuant to a series of land acts enacted by Congress, which divided communal reservations of Indian tribes into individual allotments for Indians and unallotted lands for non-Indian settlement, each male Sioux head of household took 320 acres of land and other individuals took 160 acres. The lands were held in trust by the United States in order to protect the allottees from improvident disposition of their lands to white settlers. 2. The allotment program quickly failed because the Indians leased their allotted lands to white ranchers and farmers, which resulted in parcels being splintered into multiple undivided interests that could not be alienated or partitioned, due to the fact the land was held in trust. 3. To address this problem, Congress enacted the Indian Land Consolidation Act of 1983, which contained an escheat provision. The escheat provision essentially provided that any undivided fractional interest in a tract within a tribes reservation or jurisdiction, would escheat (escheat = reversion of property to the state in the absence of legal heirs or claimants) to that tribe and could not be passed by intestacy or devise, if the interest represented two percent or less of the total acreage of the tract and it earned its owner less than $100 in the preceding year before it was due to escheat. 4. The statute became law on January 12, 1983 and it contained no provision for the payment of compensation to the owners of interests covered by the escheat provision. 5. Mary Irving, Patrick Pumpkin Seed, and Eileen Bisonette, appellees, are or represent heirs or devisees of members of the Oglala Sioux Tribe who died in March, April, and June of 1983. But for the escheat provision of the Act, $2,700 which represents 26 escheatable interests in the Cross estate and $1,816 which represents 13 escheatable interests in the Pumpkin Seed estate would have passed, in ordinary course, to appellees or those they represent. 6. Appellees filed suit in the District Court alleging the escheat provision resulted in a taking without just compensation under the Fifth Amendment. 7. The District Court held the statute was Constitutional. 8. The Court of Appeals reversed concluding that while appellees had no vested rights in decedents property, their decedents had a right to control the disposition of their property at death. The Court held that appellees had standing to invoke that right and that the taking of that right without just compensation to decedents; estates violated the Fifth Amendment. v. I 1. Did the original version of the escheat provision of the Indian Land Consolidation act of 1983 constitute a taking of appellees decedents property without just compensation? vi. R/A/H 1. HELD: Yes the escheat provision was an unconstitutional taking of decedents property w/o just compensation

Chapter 1 Introduction

1 of 120

RULE: The States, and where appropriate, the United States, has broad authority to adjust the rules governing the descent and devise of property without implicating the guarantees of the Just Compensation Clause. 3. Since the escheatable rights are not de minimis, nor does the availability of inter vivos transfer obviate the need for descent and devise, a total abrogation of these rights cannot be upheld. The regulation virtually amounts to the abrogation of the right to pass on a certain type of property-a small undivided interest-to ones heirs. vii. Notes: Right to Transfer vs Right to Receive 1. Constitutional protections seem to apply only to the decedents power to transfer, not the heir/beneficiarys right to receive b. The Policy of Passing Wealth at Death i. Halbach An Introduction to Death, Taxes, and Family Property (CB p. 16) 1. Pros: In support of Inheritance a. In a society based on private property, inheritance is not objectionable (person should have power to decide how to transfer property; rewards life of hard work) b. Inheritance is a natural & proper way to express & reinforce family ties, which are important to a healthy society & good life c. Incentive to bring forth creativity, hard work, initiative, and productivity that benefits others d. Encourages ppl to save for old age and give to family e. Encourages families to love, serve, and protect their elders ii. Bentham The Theory of Legislation (CB p. 17) 1. ??? iii. Oliver, Shapiro, and Press Them Thats Got Shall Get: Inheritance and Achievement in Wealth Accumulation (p. 18) 1. Inheritance is an unearned benefit that produces unequal opportunities 2. E.g. The richest 1% of baby boomers (btwn 1987 and 2011) will get 1/3 of the worth of estates; The next richest 9% will take the next 1/3. iv. Ascher Curtailing Inherited Wealth (CB p. 20) 1. Govt should tax inherited wealth; i.e. federal wealth transfer taxes should curtail inheritance, and increase equal opportunity, while also raising revenue. 2. This attacks inheritance by healthy, adult descendants they didnt earn wealth; luck shouldnt dictate inheritance should be allowed only where public policy clearly justifies it. 3. Aschers proposal: All property owned at death, after payment of debts and administration expenses, should be sold, and the proceeds paid to the US govt. 4. Six Exceptions to Aschers Proposal a. Marital exemption spouses could continue to provide for each other after death; amount should depend on the length of the marriage. b. Dependent Lineal Descendants Decedents would be allowed to provide for dependent lineal descendants (the amount allowed would decrease w/ the descendants age) c. Disabled lineal descendants Decedents could give generous amounts to disabled descendants (independent of age?) d. Lineal Ascendants Decedent can give unlimited inheritance to lineal ascendants (e.g. parents, grandparents, etc) e. Universal exemption Allows a moderate amount of property either 1) to pass outside the exemptions or 2) to augment amounts passing under them i.e. decedent can leave something to persons of his/her choice, regardless of whether another exemption was available f. Charity Up to a fixed fraction of an estate could go to charity v. Kristol Taxes, Poverty, and Equality (CB p. 22)

2.

Chapter 1 Introduction

2 of 120

c.

Problem is: large inherited concentrations of wealth can lead to an aristocracy (the wealthy in power; undermines democracy) 2. Kristols Proposition: Legislate that no large fortune should outlast the lifetime of the man who made it, but rather that such a large fortune should dissolve into much smaller fortunes upon his death a. Make a rule/policy: No individual can inherit more than $1 Million, and any possessor of a large fortune must distribute it to his children, his relatives, his friends, anyone, but no one can get more than the maximum legacy (tax-free). b. Institutional donations would be unlimited vi. Blum and Kalven The Uneasy Case for Progressive Taxation (CB p. 24) 1. A great source of inequality of opportunity in our society is cultural inheritance. 2. ? vii. Langbein The Twentieth-Century Revolution in Family Wealth Transmission (p. 26) 1. Wills/trusts is a dying field 2. The main means of transferring wealth is by paying for education (especially since skills/knowledge are more meaningful today than e.g. land). The Problem of the Dead Hand i. Dead hand control = Decedent conditions a gift to a beneficiary upon a beneficiary behaving in a certain way (a.k.a. incentive trust) ii. R.3d Property (Wills & Other Donative Transfers) 10.1 Donors Intention Determines the Meaning of a Donative Document and is Given Effect to the Maximum Extent Allowed by Law 1. The controlling consideration in determining the meaning of a donative document is the donors intention. 2. The donors intention is given effect to the maximum extent allowed by law. 3. Note: This favors the decedents freedom of disposition 4. Note: (p. 33) Posner believes that courts should be able to modify the conditions of wills iii. Valid Conditions on Gifts 1. Testamentary conditional gifts are valid UNLESS a. They violate public policy, OR b. Judicial enforcement would constitute a state action violating constitutionally protected fundamental rights iv. Invalid Conditions on GIfts 1. Absolute restraints on marriage a. Gifts conditioned on the beneficiary not marrying anyone (at least wrt. first marriages) generally violate the fundamental right to marry, and are VOID. b. ExceptionPartial Restraints: Partial restraints on marriage that impose only reasonable restrictions are NOT contrary to public policy, and are VALID c. ExceptionTemporal/Religion Requirement: Gifts requiring a beneficiary to marry within a reasonable amount of time, even to someone of a particular religious background, HAVE BEEN HELD VALID i. Shapira v. Union National Bank (p. 28) ii. F/P 1. David Shapira, M.D., testator, conditioned his sons (Daniel Jacob Shapira, Plaintiff) inheritance under his will upon Plaintiff being married to, or marrying within seven years of testators death, a Jewish girl with two Jewish parents. 2. Plaintiff filed suit alleging that such a condition was unconstitutional based upon the premise that the right to marry is protected by the Fourteenth Amendment to the Constitution of the United States. iii. I

1.

Chapter 1 Introduction

3 of 120

1.

Is a condition upon inheritance, which is based on marriage, is unconstitutional, contrary to public policy, and unenforceable because of its unreasonableness? No upholding and enforcing the provision of Dr. Shapiras will conditioning the bequests to his sons upon their marrying Jewish girls does not offend the Constitutions of Ohio or of the United States. The conditions contained in decedents will are reasonable restrictions. His unmistakable testamentary plan was for his possessions to be used to encourage the preservation of the Jewish faith. The condition did not pressure plaintiff into marriage by the reward of money because the seven year time limit is a reasonable grace period, which would give plaintiff ample time for reflection and fulfillment of the condition without constraint or oppression

iv. R/A/H 1.

2.

3.

Religion Requirement a. Gifts that require a beneficiary to be of a particular religion are generally held to violate public policy concerning religious freedom, and are INVALID 3. Encouraging separation and/or divorce a. Gifts that require a beneficiary to separate or divorce before receiving the gift generally are deemed against public policy and VOID b. BUT, gifts that provide for a beneficiary only in the event of separation and/or divorce are not necessarily deemed to encourage divorce. c. The controlling factor: The decedents dominant intent encouraging separation/divorce vs. merely providing support in the event of separation/divorce 4. Promoting family strife a. Gifts conditioned upon family members ostracizing and/or not communicating w/ other fam members generally have been held to violate public policy and are VOID 5. Property Destruction Directives a. Generally, testators are not free to direct people to destroy property upon the testators deaths; such directives are generally INVALID i. Note: Though people may generally destroy property when they are alive, they cant generally direct others to destroy property when theyre dead. ii. Note: Prof. Strahilevitzs Proposal for Conditional Right to Destroy Property at Death: 1. Testators directives to destroy property at death should be held VALID if: a. During life, the testator put a future interest in the property up for sale, and b. The government declined to condemn the future interest, and c. The owner turned down the highest bid for the future interest v. Remedies for Invalid Conditions 1. When a conditional gift violates public policy, the critical question is whether there is a gift-over clause: a clause that provides where the gift is to go if the condition is not met

2.

Chapter 1 Introduction

4 of 120

2.

If gift-over clause exists: Usually the courts distribute the gift as per the express gift-over clause (and not to the beneficiary) b. No gift-over clause: Usually, the courts give the gift to the beneficiary, free & clear of any conditions) vi. Incentive Trusts and the Dead Hand (p. 35) 1. In modern practice, conditional gifts, such as in Shapira (above, p. 3) tend to be made in trust (known as an incentive trust). 2. Incentive trusts usu. focus on ensuring that the beneficiary doesnt adopt a slothful or wasteful life 3. Prime examples of incentives: a. Pursue an education b. Moral incentives those that reflect the settlors (testators) moral or religious outlook, or promote a particular way of living. c. Pursue a productive career incentives designed to encourage the beneficiary to have a productive career 4. RULE: Provided that these incentives do not violate public policy, courts generally will enforce them. 5. Notes: These must be drafted carefully, so that they are not taken advantage of (see note p. CB p. 36) vii. Destruction of Property at Death (CB p. 37) 1. See Property Destruction (above, p. 4) Transfer of the Decedents Estate a. Probate and Nonprobate Property i. Probate Property: property that passes through probate under the 1) the decedents will or 2) by intestacy 1. i.e. A will disposes of the decedents probate property only 2. Note: Probate is the Default: Nonprobate property passes pursuant to the terms of the instrument in question to the transferees identified in the instrument without passing through the probate system. ii. Nonprobate Property: Property that passes outside of probate under an instrument other than a will. iii. Examples of Nonprobate Transfers 1. Joint Tenancy Property (both real and personal) a. Joint tenants hold the property in question concurrently. They own it in whole and fractional shares. b. When the decedent dies, his/her fractional share vanishes. With multiple survivors, the shares are recalculated (this is the right of survivorship). c. Technically, no property interest passes to the survivor(s). d. RULES: Survivor(s) must show death certificate of the decedent e. Examples bank accounts, brokerage and mutual fund accounts, real estate 2. Life Insurance a. Life insurance is an agreement between the insured and the insurance co that, upon the insureds death, the co will pay benefits/proceeds to the named beneficiary or beneficiaries. b. RULES: Beneficiary must show death certificate of the insured decedent c. Notes: At common law, life insurance policies were the only type of contract with a payment-on-death (POD) clause that qualified as a valid will substitute. The modern trend recognizes all contracts with P.O.D. clauses as valid, nonprobate transfers exempt from the probate process. 3. Contracts with payable-on-death (POD) provisions (and see Life Insurance, above) a. A decedent may have a contract with a bank, an employer, or some other person/corporation to distribute property at the decedents death to a named beneficiary

a.

Chapter 1 Introduction

5 of 120

Examples: i. Pension plans often provide survivor benefits ii. Tax-deferred investment plans (IRAs, 401(k)s, etc.) often survivor benefits iii. Brokerage accounts c. RULES: Beneficiary need only file a death certificate with the custodian holding the property 4. Interests in trust a. Testamentary trusts: Property held in a testamentary trust (i.e. trust created under the decedents will) PASSES THROUGH PROBATE b. Uniform Testamentary Additions to Trusts Act (UTATA) trusts: These also PASS THROUGH PROBATE. c. Inter Vivos trusts: Property put in an inter vivos trust (during decedents life) DOES NOT PASS THROUGH PROBATE d. Note: When the property is in trust, the trustee holds the property for the benefit of one or more named beneficiaries (see Trusts on p. 69) 5. Legal Life Estates and Remainders a. When the party who holds a legal life estate dies, the right to possession passes to the party holding the remainder. i. BUT the transfer is the result of the original grantors division of the property between the life estate and the remainder, not the result of the deceased life tenant passing a property interest. b. Properly created legal life estates and remainders AVOID PROBATE iv. Who Takes Nonprobate Property 1. The decedents nonprobate property goes to the transferees identified in the written instrument (as long as it properly creates the nonprobate property arrangement). v. What Happens If the Nonprobate Instrument Fails to Create Proper Nonprobate Transfer? 1. If the property in question does not qualify as nonprobate property, the property automatically falls to probate (as the default system). a. Will vs. Intestacy: i. Will: A properly executed will constitutes an expression of the persons intent as to who should take the property when he/she dies ii. Intestacy: If 1) a decedent does not have a will, or 2) the decedents will does not dispose of all the decedents property, then the property passes via intestacy to the decedents heirs. b. Intestacy is the DEFAULT: If the decedent takes no steps to opt out of intestacy (e.g. by writing a will), all of the property passes through intestacy i. 755 ILCS 5/4-14 the IL intestacy rule c. How to Opt Out of Intestacy: One can opt out of intestacy by 1) properly executing a will, or 2) by properly executing a will substitute i.e. a recognized nonprobate method of transferring property. (see Will Substitutes, p. 59) i. Will goes through probate ii. Will substitute does NOT go through probate b. Administration of Probate Estates i. The core functions of Probate 1. Provides Evidence of Transfer of Title to the new owners 2. Protects creditors a. By providing a procedure for payment of debts (probate ensures that creditors receive notice, and have a chance to present their claims and receive payment) b. Note also: Probate also extinguishes the claims of creditors who do not present their claims to the probate court 3. Distributes the decedents property a. The key is: it distributes decedents property to those intended

b.

Chapter 1 Introduction

6 of 120

b. The other key is: this happens AFTER the decedents creditors are paid ii. Probate Terminology and History 1. Testate: Describes a decedent who dies with a valid last will and testament. His/her property will be distributed according to the last will and testament 2. Intestate: Describes a decedent who dies WITHOUT a valid last will and testament. His/her property will be distributed according to the state statute on descent and distribution. a. Note: The terms testate and intestate are not mutually exclusive a will may dispose of some, but not all of the decedents property.. In that case, the decedent is both testate AND intestate 3. Testator/Testatrix: A male or female (respectively) who executes a valid will a. Note: today, testator is gender-neutral 4. Devise: A gift of REAL PROPERTY under a will a. Note: Today, devise is increasing being used to describe testamentary gifts of either real or personal property. b. The word can be used as a noun or verb. 5. Devisee: A beneficiary receiving real property under a will 6. Bequest: A gift of PERSONAL PROPERTY under a will a. Note: The word can also be used as a verb (bequeath, bequeaths, etc.) 7. Legacy: A gift of MONEY under a will 8. Legatee: A beneficiary receiving money under a will 9. Personal Representative: The person appointed by the probate court to wind up/probate the decedents affairs 10. Executor: The person appointed by the will itself to wind up/probate the decedents affairs, if the decedent dies testate a. Note: i.e. Executor is what we call a personal representative who is appointed by the will, and not the court. 11. Administrator: What the personal representative is called if the decedent either 1) dies intestate, or 2) dies testate, but does not name an executor a. Note: Today, the term personal representative is being used increasingly, regardless of whether the decedent died testate or intestate 12. Probate Court: The state court with special jurisdiction over determining who is entitled to receive the decedents probate property. 13. Statute of descent and distribution: The statute that governs how a decedents intestate property will be distributed 14. Heirs: At common law, beneficiaries who received decedents INTESTATE REAL PROPERTY were called heirs. The property was said to descend to heirs. a. Note: this is different than devisees, who received testate real property 15. Next-of-kin: At common law, beneficiaries who received INTESTATE PERSONAL PROPERTY were called next-of-kin. Property was said to be distributed to next-of-kin. a. Note: This is different than I dont know the word.. but see bequest, above. iii. A summary of Probate Procedure 1. Opening Probate a. Who has probate jurisdiction (Primary Jurisdiction) i. The probate court in the county where the decedent was domiciled at the time of death has primary (or domiciliary) jurisdiction over the decedents probate estate. ii. The court has jurisdiction over 1) the decedents personal property and 2) the decedents real property located within that jurisdiction iii. IL Law (755 ILCS 5/5-1 Place of Probate) see blue sup p. 14 1. See also 755 ILCS 5/5-2 Situs of personal estate of NONRESIDENT b. Opening the Probate process

Chapter 1 Introduction

7 of 120

2.

3.

i. Probate is opened by presenting the decedents death certificate ii. The court issues letters testamentary appointing an executor or letters of administration appointing an administrator. 1. Majority Rule: Requires notice to interested parties before selection and appointment of the executor or administrator iii. Note: Once the court issues its letters, the personal representative is authorized to begin his or her responsibilities. 1. IL Law - Representative: Any corporation qualified to accept and execute trusts in IL is qualified to act as representative (755 ILCS 5/1-3) 2. IL Law Debtor as Executor: If a debtor owes testator, and debtor is executor, he still owes testator UNLESS testator expressly says the debtor is clear in the will (755 ILCS 5/4-15) c. Ancillary Jurisdiction i. If real property is located in another jurisdiction, ancillary administration may be necessary ii. Ancillary jurisdiction ensures that 1. 1) local creditors in the jurisdiction where the real property is located receive notice and an opportunity to present their claims, and 2. 2) there is compliance with that jurisdictions recording system. Will Contests a. If a party wishes to file a claim challenging the validity of a will offered for probate, most jurisdictions have statutes requiring the contest to be brought in a timely manner after probate is opened, or the claim is barred. Probate Administration: Once the court issues its letters, the personal representative is authorized to begin his or her responsibilities a. UPC 3-108: No proceeding, formal or informal, may be initiated more than 3 yrs from the date of death. If its more than 3 years, intestacy is presumed. b. Personal Representatives Powers: Jurisdictions are split as to the PRs powers to administer the estate i. Formal (a.k.a. Notice) Probate: Require supervision by the probate court 1. UPC 3-401: Formal probate under the UPC is a litigated judicial determination after notice to interested parties 2. The ct supervises the actions of the PR in administering the estate 3. The ct must approve the inventory and appraisal of the estate; payment of debts; family allowance; granting options on real estate; sale of real estate; borrowing of funds and mortgaging of property; leasing of property; proration of federal estate tax; personal reps commissions; attys fees; prelim & final distributions; and discharge of the personal rep 4. This is SLOW and expensive 5. Formal proceedings become final judgments if not appealed ii. Informal (a.k.a. Ex Parte) Probate: The personal rep administers the estate without going back to court 1. Note: Under UPC, Informal probate is the norm a. An interested party may file a petition for formal probate (UPC 3-502) 2. E.g. rep has broad powers of a trustee in dealing w/ estate property rep may collect assets; clear titles; sell property;

Chapter 1 Introduction

8 of 120

invest in other assets; pay creditors; continue any business of the decedent; and distribute the estate ALL without going court approval (UPC 3-715) 3. The personal rep may close the estate by filing a sworn stmt that he has published notice to creditors, administered the estate, paid all claims, and sent a statement and accounting to all known distributes (UPC 3-1003) 4. (we assume the personal rep is a trusted family member) 5. Requirements for Informal Probate (UPC 3-301) a. The rep petitions for appointment (rep doesnt have to give notice of this) i. The petition contains pertinent info about the decedent and the names/addresses of the spouse, children or other heirs. ii. If a will is involved, the petition also includes devisees (rxers of real property) iii. If the petition is for probate of a will, the original will must accompany the petition c. Personal Representatives DUTIES: i. Inventory decedents assets: Duty to ascertain and take control of decedents property, which he/she inventories to the probate court ii. Give notice to and pay creditors: 1. Duty to give notice (usu by publication, or by actual notice) 1) of the opening of probate and 2) that the creditors are required to file all claims w/in a set statutory pd or else their claims will be forever barred. (Statutory pd is governed by nonclaim statutes) 2. Duty to pay those creditors 1) who present valid claims w/in the prescribed time pd 3. Duty to file federal and state estate tax returns and, if necessary, pay any taxes due 4. Probate and Titled property: Probate is necessary to transfer title to those assets, real or personal, that were titled in the decedents name. a. Where the probate asset has a written form of title in the decedents name, a probate court order is needed to transfer title properly. iv. The Costs of Probate 1. Probate is fucking expensive and slow ; even a quick probate may take up to 2 years 2. Avoiding Probate: Increasingly, people try to avoid probate because it is expensive and slow. In reality, it is difficult to put ALL property in nonprobate arrangements a. Nontitled probate assets: Probate can be avoided if all of the decedents property is non-titled personal property (e.g. furniture or personal effects). BUT if the takers (heirs) opt not to probate, then they may be subject to creditors claims. b. Small Estate probate assets: All states have statutes that allow heirs to avoid probate where the amount of property involved is small (see UPC 3-1201 to 3-1204) c. Universal Succession (European approach -- and to a limited extent, USA (e.g. California)): Title to the decedents property passes to the appropriate heirs or residuary devisees automatically and by operation of law without the need for a personal rep or probate i. Who pays the creditors?: The heirs/residuary legatees who take title to the decedents assets are then responsible for paying the

Chapter 1 Introduction

9 of 120

decedents creditors and the estates tax liability and distributing the decedents property to the appropriate takers. ii. Louisiana is the only United States state that has adopted universal succession iii. Universal succession is in limited form in e.g. CA permits universal succession for property passing to a surviving spouse) v. See also IL Statutes 1. 755 ILCS 5/1-1 to 11 (General Provisions) 2. 755 ILCS 5/25-1 to 4 (Small Estates) 3. 755 ILCS 5/28-1 to 12 (Independent Administration)

Chapter 2 Intestacy: An Estate Plan By Default


1. The Basic Scheme a. Introduction i. Intestacy is the norm roughly half the population dies intestate ii. Heirs v. Heirs Apparent 1. Heir = a person who SURVIVES the decedent (i.e. the decedent is already dead) 2. Heir Apparent = a person who is designated as the heir of someone who is still alive (i.e. no decedent yet) 3. Expectancies: An expectancy is an expectation to receive some property at the death of the decedent. a. Not a property Interest: Such an expectation by an heir apparent is NOT a property interest. The heir must survive the decedent to take anything, and even so, the decedent can defeat the expectancy by transferring property inter vivos or by executing a will that devises the property to others b. Transferability: A mere expectancy cannot be transferred at law (because it is not a property interest). i. BUT if the heir apparent agrees to transfer his/her expectancy for valuable consideration, and thereafter tries to avoid enforcement of the agreement on the grounds that an expectancy is not transferable, a court of equity will enforce the agreement if enforcement is fair and equitable under the circumstances. 4. POLICY behind Intestacy Statutes: a. Carry out the probable intent of the average intestate decedent i. E.g. favor the surviving spouse b. Protect the family; i.e. preserve the economic health of the family after a death. iii. UPC Intestacy Scheme 1. UPC 2-102 The Spouses Share: The decedents surviving spouse gets: a. 100% of intestate estate if: i. No descendant or parent of the descendant survives the decedent, OR ii. All of the deads surviving descendants are also descendants of the surviving spouse AND there is no other descendant of the surviving spouse who survives the dead b. The first $300,000 + 75% of any balance of the intestate estate if: i. No descendant of the dead survives the decedent, but a parent of the dead survives the dead c. The first $225,000 + 50% of any balanace of the intestate estate if: i. All of the deads surviving descendants are also descendants of the surviving spouse AND ii. the surviving spouse has one or more surviving descendants who are not descendants of the dead d. The first $150,000 + 50% of any balance of the intestate estate if:

Chapter 2 Intestacy: An Estate Plan By Default

10 of 120

i. One or more of the deads surviving descendants are not descendants of the surviving spouse 2. UPC 2-103 Share of Heirs Other Than Surviving Spouse: Any part of the intestate estate not passing to a decedents surviving spouse under 2-102 (or the entire intestate if there is no surviving spouse) passes in the following order to the other surviving heirs (who arent the spouse). a. If there are surviving descendants i. To the deads descendants by representation b. If there is no surviving descendant: i. 50/50 to each of the decedents parents, OR ii. 100% to the surviving parent if only one survives c. If there is no surviving descendant or parent i. To the descendants of the deads parents (i.e. to the deads siblings), or either of them (the parents) by representation d. If there is no surviving descendant, parent, or descendant of a parent (e.g. sibling), but the dead is survived on both the maternal and paternal sides by one or more grandparents or descendants of grandparents (i.e. aunts/uncles) i. 50% to 1) the deads paternal grandparents equally if both survive, OR 2) to the surviving paternal grandparent if only one survives, OR 3) to the descendants of the deads paternal grandparents (i.e. the deads aunts/uncles) if both GPs are deceased (the descendants taking by representation); AND ALSO ii. 50% to 1) the deads maternal grandparents equally if both survive, OR 2) to the surviving maternal grandparent if only one survives, OR 3) to the descendants of the deads maternal grandparents (i.e. the deads aunts/uncles) if both GPs are deceased (the descendants taking by representation); 3. UPC 2-105 No Taker a. If there is no taker under the provisions of this Article, the intestate estate passes to the state. b. Share of Surviving Spouse Who Qualifies as a Spouse? i. Usually, the spouse takes first, before anyone else ii. Policy Issues: (1) Should the surviving spouse take ALL of the deceased spouses intestate property if there are surviving issue (kids?), (2) should the surviving spouse take ALL of the deceased spouses intestate property if there are surviving parents or issue of parents (i.e. siblings and their issue?) iii. Spouse can be: 1. Actually married people 2. Domestic Partners iv. Spouse DOES NOT INCLUDE 1. Cohabitants (nonmarried couples who live together) 2. Common law marriage (not all jurisdictions recognize common law marriages) v. Same-Sex Marriage, Domestic Partners, and Intestate Succession 1. Most states do NOT recognize same sex marriages; i.e. they do not allow same-sex couples to marry, and therefore, they do not recognize spousal-like intestacy rights 2. Civil Unions some states DO allow spousal-like intestacy rights, but do not allow gay marriage vi. Contract Claims (Same-Sex Partners) 1. In some states, a surviving same-sex partner might have a claim against the deceased partner, based on contract law. a. Whether the K must be express (either oral or writtenas opposed to implied) varies from jurisdiction to jurisdiction vii. Defense of Marriage Act: In 1996, Congress enacted the Defense of Marriage Act

Chapter 2 Intestacy: An Estate Plan By Default

11 of 120

Defines marriage for federal purposes as applying to only heterosexual couples States that despite the Full Faith & Credit Clause of the Constitution, states are not required to recognize same-sex marriages contracted in other states. viii. Putative Spouses: Putative spouses generally do qualify as spouses 1. E.g. putative spouse == the couple goes through what at least one of the parties believes is a valid marriage ceremony, but the marriage is either void or voidable (e.g. one spouse is already married and not divorced). 2. RULE: As long as one party believes, in good faith, that the marriage is valid, the spouses qualify as putative spouses and are treated as spouses for purposes of most intestate schemes. ix. Married but separated: Spouses who are legally separated generally still qualify as spouses for the purposes of the intestate distribution scheme. 1. Note: Even if the parties have filed for divorce, the parties remain legally married until the court enters final judgment or decree of dissolution of marriage. 2. Spousal Abandonment: In some states, if one spouse abandons the other, the abandoning spouse may be disqualified from inheriting from the other spouse. x. Survival requirements: To be eligible to receive property from a decedent, a taker must survive the decedent. If the claimant fails to meet the survival requirement, the claimant is treated as if he/she predeceased the decedent. 1. Scope: Historically, survival requirement applied to anyone claiming a decedents probate testate or intestate property, but not nonprobate property. Modern trend applies to nonprobate, as well 2. Common law: To qualify as an heir, the party had to prove, by a preponderance of the evidence, that he/she survived the decedent by a millisecond. Whether a person survived the decedent is a question of fact. 3. Uniform Simultaneous Death Act (USDA): If there is no sufficient evidence as to who survived whom, the party claiming a right to take is to be treated as having predeceased the decedent. a. Criticism of the USDA: If 2 spouses die together (or near the same time), the 2 families should be grieving together, not suing each other 4. IL Law (755 ILCS 5/3-1): IL Law assumes that the order of deaths is apparent from evidence. 5/3-1 Governs what happens if the order of deaths cannot be learned from evidence. 5. Janus v. Tarasewicz (p. 80) 6. F/P a. Stanley and Theresa Janus returned from their honeymoon to learn that Stanleys brother had died unexpectedly. The couple was distraught and unknowingly took some Tylenol laced with cyanide. Stanley collapsed first, Theresa a short time later. b. Although there was conflicting medical evidence, Stanleys vital signs arguably disappeared during the ambulance ride to the hospital, and he was pronounced dead shortly after arrival. Theresa arguably still had a pulse and blood pressure upon arrival at the hospital, and she lived (on a respirator) for 2 more days before being pronounced dead. c. Stanleys life ins policy named Theresa as a beneficiary; if she failed to survive him, his mother was named as contingent beneficiary. d. Stanleys mother sued the ins co and the administrators of Stanley and Theresas estates for the funds, claiming that there was insufficient evidence that Theresa had survived Stanley (i.e. the mother should get the $$$) 7. I a. Did Theresa in fact survive Stanley? 8. R/A/H

1. 2.

Chapter 2 Intestacy: An Estate Plan By Default

12 of 120

c.

HELD: Stanleys mother losesthere was sufficient evidence to support the finding that Theresa had survived Stanley b. RULE: USDA rule if the title to property depends on the priority of death, and there is no sufficient evidence that the persons died other than simultaneously, the property of each person shall be disposed of as if he survived (see p. 84) c. RULE: Survivorship is a FACT which must be proven by a PREPONDERANCE OF THE EVIDENCE. d. There was sufficient evidence that Theresa was alive, with a palpable pulse and blood pressure, for about 48 hours after Stanley was pronounced dead. xi. Determining time of death: To determine survivorship, one needs to know when each party died 1. Common Law: Time of death = when there is an irreversible cessation of circulatory and respiratory functions 2. Modern Trend: Time of death = when there is irreversible cessation of total brain function (because circulation & respiration can be maintained artificially now) 3. The Clear and Convincing Evidence Standard: To minimize simultaneous death litigation a. RULE: Some states require that a claimant prove by clear and convincing evidence that he/she survived the decedent 4. UPC 120-Hour Approach: UPC 2-104 and 2-702 provide that: a. RULE: An heir or devisee or life insurance beneficiary who fails to survive by 120 hours (5 days) is deemed to have predeceased the decedent. b. The USDA was amended in 1990 to include the 120 Hour rule. c. Advantages of 120 Hour Rule: It addresses simultaneous deaths even if they dont arise from the same disaster d. Criticisms: 5. IL Law: 755 ILCS 5/3 (blue sup. p. 11) xii. Failure to meet survival requirement: Whichever standard is applied, if the claimant fails to meet the survival requirement, the claimant is treated as if he/she predeceased the decedent. xiii. Wills and Nonprobate instruments: 1. If the written instrument does NOT have its own survival requirement, the statutory survival requirement applies. 2. If the written instrument DOES have an express survival requirement, it applies. Shares of Descendants (p. 87) i. If A) there is no surviving spouse, or B) there is a surviving spouse, but he/she does not take all of the decedents property, then property goes to the decedents issue equally ii. After the spouses share (if any) is set aside, A) children and B) descendants of deceased children take the remainder of decedents property; everyone else is excluded 1. The descendants of deceased children represent the dead child, and divide the dead childs share among themselves 2. i.e. if the decedents had 3 sons while alive, and one of the sons died before the decedent did, then the dead sons descendants represent the dead son, splitting his share among themselves 3. Example of Representation: A dies intestate (and with no surviving spouse). A has 3 children, B, C, and D. C dies before A, but leaves a husband and 2 children. a. B takes 1/3 of As estate (and any of Bs children get nothing, because B got the intestate share) b. D takes 1/3 of As estate (and any of Ds children get nothing, because D got the intestate share) c. C would have taken 1/3 of As estate, but Cs dead. So, Cs 2 kids split Cs share, by representation. Cs husband gets nothing (intestacy usually excludes children in-law) iii. IL Law = 755 ILCS 5/2-1 (blue sup p. 4)

a.

Chapter 2 Intestacy: An Estate Plan By Default

13 of 120

iv. Distribution Schemes 1. English Per Stirpes: Always make the first division of decedents property at the first generation of descendants (whether there are any living takers or not); the dropping shares then drop by bloodline

2.

Assume A, B, C, D, E, H, and J all predecease the decedent, who dies with no surviving spouse, and intestate. c. Step 1: Under per stirpes, the property is always divided at the first generation (i.e. divided among decedents children), even if everyone in that generation is dead. d. Step 2: One share is given to each party who is alive, and one share is given to each party who is dead but survived by issue. i. e.g. here, even though A, B, and C are all dead, A, B, and C are all survived by issue, so each receives a 1/3 share). e. Step 3: Under English per stirpes, the shares for each party who is dead but survived by issue drop by bloodline. Each share drops only to the issue of the predeceased party. i. E.g. here, As 1/3 share is divided equally between E and F (so they each get 1/6). But, E is dead, so Es 1/6 drops to K, L, and M, equally (so each of them gets 1/18). ii. Bs 1/3 all goes to G. Because G is alive, N gets nothing iii. Cs 1/3 is divided among H, I, and J. But because H is dead and has no surviving issue, H gets nothing. So actually, Cs share is split into 2, not 3. I and J each get 1/6. I is alive, I gets 1/6. Because J is dead, Js 1/6 goes to O and P, who each get 1/12. f. Criticism: It is possible for descendants of equal degree to the decedent to take unequally.` Modern Per Stirpes (a.k.a. Per Capita with Representation): Make the first division of the decedents property at the first generation where there is a live taker; the dropping shares then drop by bloodline.

a. b.

a. b. c. d.

Assume A, B, C, E, H, and J all predecease the decedent, who then dies intestate (with no surviving spouse). Who takes the property? Step 1: Always divide at the first generation where there is a living taker. i. E.g. here, we start divding at the E, F, G, H, I, J level Step 2: Divide equally among all parties who are 1) alive or 2) dead, but survived by issue).

Chapter 2 Intestacy: An Estate Plan By Default

14 of 120

3.

i. E.g. Here, F, G, and I are alive; E, and J are dead, but are survived by issue; H is dead and has no surviving issue ii. So, the property is split into 5. F, G, and I each get 1/5. e. Step 3: Drop the remaining shares by bloodine. i. Here, Es issue (K, L, and M) each get 1/15 (a 3-way split of Es 1/5 share). Js issue (O and P) each get 1/10 (a 2-way split of Js 1/5 share). f. Criticism: It is possible for descendants of equal degree to the decedent to take unequally. g. Benefit: The per capita at each generation approach ensures that all descendants who are equally related to the decedent take equally. This is as opposed to the per-capita-at-each-generation approach pools the dropping shares (the shares for descendants who are dead but survived by issue). Pooling means that the dropping shares are added together, and then divided equally among all of the eligible takers at the next generation. Per capita at each generation: Always make the first division of decedents property at the first generation where there is a living taker, and the dropping shares drop by poolingcombine them and distribute them equally among the eligible takers at the next generation

Assume A, B, C, E, H, and J all predecease the decedent, who then dies intestate and with no spouse. c. Step 1: Always divide property at the first generation where there is a living taker. i. Here, begin dividing at Es generation. d. Step 2: One share is given to each party who is alive, and one share is given to each party who is dead but survived by issue. i. Here, F, G, and I are alive they all get 1/5. E and J are both dead, but survived by issue. H is dead and has no surviving issue. So H gets no share. e. Step 3: Pool the dropping shares by adding them together, and then dividing the total equally among the eligible takers at the next generation. i. Here, there are 2 dropping shares. Add them together (Es 1/5 + Js 1/5 = 2/5). Divide the 2/5 equally among the eligible takers at the next generation (K, L, M, O, and P N is not eligible, because her parent took already). So K, L, M, O, and P each get 2/25. 4. IL Law (755 ILCS 5/2-1): (see blue sup, p. 4) v. Power to Opt Out: An individual can opt out of a jurisdictions default intestate distribution approach by executing a valid will or nonprobate instrument that expressly provides for an alternative method of distributing decedents estate. vi. UPC approach: The original UPC (1969 version) adopted the per capita with representation approach. The revised version has adopted the per capita with each generation approach (UPC 2-106) vii. Negative Disinheritance (p. 91)

a. b.

Chapter 2 Intestacy: An Estate Plan By Default

15 of 120

2.

UPC 2-101(b) allows a negative will. The barred heir is treated as if he disclaimed his intestate share; i.e. he is treated as having predeceased the decedent a. Common law: Execute a valid will that disposes of all of the decedents property so that nothing passes through intestacy (depriving the heir of any chance of taking. This is because the heir could inherit against the clear intent of the will if any property is left to intestacy) b. Modern trend/UPC approach: Execute a will that merely expresses intent to disinherit an potential heir, even if some or all of the decedents property passes through intestacy and the heir otherwise would have qualified to take property. (UPC 2-101(b)) d. Shares of Ancestors and Collaterals i. If a decedent has no surviving spouse or issue, the property flows up the family tree to the decedents ancestors and collateral relatives. There are 3 major approaches to this: 1. Parentelic Approach: The intestate estate passes to grandparents and their descendants; if no grandparents/descendants, then to great-grandparents and their descendants, etc. a. In distributing the property, the per stirpes, per capita, or per capita at each generation doctrines apply, depending on the default approach in the state. 2. Degree of Relationship Approach: The intestate estate passes to the closest of kin, counting degrees of relationship a. Determining the degree of relationship: On the family tree, count the steps (one for each generation) up from the decedent to the nearest common ancestor of the decedent and the claimant, and then count the steps down to the claimant from the common ancestor (The count always goes up.. i.e. dont start subtracting when you count down from the nearest ancestor to the claimant). 3. Degree of Relationship with a Parentelic Tiebreaker Approach: a. Step 1: Determine the degree of relationship of the possible takers. Those of closer degree (lower number) take to the exclusion of those of a higher, more remote relationship. b. Step 2: If there are multiple takers sharing the lowest degree of relationship, under the parentelic tiebreaker, those in closer parentelic/collateral lines take to the exclusion of those in the more remote parentelic/collateral lines. ii. RULE: When the intestate decedent is survived by a descendant, the decedents ancestors and collaterals do not take. iii. Definition: Collateral relatives = Persons who are related by blood to the decedent, but who are not descendants or ancestors (e.g. siblings a.k.a. first-line collaterals, cousins) iv. UPC Approach If the decedent is not survived by a spouse, descendant, or parent, then intestate property passes to brothers/sisters and their descendants. The descendants of any deceased brothers and sisters (nephews and nieces) take by representation, usually in the same manner as the decedents descendants. (UPC 2-106(c) calls for representation per capita at each generation). v. Half-Bloods (p. 96) 1. Half-bloods are relatives who share only one common parent, as opposed to the traditional relationship where siblings share both parents. 2. At Common Law: Only whole-blood relatives could inherit from intestate parents 3. UPC and modern trend majority: Under UPC 2-107, half-bloods are treated the same as whole-blood relatives they can inherit equally (e.g. if A and B have the same mother, but different fathers, and the mother dies intestate, A and B have equal rights to inherit from W) 4. IL doesnt distinguish btwn whole and half-bloods (755 ILCS 5/2-1) Transfers to Children a. Meaning of children 1.

Chapter 2 Intestacy: An Estate Plan By Default

16 of 120

i. Adopted Children 1. 755 ILCS 5/2-4 2. Hall v. Vallandingham (p. 97) 3. F/P a. Earl J. Vallandingham, died in 1956 and was survived by his widow, Elizabeth, and their four children, appellants. Two years later, Elizabeth married Jim Walter Kilgore, who adopted the appellants. In 1983, Earls brother, William Jr., died intestate with his sole heirs being his surviving brothers and sisters and the children of his brothers and sisters who predeceased him. After the Inventory and First Accounting of Williams estate were filed, appellants alleged they were entitled to a distributive share of their natural uncles estate that their natural father would have received if he had survived William. The Orphans Court transmitted the issue to the Circuit Court for St. Marys County and the tribunal determined the four natural children of earl were not entitled to a distribution from Williams estate because of their adoption by Kilgore. Appellants, unwilling to accept the courts disposition, appeal. 4. I a. Whether the trial court erred in denying the appellants the rights to inherit through their natural paternal uncle, due to the fact appellants were adopted as minors by their stepfather after the death of their natural father? 5. R/A/H a. HELD: No. Judgment affirmed. b. RULE: Under the general rule, the adopting parent steps into the shoes of the natural parent of the same gender, and the parent-child relationship with the natural parent is completely severed. c. The court held that because Est. & Trusts Art. Section:1-207(a) eliminates the adopted childs right to inherit from the natural parent, it also abrogated the right to inherit through the natural parent by way of representation. d. The right of inheritance was removed by the Maryland Legislature in 1963 when it declared: Upon entry of a decree of adoption, the adopted child shall lose all rights of inheritance from its parents and from their natural collateral or lineal relatives. e. Absent clear legislative intent, adopted children should be no better off than nonadopted children and should have only two parents from whom they can inherit. ii. Adoption by Relative of Parent (2008 Amendments to the UPC): A parent-child relationship exists between adopted child and the adoptive parent (UPC 2-118(a)), but not between an adopted child and the childs genetic parents (UPC 2-119(a)) 1. A parent-child relationship exists between an individual who is adopted by the spouse of either genetic parent and (1) the genetic parent whose spouse adopted the individual; AND (2) the other genetic parent (but only for the purpose of the right of the adoptee or a descendant of the adoptee to inherit from or through the other genetic parent) (UPC 2-119(b)) 2. Where a child is adopted by a relative of either natural/genetic parent, or the spouse or surviving spouse of a relative, the child retains the right to inherit from and through BOTH natural/genetic parents (UPC 2-119(c)) 3. IL Law: 755 ILCS 5/2-4 (blue sup p. 6) 4. R.3d Property (Wills) - 2.5 (CB p. 100): General rule adopted child has parent-child relationship w/ the adoptive parents, not his/her natural parents. iii. Post-death adoption: Under the 2008 revisions to the UPC, where a child is adopted after the death of both natural/genetic parents, the child retains the right to inherit through BOTH natural/genetic parents. (UPC 2-119(d)).

Chapter 2 Intestacy: An Estate Plan By Default

17 of 120

iv. Adoption of Adults: As a general rule, adopted adults are treated the same as adopted children for inheritance purposes. 1. Minary v. Citizens Fidelity Bank & Trust Co. (p. 103) 2. F/P a. Amelia S. Minary, died in 1932, leaving a will that devised her residuary estate in trust, to pay the income to her husband and three sons, James, Thomas, and Alfred, for their respective lives. The trust was to terminate upon the death of the last surviving beneficiary, at which time the corpus was to be distributed to decedents then surviving heirs, according to the laws of descent and distribution then in force in Kentucky, and if no such heirs, then to the First Christian Church, Louisville, Kentucky. b. Minarys husband died, then James died without issue, and then Thomas died leaving two children. In 1934, the only surviving son, Alfred, married Myra, respondent, and in 1959 adopted her as his child. The trust terminated in 1963 upon Alfreds dying without natural issue. 3. I a. Is respondent is included in the term my then surviving heirs according to the laws of descent and distribution in force in Kentucky? 4. R/A/H a. RULE: An adult person may be adopted in the same manner as provided by law for the adoption of a child and with the same legal effect. b. HELD: The respondent is NOT included in my then surviving heirs c. Although the adoption technically fell within the express terms of the statute, such an adoption was a subterfuge that thwarted the remote ancestors intent. v. Adoption and Its Impact on Class Gifts 1. UPC 2-705(f) excludes a person adopted after reaching the age of 18 from a class gift that is 1) GIVEN BY someone other than the adopting parent and 2) GIVEN TO the adopting parents children, issue, descendants, or heirs; UNLESS the adopting parent was the adoptees stepparent or foster parent, or the adopting parent functioned as a parent of the adoptee before the adoptee turned 18. See also R.3d Will 14.5. 2. UPC 2-705(f) excludes a person adopted after reaching the age of 18 from a class gift to the adoptive parents children, issue, descendants, or heirs by someone other than the adoptive parent unless the adoptive parent was the adoptees stepparent or foster parent, or the adoptive parent functioned as a parent of the adoptee before the adoptee turned 18. vi. Posthumous Children 1. Typical case = child conceived before, but born after fathers death a. If posthumously born children are born to a couple that was married during husbands life, the child is a descendant of the couple. 2. General Rule: Where it is to the childs advantage to be treated as in being from the time of conception, rather than from the time of birth, the child will be so treated, if born alive 3. Common Law: Courts have established a rebuttable presumption that the normal period of gestation is 280 days (10 lunar months). If the child claims that conception dated more than 280 days, the burden of proof is on the child. 4. Uniform Parentage Act (UPA 204) establishes a rebuttable presumption that a child born to a woman within 300 (rather than 280) days after the death of her husband is a child of that husband. 5. 755 ILCS 5/2-3: Posthumous children receive the same share of an estate as if the child had been born in the decedents lifetime. vii. Nonmarital Children (Children Born out of Wedlock) 1. Common Law: Children born out of wedlock were considered illegitimate and could not inherit from either the mother or the father.

Chapter 2 Intestacy: An Estate Plan By Default

18 of 120

Modern Trend/UPC Approach: A child has a parent-child relationship with both natural/genetic parents regardless of their marital status (UPC 2-117). a. Parent-Child Relationship with Mother: Under the modern trend, a child born out of wedlock automatically has a parent-child relationship with his/her natural mother (assuming no surrogate mother) and can inherit from and through the natural mother b. Parent-Child Relationship with Father: Inheritance from and through a father usu requires proof of paternity i. E.g. Evidence of subsequent marriage of the parents; by acknowledgement by the father; by an adjudication during the life of the father; or by clear and convincing proof after his death. c. 755 ILCS 5/2-2: (see blue sup p. 5) Note: This statute governs both (1) children born out of wedlock as testators (e.g. who has rights to inherit from them), and (2) children born out of wedlock as heirs/devisees/legatees (how children born of wedlock receive distributions) viii. Reproductive Technology and New Forms of Parentage 1. Posthumously conceived children (i.e. a mother uses a fathers frozen sperm to conceive) Note this is children who are conceived after the fathers death, e.g. by artificial insemination. a. Key Issue: Should the posthumously conceived child be treated as a child of the predeceased natural parent for purposes of distributing his or her estate? b. Woodward v. Commissioner of Social Security (p. 118) c. F/P i. James Woodward married the appellant in 1993. Three years later, Woodward learned that he had leukemia. He was advised that the leukemia treatment my leave him sterile. Therefore he decided to arrange for a quantity of his semen to be preserved. Woodward died later that year. ii. In 1995, the appellant gave birth to two children. She applied two forms of social security, childs benefits and mothers benefits. iii. The Social Security Administration rejected her claim on the ground that she had not shown that the twins were the husbands children under the meaning of the Act because they were not entitled to The wife appealed to the US Dist Ct for the District of Mass, seeking decl judgment to reverse the commissioners rulinginherit under the Massachusetts intestacy and paternity laws. iv. Dist ct judge certified the question to the Mass Supreme Court d. I i. May posthumous children who are the result of artificial insemination receive social security benefits if their genes can be traced to the alleged father? e. R/A/H i. HELD: In certain circumstances, yes posthumously conceived children may enjoy the inheritance rights of "issue" (under Masss intestacy scheme) ii. RULE: As a threshold matter, the surviving parent must show that the prospective donor parent: 1. Clearly and unequivocally consented to posthumous reproduction AND 2. Clearly and unequivocally consented to the support of any resulting child 3. AND ALSO that the deceased parent is the genetic parent of the child 2.

Chapter 2 Intestacy: An Estate Plan By Default

19 of 120

2.

3.

4.

Notes: After the donor parent's death, the BURDEN OF PROOF is on the surviving parent, or the posthumously conceived child's other legal representative, to prove the deceased genetic parent's affirmative consent to both requirements Intestacy, and Construction of Wills, Trusts, and Other Instruments a. Technically, whether a posthumously conceived child qualifies as child or heir for purposes of a will, trust, or other written instrument is a question of the intent of the decedentnot a question of intestate rules. The written words of the instrument are presumed to be the best evidence of the decedents intent. But often the instrument fails to address the issue. In such cases, the modern trend is to treat the child as a child of the decedent. b. In re Martin B. (p. 126) c. F/P i. Grantor established 7 trusts for the benefit of his issue/descendants. Grantors son died six months earlier, with no children, but before he died he banked sperm and authorized his wife to use it. Using his sperm, three years later she gave birth to a son, and then two years later, to a second son. ii. The trust gave the trustees the discretion to use the principal for the settlors issue during the grantors wifes life, and upon her death, to distribute the principal to their issue. d. I i. May posthumously conceived children receive trust property? e. R/A/H i. HELD: James and Warren (the sons) are issue and descendants for all purposes of these trusts. ii. RULE: Where a governing instrument is silent, children born of this new biotechnology with the consent of their parent are entitled to the same rights for all purposes as those of a natural child. f. UPC Approach: The 2008 revisions to the UPC expressly provide that a posthumously conceived child is included in a class gift in a trust or will of a third party (someone other than the predeceased donor parent) as long as i. (1) the predeceased parent authorized the posthumous use of the genetic material in a signed writing or there is clear and convincing evidence of such consent (UPC 2-705(b) and 2-120(f)), AND ii. (2) the child is living on the distribution date or is in utero within 36 months of, or is born within 45 months of the distribution date (UPC 2-705(g)) Surrogate Motherhood and Married Couples (p. 130) a. UPC Approach: With respect to a child born to a surrogate (a gestational carrier), UPC 2-121 (2008) provides that: i. In the absence of a court order to the contrary, the surrogate does not have a parent-child relationship with the child UNLESS the surrogate is the childs genetic mother AND no one else has a parent-child relationship with the child. ii. An intended parent of the child (i.e. a person who entered into an agreement with the surrogate stating that the person would be the childs parent) has a parent-child relationship with the child. Assisted Reproduction and Same-Sex Couples (p. 132) a. UPC Approach to Parent-Child Relationship: i. A child conceived by assisted reproduction other than gestational surrogacy is in a parent-child relationship (and thus entitled to inherit by, from, or through) the childs birth mother (UPC 2-120(c))

f.

Chapter 2 Intestacy: An Estate Plan By Default

20 of 120

ii. There can also be a parent-child relationship with another person if the other person either A) consented in writing to assisted reproduction by the birth mother with the intent to be the other parent of the child or B) functioned as a parent of the child within two years of the childs birth (UPC 2-120(f)) 5. Advancements (p. 133) a. 755 ILCS 5/2-5: Advancements (blue supp p. 7) b. Advancements doctrine addresses issue whether inter vivos gifts a decedent made to an heir should count against the heirs share of the decedents INTESTATE estate. i. Common Law: Any lifetime gift given by the testator/decedent to a child was presumed to be an advancement essentially a prepayment of the childs intestate share. The burden of proof was on the child to prove that the transfer was intended as an absolute gift, not to be counted against his share of the estate. 1. Hotchpot: All inter vivos gifts to the child are added back (on paper) into the parents probate intestate estate to create the hotchpot. Then the hotchpot is divided equally among heirs 2. Ex: D died intestate w/ 3 children, A, B, and C. D gave A $25,000 in inter vivos gifts. D gave A $50,000, and D gave C $75,000. Then D died with a $150,000 intestate estate. What do the kids get? a. The Hotchpot is $300,000: The Advancements to A, B, and C add up to $150,000. That is added to Ds intestate estate of $150,000. So the total hotchpot is $300,000. b. Then, A, B, and C each get an even split--$100,000 each total. So, from the estate, A gets $75,000, because A had already had gotten $25,000 in advancements (pre-payments) in life. B gets $50,000 from the estate; C gets $25,000 from the estate. So here Estate + Advancement = $100,000. ii. Modern/UPC Approach: Inter vivos gifts do not count as an advancement UNLESS a writing expressly indicates that the donor wants the gift to be an advancement. UPC 2-109 (see CB p. 135): Requires writing see also Emanuels p. 46 b. Bars To Succession i. There are situations when an otherwise eligible taker is barred from taking a share of an intestate estate. ii. Homicide: Where a party (a killer) who otherwise is entitled to take from a decedent kills the decedent, the killer (i.e. the now-DQed taker) cannot take. The equitable principle that one should not profit from ones own wrongdoing 1. IL Law (755 ILCS 5/2-6) (blue supp p. 7) IL law says the same thingsays the same thing 2. In re Estate of Mahoney (p. 145) 3. F/P a. On May 6, 1961, Howard Mahoney, decedent, died intestate of gunshot wounds. b. His wife, Charlotte Mahoney, appellant, was tried for the murder of decedent and was convicted of manslaughter by a jury in March 1962. Appellant is currently serving a sentence of not less than 12 nor more than 15 years at the Womens Reformatory in Rutland.

Chapter 2 Intestacy: An Estate Plan By Default

21 of 120

c.

d.

e. 4. I a. 5. R/A/H a. b.

Decedent left no issue, but was survived by appellant, and his mother and father. His father, Mark Mahoney, was appointed administrator of his estate, which amounts to $3,885.89. The probate Court for the District of Franklin entered a judgment order decreeing the residue of decedents estate, in equal shares, to decedents mother and father. Appellant appeals from the judgment order and decree. Can a widow convicted of manslaughter in connection with the death of her husband inherit from his estate?

6.

7.

8.

HELD: Yes. Decree reversed and cause remanded. RULE: The rule should be drawn between VOLUNTARY and INVOLUNTARY manslaughter. i. Here, the conviction did not indicate the degree the case was remanded to determine the degree of manslaughter. c. Legal title passes to the slayer but equity holds him to be a constructive trustee for the heirs or next of kin of the decedent d. The probate court was without jurisdiction to impose a constructive trust on the estate in the hands of the appellant. The jurisdiction over charging the appellant with a constructive trust on the estate of Mahoney lies in the court of chancery, and not in the probate court. e. The principle that one shouldnt be able to profit thru their wrong must not be extended to every case in which a killer acquires property from his victim as a result of the killing. Statutory/UPC Approach: A majority of jurisdictions and the UPC have an express statute providing that a killer shanll not take from his/her victim (UPC 2-803). Under 2803, the killer is treated as disclaimed the property (under UPC 2-1106). Most statutes treat the killer as having predeceased the victim. a. NOTE: UPC 2-803 also includes that a killer shall not take even nonprobate property Intentional and Felonious Killing: The general rule (both case law and statute) is: In order for killing to bar the killer from taking (either outright or through the constructive trust), the killing must be intentional and felonious. a. Manslaughter: Must distinguish btwn voluntary and involuntary manslaughter (see Mahoney above, p. 21). b. Self-Defense: Killing in self-defense is not felonious and does not trigger the doctrine c. Assisted suicide: Mercy killings and assisted suicides technically are intentional and felonious and come within the scope of the homicide doctrine. i. However, whether the doctrine SHOULD include such acts is greatly debated. Burden of Proof: Whether a killer takes from his/her victim is a CIVIL issue, not a CRIMINAL issue. a. A criminal conviction has res judicata effect upon the civil issue (UPC 2-803(g)) i. i.e. res judicata = [Latin, A thing adjudged.] A rule that a final judgment on the merits by a court having jurisdiction is conclusive between the parties to a suit as to all matters that were litigated or that could have been litigated in that suit. b. BUT an acquittal is not the final word because the burden of proof in a criminal case if proof beyond a reasonable doubt, while the burden of proof in a civil case is merely preponderance of the evidence. (i.e. the criminal burden is higher than the civil burden)

Chapter 2 Intestacy: An Estate Plan By Default

22 of 120

i. E.g. if the defendant is acquitted on homicide charges, but civilly found liable for the decedents intentional and felonious wrongful death (e.g. voluntary manslaughter?), the killer is still barred from taking. Remedy: a. General Rule: If the doctrine applies, then treat the killer as if he/she predeceased the victim b. UPC Approach: UPC 2-803 says to treat the killer as having disclaimed the property (this is not exactly the same as predeceasing); though depending on statute, the effect of UPC and other statutory approaches might be similar 10. Do the Killers Issue Get To Take? a. Jurisdictions are split as to whether the homicide doctrine applies to the killers issue (who even cares about this) b. In Cali, the question hinges on whether or not the victim died testate or intestate c. UPC Approach: The UPC treats the killer as if he/she disclaimed the property (UPC 2-803), which ARGUABLY permits the killers issue to take the killers share under anti-lapse and the per stirpes/per capita doctrines (if they would otherwise qualify to take under those doctrines) 11. Scope of Doctrine (Probate and Nonprobate): a. UPC 2-803 says the homicide doctrine applies to probate AND nonprobate property b. Joint Tenancy If the victim and killer held property together in jt tenancy, then by operation of law, the joint tenancy is converted into tenancy in common. i. The killer keeps his/her interest, and the victims interest is distributed as if the killer predeceased the victim. c. Other Statutes: Where the statute expressly covers probate property only, arguably, a constructive trust should be imposed on the nonprobate property i. This is based on the equitable principle that one should not benefit from ones own wrongdoing. The constructive trust would be in favor of those who would have taken if the killer had predeceased the victim. iii. Abandonment/Elder Abuse: Some states have doctrines that bar a would-be taker from receiving if the taker is guilty of misconduct short of homicide. 1. Some states bar the taker if he/she is guilty of abandonment 2. Some states bar the taker if he/she is guilty of elder abuse a. Elder abuse involves acts that amount to physical abuse, neglect, or fiduciary abuse of the decedent while he/she was an elder or dependent adult 3. Some states bar parents from taking from a child decedent if the parent refused to support the child 4. Some states (including IL) bar inheritance from children or elderly relatives who were abused by the heir 5. The Chinese System and Other Conduct-Based Restrictions on Inheritance (p. 151) a. Chinas inheritance scheme punishes bad behavior and rewards good behavior b. Looks at the conduct of those surrounding the decedent to determine who is worthy to take, and who is not c. Cons: The approach is highly fact-sensitive, time consuming, and expensive this may create a potential/incentive for fraud d. Pros: It rewards contributions to the decedents welfare, even by those outside the nuclear family, and penalizes neglectful behavior/lack of care. iv. Disclaimer: Disclaimer occurs when an heir or devisee declines to take the property they are entitled to (i.e. a testamentary gift requires 1) intent by the decedent; 2) delivery of the gift (e.g. 9.

Chapter 2 Intestacy: An Estate Plan By Default

23 of 120

by will?); and 3) acceptance. Waiver is the recipients right to decline step 3, acceptance of the gift) 1. The most common motivations for disclaiming: Avoid taxes (e.g. avoiding estate taxes) and avoiding creditors 2. How are Disclaimers Treated?: General Rule: The disclaiming party is treated as though he/she predeceased the decedent. The property is distributed to the next eligible taker under the applicable rules of the state/jurisdiction. 3. Benefits to Disclaiming: Several reasons a. Redistribute Property: Disclaimers can be used to adjust who takes and how much they take i. Example: D dies intestate, survived by a spouse and 2 children. If the children are both adults, they may disclaim their interests to increase the share going to the surviving spouse (e.g. if the children have no surviving issue) b. Avoid Gift Tax Consequcnes: If one accepts the property and then gives it to the next taker in line, the gift may be subject to gift taxes. BUT if one simply disclaims, the next taker in line receives the disclaimed gift, with no taxes. c. Avoid Creditors: Generally, creditors are entitled to reach any transferable property that the debtor holds. If an heir/devisee is facing creditors claims, such that any inheritance or devise would, for all practical purposes, go directly to the creditors, the heir or devisee can elect to disclaim the property in question to avoid the property going to the creditors. If the taker disclaims, the legal significance is that the disclaimer is tantamount to rejecting the gift. If the gift is rejected, then it was never accepted, so the taker never had a property interest in the property in question. If the taker never held a property interest in the property, the takers creditors never had a right to reach it (but the plot thickens) i. Federal Government as Creditor: All of the above sounds nice, but if the GOVERNMENT is your creditor of the disclaimant (e.g. for tax purposes or under Medicaid), the property disclaimed often is subject to the claim of the federal govt. ii. It doesnt matter how you try to work the system, the government will get its money from you, or whoever you try to pass the property to. (see Drye, below). 4. Drye v. United States (p. 155) 5. F/P a. Irma Deliah Drye died intestate, leaving her son, Rohn Drye (Drye), as the sole heir to her $233,000 estate. Before Irma died, Drye ran up a $325,000 tax bill the IRS filed tax liens against all of Dryes property and rights to property b. To keep his mothers estate away from the IRS, Drye disclaimed his interest. This allowed the entire estate to pass to his daughter, Theresa, who was next in line under the applicable state intestacy statute c. Theresa used the funds to set up a trust under which appellant was a beneficiary under the Trust, the assets were shielded from creditors seeking to satisfy the debts of the Trusts beneficiaries (i.e. shielded the estate against the IRS) 6. I a. Was Dryes disclaimer effective to pass the property to his daughter free of the federal tax lien? 7. R/A/H a. HELD: No because Drye was able to determine who would receive the property (either himself if he chose to accept, or his daughter if he chose to

Chapter 2 Intestacy: An Estate Plan By Default

24 of 120

disclaim), he held property or a right to property therefore, the property was subject to the IRS lien 8. Disclaimers to Qualify for Medicaid (p. 156) refer to Federal Government as Creditor, above 9. Scope of Disclaimer Statutes: a. BE CAREFUL see if the statute applies to only to Probate property (the Traditional Approach) or if it also applies to Nonprobate property (the Modern Trend) 10. Execution Requirements a. Most disclaimer statutes have technical rules requiring that the party disclaiming must do so 1) in writing 2) within 9 months of decedents death. v. See also IL Statutes 1. 755 ILCS 5/2-1 to 9 (Descent and Distribution) 2. 755 ILCS 5/3-1 to 2 (Simultaneous Deaths) 3. 755 ILCS 5/9-1 to 10 (Letters of Administration) 4. 755 ILCS 5/13-1 to 5 (Public Administrators, Guardians and Conservators) 5. 755 ILCS 5/26-1 to 2 (Appeals) 6. 755 ILCS 5/27-1 to 9 (Miscellaneous) 7. 755 ILCS 5/30-1 to 2 (RepealSavings ClauseEffective Date) 8. 755 ILCS 15/0.01 to 1 (Safety Deposit Box Opening Act)

Chapter 3 Wills: Capacity and Contests


1. Mental Capacity a. The Test of Mental Capacity i. R.3d of Property: Wills and Other Donative Transfers 8.1(b): To be competent to make a will, the testator must be 1. 1) an adult (age 18 or older) and 2. 2) capable of knowing and understanding: a. The nature and extent of his/her property b. The natural objects of his/her bounty c. The disposition that he/she is making of that property, and d. Of relating these elements to ano another and forming and orderly desire to dispose of the property 3. 755 ILCS 5/4-1 says anyone 18 yrs+ and of sound mind and memory can write a will. b. Policy Justifications for Capacity Requirement i. Public opinion ii. Assures testators that the intent expressed when they have capacity will be protected from the risk that they may lose capacity later in life (e.g. memory loss, etc) iii. Protects family members (who stand to gain from testators gifts) rd iv. Protects the testator from unscrupulous 3 parties who may try to take advantage of a testator of weakened capacity (againprotecting intent) v. Etc. c. Powers of Appointment Exercisable by Will: A power of appointment may be exercised by anyone 18 years of age, UNLESS the will expressly requires someone older. (755 ILCS 5/4-2) i. Insurance and Death Benefits Payable to Testamentary Trustee: IL Law allows a trustee to be named as a beneficiary (755 ILCS 5/4-5) d. Temporal Application of Capacity Rule: The testator must have the requisite capacity at the time he or she performs a testamentary act (executes or revokes a will) i. Lucid Moment: If a person who otherwise lacks testamentary capacity executes a will during a lucid moment, the will is valid even though the testator lacked capacity for some time before and/or after executing the will 1. Note: The testators capacity immediately before and after executing the will is relevant to the issue of capacity at the moment of execution.

Chapter 3 Wills: Capacity and Contests

25 of 120

e.

f. g.

Presumption of Capacity: By default, there is a STRONG presumption that the testator has testamentary capacity. i. IL Statute assumes testamentary capacity Standing: General Rule: A party has standing to contest the validity of a will, or provision in a will, only if that party will financially benefit from a successful challenge. Burden of Proof i. Majority Approach: Once a proponent offers prima facie proof that a will was duly executed, it creates a rebuttable presumption that the testator had testamentary capacity, and the burden is on the contestant to prove a lack of testamentary capacity (see Wilson v. Lane, below p. 26; see also Breeden v. Stone below, p. 29) ii. Minority Approach: The will proponent bears the burden of proving testamentary capacity. See In re Estate of Washburn (below, p. 26). iii. In re Estate of Washburn (p. 159) iv. F/P 1. Katherine Washburn, testatrix, executed 3 wills. a. Will #1 (Oct 1986): Made several $1,000 bequests to several named individuals; provided that her home, her personal effects, and the residue of her estate should go to her sister, Margaret Fay, or in default of that, to her niece, Catherine Colonna (petitioner) b. Will #2 (Mar 1992): Left $1,000 bequests to certain named individuals; $5,000 to the respondent, her caretaker, and companion; and the residue to the petitioner c. Will #3 (Apr 1992): Left $5,000 bequests to the petitioner and another individual and provided that the respondent receive the residue, which included testatrixs home and personal estate 2. Petitioner (Colonna) challenged the testatrixs testamentary capacity. The probate ct held a hearing on proof of the will in solemn form. 3. The ct found that the testatrix was suffering from Alzheimers diseases at the time of execution of the April 1992 will she could not recollect the property she wished to dispose of, nor could she understand its general nature v. I 1. Did the testatrix have the testamentary capacity to bestow her property by will? vi. R/A/H 1. HELD: No, the testatrix did not have testamentary capacity 2. RULE: Every person is presumed sane until there is some evidence shown to rebut that presumption 3. Petitioner offered sufficient evidence to rebut the presumption of capacity. Medical testimony established Alzheimers disease. Further testimony established testatrixs confusion, forgetfulness, and a lack of competency at the time in question. vii. Wilson v. Lane (p. 161) viii. F/P 1. The will in question distributed Greers property equally to 17 beneficiaries, 16 of whom are blood-relatives to Greer. The only non-relative beneficiary is Lane, who spent much of her time caring for Greer before her death. 2. Supporting testimony from drafting atty and other friends established that, at the time the will was signed, Greer was mentally competent, and that she emphatically selected every beneficiary named. 3. The caveators attempted to show that Greer was eccentric, aged, and peculiar in the last years of her life. 4. Jasper County Superior Court found that Greer lacked testamentary capacity at the time she executed her will, BUT the trial ct granted Lanes motion for judgment notwithstanding the verdict. ix. I

Chapter 3 Wills: Capacity and Contests

26 of 120

2.

HELD: Trial court affirmed RULE: A person is mentally capable to make a will if she has sufficient intellect to enable her to have a decided and rational desire as to the disposition of her property. 3. None of the evidence showed that Greer was INCAPABLE of forming a decided and rational desire as to the disposition of her property. She may have been peculiar, or eccentric, but that didnt make her incapable of making a rational decision. 4. NOTE: Sufficient Intellect is somewhat of a weak standard to meet xi. Dissent (Carley) 1. Though he agrees that Greer had capacity, the JURY found that she did not, and the court should honor that. 2. When the evidence is construed most strongly in support of the jurys verdict in favor of the Caveators , it authorized the finding that Ms. Greer did not have sufficient intellect to enable her to make a decided and rational decision regarding disposition of her estate. h. Comparison of Testamentary Capacity to Other Capacity i. Testamentary Capacity vs. Capacity to Contract: Contractual capacity requirement is higher than testamentary capacity 1. Rationale: Contractual capacity is concerned with one improvidently disposing of assets during ones lifetime. The risk is that the person may become destitute and depend on the state for support. The state has a legitimate interest in preventing costs of caring for people who could have cared for themselves. Testamentary capacity, on the other hand, is concerned with the level of capacity necessary to transfer ones assets at the time of death. The state has less of an interest in the possible consequences of testamentary transfers 2. Appointment of conservator: If a person lacks contractual capacity, a conservator is appointed to handle the persons affairs. Because testamentary capacity is lower than contractual capacity, the mere appointment of a conservator does not mean that the person necessarily lacks testamentary capacity. More facts would be necessary to determine if the person lacked testamentary capacity. ii. Testamentary Capacity vs. Marriage Capacity: Testamentary is HIGHER than the capacity to marry. The right to marry is a fundamental right it has special status that limits the states ability to regulate it. i. Attorneys Ethical Duty: A lawyer has an ethical duty to assess the capacity of an individual before drafting his/her will. It is unethical to draft a will for a person who lacks capacity. i. But the atty is authorized to rely on his/her judgment in determining testamentary capacity j. Defects in Capacity: Even if a person has general testamentary capacity, a person may suffer from a defect in capacity that may invalidate all or part of the will. There are 4 possible defects: i. Insane Delusion ii. Undue Influence iii. Fraud iv. Duress k. Remedy: If the testator suffers from a defect that causes him or her to dispose of his/her property in a way that the testator would otherwise not have, the general rule is the court will strike as much of the will as was caused by the defect. i. Capacity Thresholds (p. 166) 1. It takes more capacity to will than to contract. Policy = protection because a will cannot be reviewed by the testator once hes dead 2. Oddly, legal capacity to make a will requires greater mental competence than to marry. Policy = if youre happy, youre happy. State doesnt care. Plus, marriage is closer to a fundamental right, so the govt doesnt want to touch it. Insane Delusion

1. x. R/A/H 1. 2.

Did Greer have testamentary capacity?

Chapter 3 Wills: Capacity and Contests

27 of 120

a.

b.

c. d.

Definition: A delusion is a false sense of reality to which a person adheres despite all evidence and reason to the contrary i. Distinction from Psychological Delusion: Insane delusion is a LEGAL PRINCIPLE, not a psychological one. ii. Distinction from Mistake: A mistake is merely incorrect knowledge; a mistake is correctible if the testator is told the truth. In contrast, an insane delusion is not correctible, no matter how much you tell the testator the truth 1. Common Law: Courts do not reform or invalidate wills because of mistake; but they do invalidate or reform wills or provisions of wills resulting from an insane delusion 2. Minority View (Any Factual Basis Approach): If there is any factual basis at all for testators delusion, it is not deemed insane a. Policy Note: This approach protects the testators intent if ANY basis at all exists for the testators view, the jury has no leeway to call it an insane delusion; this honors intent 3. Majority View (Rational Person Test): A delusion is insane even if there is some factual basis for it if a rational person in the testators situation could not have drawn the conclusion reached by the testator. (see R.3d of Property: Wills and Other Donative Transfers 8.1) a. Policy Note: This does not protect testators intent. If the jury thinks the testators belief is too bizarre, the jury can substitute its own belief, which can lead to reformation of the will. iii. Traumatic Events: (We didnt really talk about this, but) Although not explicitly part of either the Minority or Majority tests, case law tends to indicate a rule: If a traumatic event in a persons life alters how he/she views the world, the contestant has a better chance of convincing the court that the testator suffered from an insane delusion Causation: i. Majority (but for causation): For insane delusion contests, in most jurisdictions, the contestant must show that (1) the testator worked under an insane delusion, AND (2) the will (or some part of it) was a product of the insane delusion. (i.e. the delusion materially affected the disposition) 1. i.e. but for the insane delusion, the testator would not have made the will as it appeared (would not have disposed of property as he/she did) 2. Breeden represents the majority approach to causation: In Breeden (below p. 29) , the court upheld the will because Breedens insane delusions did not materially affect or influence the wills provisions. ii. Minority Approach (Might have caused): This test requires the contestant to prove only that the insane delusion might have affected the disposition of property (lower/easier standard to meet) 1. In re Honigmans Will represents the minority approach to causation: A minority of courts apply a lower standard for causation the court denied probate to a will on the ground that its dispositive provisions might have been caused or affected by the testators insane delusion. (see Honigman, below p. 29) In re Strittmater (p. 169) F/P i. Louisa F. Strittmater, decedent, was born in 1896 and lived with her parents until their death in 1928. her admiration and love of her parents persisted after their death until at least 1934. However, four years later, she wrote derogatory comments about her mother and father in the margins of books and on photographs of her parents. In the opinion of the only medical witness, Dr. Sarah D. Smalley, who was decedents physician all of her adult life, decedent suffered from paranoia of the Bleuler type of split personality. In 1925 decedent became a member of the New Jersey branch of the National Womens Party and worked for them as a volunteer in the New York office from 1939 to 1941. During this period she spoke about leaving her estate to The Party and on October 31, 1944, she executed her last will, putting this intention into effect. A month

Chapter 3 Wills: Capacity and Contests

28 of 120

later she died. Decedents will was denied admission to probate based on the ground that decedent was insane. The appellants, The National Womens Party, appeal. e. f. I i. Whether the decedents will was the product of her insanity? R/A/H i. HELD: Yes, the will was the product of her insane delusion. Judgment affirmed. ii. Decedents disease seemed to have become well developed by 1936. She had been a member of the Womens Party for eleven years at that time but the evidence doesnt show that she had taken great interest in it. The court believed it was her paranoiac condition, especially her insane delusions about the male, that led her to leave her estate to the National Womens Party. iii. The Master who heard the case in court found that the decedents comments she had written in books and on photos of her parents demonstrated incontrovertibly her morbid aversion to men and feminism to a neurotic extreme. The court found that characterization was not strong enough to conclude the decedent suffered from an insane delusion. g. Breeden v. Stone (p. 171) h. F/P i. Testator Spicer Breeden regularly abused alcohol and cocaine and suffered from delusions, mood swings, and paranoia, including that everyone was spying on him (including his friends, family, and repairmen working in the area; and as it turns out, one of his friends, Crow, was an FBI informant. Whodathunkit?). Testator cut off his TV antennae and his cable service because he thought the FBI could monitor his conduct through the TV screen. After a weekend of drinking and getting high, while driving 110 MPH, he hit a another car, killing the driver (testator did not stop, but switched cars and continued to party). ii. When the police came to question him about the accident (2 days later), he barricaded himself inside his house, hand-wrote a will that excluded his family (from whom he was estranged) and left all of his property to his friend Sydney Stone. iii. Breedens family challenged Stones inheritance i. I i. Did Breeden suffer from a lack of testamentary capacity (insane delusion)? j. R/A/H i. HELD: No, at least not enough to defeat the will. Decedent was of sound mind i.e. the testator suffered from an insane delusion, but there was not sufficient evidence that it materially affected the provisions of his will. ii. RULE: 1. Rational Person Analysis (Majority): Under the majority approach, it is easy to conclude that a rational person could not hold the delusions/paranoia against the govt, and his friends and family. 2. Any Factual Basis Analysis (Minority): Under the minority approach, it came out in a later proceeding that one of the testators friends may have been an FBI informant (which would give a factual basis for decedents paranoia against the govt). But it is unclear whether the probate court was aware of this or whether it would have found that fact alone supported testators delusions and paranoia. iii. k. In re Honigmans Will (p. 178) l. F/P i. The testator, after coming to believe that his wife was having an affair, left his wife only the minimum necessary to satisfy her statutory forced share, leaving the rest to his brothers and sisters. m. I i. Insane delusion? n. R/A/H i. HELD: Yes the court DENIED probate to the will because its provisions might have been caused or affected by the testators insane delusion

Chapter 3 Wills: Capacity and Contests

29 of 120

3.

NOTE: This is a lower standard than in Breeden under this analysis, Breedens will probably would have been reformed ii. RULE: Might have been iii. The Honigman court assumed causation by putting the burden on the proponent to show that an unnatural disposition, which could arguably be caused by the insane delusion, was not in fact a product of that delusion. iv. The brothers and sisters failed to meet this burden therefore, the wife won. o. Dead Mans Statutes (p. 179) i. NOTE: Dead Mans Statutes are good law only in a minority of states ii. Definition: Dead Mans Statutes prohibit an interested party in the will from testifying about a decedents oral statement in support of a claim against the decedents estate iii. Policy/Pros: Dead Mans Statutes are meant to protect the decedent, because the decedent is dead; he/she cannot refute the testimony of the surviving party iv. Cons: But, Dead Mans Statutes effectively bar even potentially legitimate testimony/claims against the decedents estate. v. Dead Mans Statutes are Unpopular: Some states have totally repealed Dead Mans Statutes. Basically, theyre not good law. But we still need to study them. Word. Undue Influence a. Introduction/Definition i. Undue Influence is hard to define ii. Traditional View: Undue Influence = coercion; when the will of the person who becomes testator is coerced into doing that which he/she does not desire to do, that is undue influence iii. R.3d Wills 8.3 Undue Influence, Duress, or Fraud 1. (a) A donative transfer is INVALID to the extent that it was procured by undue influence, duress, or fraud. 2. (b) A donative transfer is procured by undue influence if the wrongdoer exerted such influence over the donor that it overcame the donors free will and caused the donor to make a donative transfer that the donor would not otherwise have made. 3. Notes: The wrongdoer need not be present when the will is executed to have exerted undue influence 4. Policy Notes: The doctrine of undue influence protects against overreaching by a wrongdoer seeking to take unfair advantage of a donor who is susceptible to such influence, based on e.g. age, inexperience, dependence, physical/mental weakness, or other. iv. How to Prove Undue Influence (Traditional Approach) 1. Direct evidence (yeah right): Prima facie case 2. Circumstantial Evidence: For circumstantial evidence to raise an inference of undue influence, the contestant must prove a. (1) Susceptibility: The donor was susceptible to undue influence; b. (2) Opportunity: The alleged wrongdoer had an opportunity to exert undue influence; c. (3) Motive: The alleged wrongdoer had a motive for exerting undue influence; and d. (4) Causation: There was a result appearing to be the effect of the undue influence i.e. the undue influence caused the testator to dispose of his/her property in a way that the testator would not have done otherwise? e. v. Burden of Proof: Under this approach, the party challenging the will bears the burden of proof. vi. Presumptions and Burden Shifting (Majority Approach): Because direct evidence is rare, and the DEFENDANT is in the best position to present whatever evidence IS available, most jurisdictions use a burden-shifting approach to undue influence.

1.

Chapter 3 Wills: Capacity and Contests

30 of 120

1.

2.

3. 4.

5.

6.

7. 8.

9.

Presumption of Undue Influence: If the elements of the burden-shifting doctrine are met, a presumption of undue influence arises, and the burden shifts to the defendant to rebut the presumption Rule: The presumption of undue influence arises if: (1) There was a confidential relationship between the defendant and the testator; (2) the defendant receives the bulk of the testators estate; and (3) the testator was of weakened intellect Estate of Lakatosh (Burden Shifting) (p. 182) (Burden Shifting) F/P a. In March 1988, Roger Jacobs, appellant, befriended Rose Lakatosh, decedent, who was then in her seventies and living alone. Appellant assisted decedent around her house and drove her to various appointments. b. A few months after they met, appellant suggested that decedent give him power of attorney. On November 11, 1988, decedent executed a power of attorney as well as a new will that left all but $1,000 of her $268,000 estate to appellant. Appellants second cousin drafted the will, and a tape recording of the execution ceremony evidenced that while decedent didnt entirely lack competence, she had a weakened intellect. c. (We presume that a family member or someone other than Jacobs challenged the will) I a. Did the trial court err in finding that decedents will should be revoked because appellant failed to carry his burden of proving the absence of undue influence? R/A/H a. HELD: No. Order of the trial court is affirmed appellant unduly influenced Lakatosh b. RULE: (See presumption/burden shifting rule above) c. The three elements of the test for undue influence were easily met. i. First, the confidential relationship was established by decedents dependency on appellant as well as the power of attorney. ii. Second, appellant received the bulk of the estate (he got all but $1,000 of her $268,000 estate). iii. Third, decedents intellect was weakened as evidenced by the fact she was an elderly woman unable to prevent the consumption of her assets by the appellant and she was living in filth with her bills unpaid. she had trouble remembering things and had no understanding of her estate or assets iv. Jacobs was unable to overcome the presumption of undue influence here. In re Will of Moses (p. 186) F/P a. Fannie Traylor Moses, decedent, was married three times and each of her husbands died. (because shes a man-eater) b. During her second marriage she became friends with appellant, Clarence Holland, an attorney, fifteen years her junior. After her third husband died, appellant became decedents lover as well as her attorney. This relationship continued for several years until decedent died at age 57. c. Three years before she died, decedent made a will devising almost all of her property to appellant. The will was drafted by an attorney, Dan Shell, who had no connection with appellant, and who did not tell appellant about the will. d. Decedents closest relative was her older sister, who attacked the will on the ground that of undue influence. The Chancellor found undue influence and denied probate. Appellant appeals. I

Chapter 3 Wills: Capacity and Contests

31 of 120

a. 10. R/A/H a. b. c.

Is a presumption of undue influence is overcome when independent advice and counsel is sought?

vii. viii.

ix.

x.

xi.

HELD: Not necessarily. The decree of the chancery court will be affirmed. RULE: (See presumption/burden shifting rule above) The attorneys testimony supports the chancellors finding that nowhere in the conversations with the decedent was it at all discussed the proposed testamentary disposition whereby preference was given to a non-relative to the exclusion of her blood relatives. d. There was no discussion of her relationship with appellant, nor was there discussion as to who her legal heirs might be, nor was there discussion as to their relationship to her, after it was discovered she had neither a husband nor children. Its clear from the testimony that the attorney-draftsman did no more than write down, according to the forms of law, what decedent told him. e. There was no meaningful independent advice or counsel touching upon the area in question. f. DISSENT (Robertson): The judgment of the lower court should be reversed and the decedents will should be admitted to probate. There is no testimony that indicates that appellant even knew of decedents will, much les participated in the preparation and execution of it. Furthermore, the evidence is clear that decedent executed her will after full deliberation, with full knowledge of what she was doing, and with the independent consent and advice of an experienced and competent attorney. g. OTHER NOTES: The sexual morality of the personal relationship between the decedent and the appellant is not an issue. However, the intimate nature of this relationship is relevant to the present inquiry to the extent that its existence, under the circumstances, warranted an inference of undue influence, extending and augmenting that which flowed from the attorneyclient relationship. In re Kaufmanns Will (p. 191) Significance: A will is invalid where the evidence shows that the testator did not freely and voluntarily creates his or her will because another individual exerted undue influence over his mind. F/P 1. In about 1949, Kaufman met Walter Weiss at age 39. Weiss did not have any significant material assets at the time. A year later Kaufman hired Weiss as his financial consultant. About the same time, Kaufman moved in with Weiss. Kaufman remodeled the top floor of his apartment building into an office for Walter. Weiss completed most of the domestic duties of the household, answered the mail and telephone, and paid the bills. Weiss took charge of both of Kaufmans bank accounts and investments as if they belonged to both of them. The two lived together until 1959 when Kaufman died unexpectedly. Robert executed a will before he died leaving substantially all of his property to Weiss. He also included a letter to his family stating his appreciation for having Weiss in his life, hoping that his family would be happy for him. 2. At Kaufmans death, his brother sought to have his will set aside on grounds of undue influence after suspecting that Kaufman had a homosexual relationship with Weiss. The trial court found that Weiss made false accusations against Kaufmans brother and caused Kaufman to believe that his family was resentful of his drive for independence. I 1. Does a beneficiary exert undue influence over a testator when the testator could easily have been taken advantage of, the beneficiary dominates the testator and the testator relies exclusively on the beneficiarys knowledge in making dispositions? R/A/H

Chapter 3 Wills: Capacity and Contests

32 of 120

xii. xiii.

xiv. xv.

xvi.

HELD: Yes. RULE: A will is invalid for reason of undue influence where the evidence shows that (1) the beneficiary dominated the testator, (2) the testator could be easily taken advantage of, and (3) the testator relies exclusively on the beneficiarys knowledge and judgment concerning the disposition of all material things in the testators life. 3. Here the proponent of the will planted in the testators mind that his family was obstructing his goal of independence and made false accusations against the testators brother. The testator was inexperienced, and the proponent exerted undue influence over him when he disposed of almost his entire estate to the proponent. Lipper v. Weslow (p. 193) F/P 1. Sophie Block, decedent, was married three times. From her first marriage she had one son, Julian Weslow, who died in 1949, and was the father of decedents grandchildren, Julian Weslow Jr., Julia Weslow Fortson, and Alice Weslow Sale, plaintiffs herein. 2. After the death of her first husband, decedent married a Mr. Lipper. Frank Lipper and nd Irene Lipper Dover, defendants, are the sons of their marriage (her kids from her 2 marriage). 3. After Mr. Lippers death, decedent married Max Block and there were no children from this marriage. 4. On January 30, 1956, decedent, executed a will wherein she left her estate to her two surviving children, defendants (on the Lipper side). She explicitly disinherited the Weslow side (the widow of Julian, and also Julians children). 5. Decedents will was prepared by her son Frank, an attorney, one of the beneficiaries of the will, and Independent Executor of the will. The will was witnessed by two former business associates of the decedent. 6. Plaintiffs contested the will, claiming undue influence on the part of Frank (her son, and the lawyer who prepared the will)and the jury found that decedents will was procured by undue influence on the part of defendant Lipper. The trial court entered judgment on the verdict and set aside the will. I 1. Was there any evidence of undue influence? R/A/H 1. No. The cause is reversed and rendered for defendants. 2. The will contained a paragraph that stated that the testatrix disinherited the grandchildren because they and their mother had been most unfriendly to her since the death of her son, though there was conflicting evidence as to the accuracy of this assertion. 3. Here the will and the circumstances may raise suspicion, but does not give proof of the vital facts of undue influence. All of the evidence reflected that decedent was of sound mind, strong will, and in excellent physical condition. Additionally, subsequent to the execution of the will decedent told three disinterested witnesses what she had done with her property in her will as well as the reason for it. 4. Decedents will did make an unnatural disposition of her property in the sense that it did prefer her two children over the grandchildren by a deceased son. However, the decedent had the right to do as she did and the record contains an explanation from the decedent herself as to why she chose to do such. 5. A person of sound mind has the legal right to dispose of his property as he wishes, with the burden on those attacking the disposition to prove that it was the product of undue influence. NOTES 1. Presumption Approach: It is unclear whether the presumption of undue influence would arise. 1) There was a confidential relationship (Frank, the drafter, was the testators son). 2) It is unclear whether he took the bulk of the estate (he got half). 3)

1. 2.

Chapter 3 Wills: Capacity and Contests

33 of 120

b.

it is questionable whether the testatrix was of weakened intellect. She was 81 yrs old, but she had also indicated in various forms that she intended to disinherit her grandchildren. 2. The Interested Drafter Approach: The interested drafter doctrine does not apply if the atty is related to the testator. xvii. No-Contest Clauses (p. 198) 1. No-Contest Clauses (a.k.a. in terrorem clauses) provide that a beneficiary who contests the will shall take nothing, or a token amount, instead of taking whatever was provided for in the will. a. Public Policy Considerations: The good: No contest clauses may deter frivolous suits and protect testators intent. The bad: No contest clauses may shield a partys wrongful conduct 2. Enforceability: a. Majority/UPC Approach: A majority of jurisdictions and the UPC refuse to enforce a no-contest clause if there is probable cause to support the will contest, whatever the nature of the contest. See UPC 2-517 and 3-905. i. Rationale : If there is probable cause to support the claim, the risk that the no-contest clause is being used to shield wrongful conduct is too great to ignore. b. Minority Approach: The no-contest clause is unenforceable, regardless of the amount of evidence supporting the claim, if the claim is one of forgery, revocation, or misconduct by one active in the procurement or execution of the will. xviii. Bequests to Attorneys 1. Presumption of Undue Influence (Atty Drafter): Many courts hold a presumption of undue influence arises when an attorney-drafter receives a legacy, EXCEPT when the attorney is related to testator. 2. How to beat the presumption: The presumption can be rebutted only by clear and convincing evidence provided by the attorney 3. Ethical Considerations: an interested drafter may be subject to ethical discipline. Model Rule 1.8(c) (of Model Rules of Professional Conduct) says: A lawyer shall not prepare an instrument giving the lawyer or a person related to the lawyer any substantial gift from a client,except where the client is related to the donee. Planning for and Avoiding a Will Contest i. Anticipating Which Grounds A Will Contest Will Be Brought On: 2 common grounds = 1) Lack of Capacity and 2) Undue Influence ii. Warning Signs that A Will Contest is Coming: Things to watch out for: 1. Unusual Disposition: (Perhaps the largest warning sign) e.g. the omission of a close family member or an unexplainable distinction among family members of equal relation to the testator 2. Deviation from Plan: An eccentric clients new testamentary scheme departs radically from previous plans; 3. Blended Family: The testator has multiple or blended families arising from multiple marriages; 4. Unfavorable Conditions: The testator imposes the sort of conditions on a bequest that are likely to cause the beneficiary to bristle; 5. Dispositions to Unfavorable Parties: The testator makes a disposition to a mistress or other person or group unpopular with the testators family iii. Strategies for Avoiding Will Contests (As Testator/Testators Lawyer): The lawyer should take extra precautions to prevent will contests 1. Statement of Reasons: List the reasons for dispositions in the will (this is weak) -2. Letter to the Lawyer: Have the client (testator) write a letter to the atty, saying in detail the dispositions the client wants to make. The lawyer should reply and list the

Chapter 3 Wills: Capacity and Contests

34 of 120

consequences. Having this trail of evidence shows that the testator was of sound mind in drafting the will, however it turns out. 3. Video Discussion: Between the testator and lawyer, and in front of witnesses, during which the testator explains why he wants to dispose of the property in the way written in the will. 4. Family Meeting: Testator explains his rationale to his/her family 5. Professional Examination: Have the clients level of capacity examined by a professional; have written documentation of the capacity 6. No-Contest Clause: Attempt to use a no-contest clause iv. Other Notes: 1. Fact-Intensive Suits: Although an unnatural disposition does not itself constitute lack of capacity, at a minimum, it opens the will to attack. This forces the estate to defend against the claim. Juries sometimes substitute their intent for the testators intent this is bad; it does not protect the testators intent. v. Using Inter Vivos Trusts Instead of Wills: Using an inter vivos trust increases the chances that the testamentary scheme will survive a challenge. 4. Fraud a. b. Definition: Fraud occurs when 1) the testator is deceived by a deliberate misrepresentation and 2) does that which he would not have done had the misrepresentation not been made. Rule: The misrepresentation must be made with both 1) the intent to deceive the testator and 2) the purpose of influencing the testamentary disposition. i. Note: The fraud must cause the testator to dispose of his/her property in a way that he/she would not have otherwise 2 Kinds of Fraud: In the Inducement, and In the Execution i. Fraud in the Inducement: Occurs when a person misrepresents a fact to the testator to 1) induce the testator to execute or revoke a will, or 2) to refrain from executing or revoking a will, or 3) to include particular provisions in the wrongdoers favor. 1. Key: The misrepresentation does not go to the terms of the will per se, but rather concerns a fact that is important to the testator and may induce the testator to dispose of his or her property differently in light of the misrepresentation ii. Fraud in the Execution: Occurs when a person misrepresents the nature of the document the testator is signing. E.g. A person tricks another into signing a document that is the signers will, but the signer does not realize it; or when the testator realizes he/she is signing his/her will, but the person misrepresents some portions of the will. Remedy for Fraud: A provision in a will, made because of fraud, is invalid. The remaining portion of the will stands as enforceable. BUT the ENTIRE WILL IS INVALID IF the fraud permeates the entire will or the portions invalidated by fraud are inseparable from the rest of the will. i. Remedy for Fraudulent Provisions: Strike as much of the will as was affected by fraud; if necessary, strike the whole will ii. Remedy for Fraudulent Failure to Revoke: If the fraud causes the testator not to revoke a will (or clause) that he/she otherwise would have, the appropriate remedy is to strike the will (or clause) that the testator would have revoked but for the fraud. iii. Remedy for Fraudulent Failure to Execute: If the fraud causes the decedent not to execute a will that he/she otherwise would have, although the court will not execute the will for the decedent, the court can impose a constructive trust on the parties who take the decedents probate property and order the property distributed to the parties who would have taken the property had the decedent executed the will that the misconduct prevented the decedent from executing. 1. Constructive Trust: The practical effect of this remedy is to give effect to the will that the decedent did not execute. The constructive trust remedy is rare, but courts have imposed it, where appropriate, to prevent unjust enrichment by those who would otherwise receive the decedents property. 2. Constructive Trust General Rule: For constructive trust to be appropriate, the unjust enrichment must result from some partys misconduct.

c.

d.

Chapter 3 Wills: Capacity and Contests

35 of 120

e. f.

g.

h.

Puckett v. Krida (p. 209) F/P i. Laverne Krida and Mattie Ruth Reeves, defendants, were the sole beneficiaries in Nancy Porch Hoopers will. The defendants were employed to provide care around the clock care for the Hooper. ii. The defendants were hired as nurses to provide around the clock care for the testator. After their employment, the nurses persuaded the testator that her family wanted to put her in a nursing home and were misappropriating her funds. (This was not true). The defendants were aware when they began work that the testators biggest fear was going to a nursing home. They separated the testator from all of her family, friends, and former professional contacts. At different times, the defendants made false statements to the testator and concealed facts from her. The defendants listened to Hoopers phone conversations and told her that her niece was wasting or misappropriating funds and then reimbursing herself for renting fancy cars and airline expenses. iii. The testator entrusted her niece Jean Law to handle her financial affairs. The evidence at trial showed that Law managed her affairs properly, made every effort to keep the testator out of a nursing home, and never reimbursed herself for any of the expenses. The trial court also found that the niece kept meticulous records. The defendants however, had irregular dealings with the testators money. iv. The defendants isolated the testator and controlled access to her. In regards to the testators professional and business relationships, the defendants (1) made new professional relationships for the testator and eliminated all former associations, (2) made serious decisions about the testators real property to avoid contact with her previous realtor, and (3) replaced her long time tenant with one of their family members. Furthermore, the defendant made the testators neighbors feel unwelcome and threatened the family with a lawsuit. v. The trial court set aside a deed and a will on grounds of fraud and undue influence finding that the defendants made misrepresentation to Hooper and controlled her access to legal and financial professionals. I i. Whether a presumption of fraud and undue influence is raised where the testator is in a confidential relationship with people who involuntarily isolate the testator and make misrepresentations on which the testator relied in making her will? R/A/H i. Yes. There is sufficient evidence here to support a finding of fraud and undue influence because the defendants, standing to benefit from the will, had exclusive access to the testator and control over her physical person. Furthermore, the defendants were in a confidential relationship with the testator because they were employed as nurses to provide the testator with around the clock care and one of them was her attorney-n-fact. These facts together with evidence showing the defendants told false statements which they were aware would cause the testator to exclude her family from the will. ii. A court will hold that there is sufficient evidence to support a finding of fraud and undue influence where the defendants controlled the testators physical person and told her lies which they knew would cause her to exclude certain persons from her will.

5.

Duress a. Definition: Duress is overtly coercive undue influence b. RULE: A donative transfer is procured by duress if the wrongdoer threatened to perform or did perform a wrongful act that coerced the donor into making a donative transfer that the donor would not otherwise have made (see R.3d Wills 8.3(c) c. Remedy for Transfers Made By Duress: The law INVALIDATES transfers compelled by duress d. Latham v. Father Divine (p. 210) e. F/P: i. Mary Sheldon Lyon left a will to Father Divine, two corporate defendants, and another defendant who was one of Father Divines followers. Plaintiffs claim that on several occasions after the

Chapter 3 Wills: Capacity and Contests

36 of 120

execution of her will, she expressed a desire and determination to revoke the will and create another in which plaintiffs would receive a substantial amount of her estate. Shortly before her death, Lyon had attorneys draft a new will that named the plaintiffs as legatees for a large amount, approximately $350,000. However the defendant through false representations, undue influence, and physical force, prevented the testator from executing the will. Furthermore, the defendants conspired to kill and did kill the testator without the consent of the testators relatives and before she could execute a new will. f. I i. Whether a beneficiary may be liable for undue influence and fraud where he prevents a testator from executing a will that would give a gift to another by making false representations, using physical force, and murdering the testator. g. R/A/H i. HELD: Yes. A beneficiary is liable for undue influence and fraud where he makes misrepresentations, uses force, and murders a testator to prevent him from signing a will that names another person as a beneficiary. In such a case, the beneficiary holds the property in constructive trust for the intended beneficiaries. Anything short of an equitable remedy would be unjust. ii. RULE: When an heir or devisee under a will prevents the testator from making a will or deed in favor of another, by fraud, duress, or undue influence, such heir or devisee will be deemed a trustee over the gift in favor of the intended beneficiary. iii. The validity of a will rests upon testamentary intent and meeting the proper staturory requirements. When the evidence proves an heir through misrepresentations and physical force prevents a testator from carrying his testamentary desires, the court will impose a constructive trust to carry out equity. Though the court will enforce the will, valid on its face, it will not allow an heir to benefit from such wrongful conduct. Tortious Interference With An Expectancy a. Definition: Tortious Interference w/ and Expectancy occurs when a third party (not the testator) has intentionally committed tortious conduct in the testamentary process (undue influence, fraud, or duress). rd Others who would have taken but for the misconduct can sue the 3 party for tortious interference with an expectancy. b. RULE: The plaintiff has to prove 1) the existence of an expectancy; 2) a reasonable certainty that the expectancy would have been realized but for the interference; 3) that the interference was intentional; 4) that the interference involved conduct tortious in itself, such as fraud, duress, or undue influence; and 5) damages resulting from the interference. i. NOTE: The theory of Tortious Interference MAY NOT be used when the challenge is based on the testators mental capacity c. Schilling v. Herrera (p. 215) d. F/P i. Schilling, the decedents brother, sued Herrera, the decedents caretaker, claiming tortious interference with an expectancy. ii. Decedents health began to fail, and she executed a Durable Power of Attorney naming her brother as her attorney-in-fact. Decedent lived in Fla, her bro in NJ. When the decedent was in a rehabilitation center, Herrera began to care for her, and Herrera continued to care for her when the decedent was released, visiting her as needed, and then converting her garage into a bedroom. Decedent moved in with Herrera (to the new garabe-bedroom). The brother helped pay for the care. iii. Herrera convinced decedent to execute a new Power of Attorney naming Herrera, as well as a new will naming Herrera personal representative and sole beneficiary. iv. When decedent died, Herrera did not inform the decedents brother until after she probated the will and probate had closed. The decedents brother claimed that he called Herrera during this time but that she intentionally refused to answer or return his calls. v. The trial court dismissed the brothers claim because 1) Herrera owed the decedents brother no duty, and 2) the brother had failed to exhaust his probate remedies.

6.

Chapter 3 Wills: Capacity and Contests

37 of 120

vi. . e. f. I i. Did R/A/H i. Trial Ct reversed & remanded ii. HELD: The court ruled that the brother DID state a claim because he alleged that Herreras fraudulent actions and undue influence caused the decedent to change her will, leaving all of her property to Herrera and revoking a prior will that left her estate to her brother iii. RULE: To state a cause of action for intentional interference with an expectancy of inheritance, the complaint must allege: 1) The existence of an expectancy; 2) Intentional Interference with the expectancy through tortious conduct; 3) causation (i.e. but for the interference, the testator would have made the disposition differently); and 4) damages. iv. It is the testator who was defrauded or unduly influenced by the defendant, not the claimant (brother). v. The brother satisfied the rule: 1) He expected to inherit the decedents estate upon her death; 2) Herrera intentionally interfered with his expectancy of inheritance by convincing the decedent (while ill and completely dependent on Herrera) to execute a new last will & testament naming Herrera as the sole beneficiary; and 3) Herreras fraudulent actions and undue influence caused Schilling damage by preventing him from inheriting the estate. vi. NOTES: The ct of appeals also ruled that while the claim must be brought in probate court if adequate releief is available in probate court, and exception exists where the circumstances surrounding the tortious conduct effectively preclude adequate relief in the probate court. Such circumstansces existed here because of Herreras refusal to answer/return the brothers calls g. Anna Nicole Smith and the Probate Exception to Federal Jurisdiction (p. 220) See also IL Statutes a. 755 ILCS 5/8-1 to 2 (Will Contests)

7.

Chapter 4 Wills: Formalities and Forms


1. Execution of Wills a. The Functions of Will Formalities i. Evidentiary: The Wills Act formalities serve an evidentiary function by ensuring that the document offered for probate truly reflects the testators last wishes as to who should take his or her property. ii. Protective: The Wills Act formalities serve a protective function by making it more difficult for fraudulent claims to be brought and by protecting testators intent as expressed in the properly executed will. iii. Ritualistic: The Wills Act formalities serve a ritualistic function by impressing upon the testator the finality of the act he or she is performing. iv. Channeling: The cumulative effect of the Wills Act formalities serve a channeling function by encouraging individuals to consult an attorney to draft and supervise the execution of their wills, thereby facilitating the probating the will and decreasing administrative costs. b. Attested Wills i. The Function of Formalities 1. Baron Gifts, Bargains, and Form (p. 223) 2. Lindgren The Fall of Formalism (p. 223) a. Policy: Formation/Enforcement of a Will requires more formalities than contracts this is to protect the testator from the finality of their decisions i. Wills: Assume that their seriously intended statement about their property cant be trusted. They are so weak, old, feeble, and subject to pressure that they need extraordinary protection from themselves. Their spoken words mean nothing. Their writings mean nothing unless theyre witnessed by two people

Chapter 4 Wills: Formalities and Forms

38 of 120

ii. Right To Contract: People entering into contracts are generally of sound mind. Others can rely on only spoken statements. This is because, generally, people entering into contracts are alive, and will be alive. 3. Gulliver & Tilson Classification of Gratuitous Transfers (p. 224) a. (See above about the Functions of Will Formalities. Thats what this article was about) 4. UPC 2-502 Execution; Witnessed or Notarized Wills; Holographic Wills (p. 227) a. (a) Witnessed or Notarized Wills: A will must be i. (1) In writing ii. (2) signed by the testator, or signed by someone else in the testators conscious presence and by the testators direction iii. (3) either 1. A) signed by at least 2 individuals, each of whom signed within a reasonable time after witnessing either (i) the signing of the will (as described in (2)) or (ii) the testators acknowledgement of that signature; OR 2. B) acknowledged by the testator before a notary public or other individual authorized by law to take acknowledgements b. (b) Holographic (Hand-Written) Wills: A will that does not comply with subsection (a) is VALID as a holographic will (whether witnessed or not witnessed) if the signature and material portions of the ii. Writing, Signature, and Attestation: Strict Compliance 1. The Basic Formalities of an Attested Will: a. IL Law (755 ILCS 5/4-3): Requires writing; signed by testator (or by some person in his presence and by his direction); and attestation in the presence of the testator by 2 or more credible witnesses. b. Writing: There must be a writing. Generally, oral wills are not permitted. (This is the law in all states, and in UPC 5-502(a). i. A will need not be on paper; all that is required for a writing is a reasonably permanent record of the markings that make up the will. (see R.3d Wills 3.1, cmt i) ii. Video and Electronic Wills: 1. Videotaped Wills: Generally, a videotaped will (e.g. description of dispositions) is not enough to satisfy that the will is a signed writing. No courts (to date) have upheld videotaped wills. The typical videotape scenario lacks ritualistic function, lacks direct evidence as to whether the person intended the taped statement to constitute his or her last will/testament, and has the potential for high costs. a. BUT!!! a video of the execution ceremony MAY BE ADMISSIBLE to prove due execution. 2. Electronic Wills: Generally, electronic wills do not satisfy the signed writing requirement of the traditional Wills Act. a. BUT!!! NV enacted a statute allowing probate of electronic wills under strict requirements including (1) there is a single original file and (2) there is some way to determine whether the file has been altered b. ALSO!!! An electronic will might be allowed under the Substantial Compliance or Harmless Error doctrines (see below, p. 44) c. Signature by the Testator i. Signature = anything the testator intends as his/her signature.

Chapter 4 Wills: Formalities and Forms

39 of 120

ii. There is no requirement that the testator sign his/her full name. BUT if a person intends to write his/her full signature, and does not complete it, the general rule (under strict compliance) is that the partial signature doesnt qualify as the persons signature. iii. Even a mark (e.g. an X) counts as a valid signature if the testator intends for it to be. iv. Assisted Signatures: Assisted signatures (e.g. A helps T sign his name, because Ts hand is too shaky, because of Parkinsons disease or something) is valid if the testator intends to adopt the document as a will (see R.3d Wills 3.1, cmt j) v. Signatures by Someone Other Than Testator: If someone else entirely signs at Ts discretion, and in Ts presence, the will is also valid (R.3d Wills 3.1, cmt n) vi. Acknowledgement by the Testator: Under most statutes, the testator need not sign in front of the witnesses, as long as the testator acknowledges, in front of both witnesses at the same time, that the signature already present on the document is the testators signature. vii. Order of Signatures: 1. General Rule (Traditional Approach): In general, the testator must sign/acknowledge the will before either of the witnesses do (see R.3d Wills 3.1, cmt m) 2. Modern Trend: A Witness may sign the will before the testator signs or acknowledges, as long as no one leaves the room during the execution ceremony a. Notes on UPC: The Modern Trend does not exactly line up w/ UPC because UPC expressly requires witnesses to sign after witnessing the testator perform (UPC 2-502(a)(3)). BUT if the jurisdiction has also adopted the UPC harmless error doctrine, then as long as there is clear and convincing evidence that the decedent intended the document to be his/her will, then the order of signing is irrelevant (UPC 2-503) viii. Writings Added Below the Signature: WRT attested wills, if writing appears physically below the testators signature, 2 variables must be analyzed: (1) whether the jurisdiction requires the will to be subscribed, i.e. signed at the foot or end of the document; and (2) temporally, when the writing was added. 1. Subscription Required + Writing Added AFTER Will Signed: If the will was properly executed and subscribed, then the original will is valid, and only the writing that was added later is null and void (assuming it does not qualify as a codicil a will that merely amends an existing will see below, p. 46) 2. Subscription Required + Writing Added BEFORE Will Signed: If a writing is added BELOW where the signature goes, but it was added before the will was signed, then technically, the signature is not at the end of the will. The will fails the subscription requirement a. Strict Compliance: The whole will is invalid, because the writing below the signature line fails the subscription requirement

Chapter 4 Wills: Formalities and Forms

40 of 120

d.

e.

Modern Trend: A court might be willing to simply strike the provision below the signature and hold that whatever is above the signature is valid. 3. Subscription NOT Required + Writing Added AFTER Will Signed: The added writing is not considered part of the will. The will as it existed when it was executed (signed) can be given effect, but the writing added later cannot be given effect unless it qualifies as a codicil. 4. Subscription NOT Required + Writing Added BEFORE Will Signed: The whole will is valid, including the later writing added below the signature. Attestation by Witnesses: Most jurisdictions require that the testator sign or acknowledge (that the signature on the document is testators) in the presence of at least 2 witnesses (usually both witnesses must be present with testator at the same time) i. The witnesses must sign the will, and in some jurisdictions, the witnesses must know that what they are signing is the testators will. ii. IL Law (755 ILCS 5/4-6): If any beneficial legacy/interest is given in a will to a person attesting to its execution (or to his spouse), the legacy/interst is VOID to him and all beneficiaries claiming under him, UNLESS the will is otherwise duly attested by a sufficient # of witnesses other than him. 1. Beneficiaries and creditors may also be witnesses (blue sup p. 13) iii. Delayed Attestation: Recall: If the jurisdictions statute Wills Statute requires witnesses to sign in the testators presence, the witnesses must sign at the same time as the testator, in the testators presence. BUT, if the statute does not expressly require the witnesses to sign in the testators presence, the Modern Trend/UPC allow witnesses to sign the will later (delayed attestation), even after the death of the testator, as long as the witnesses sign w/in a reasonable period of time (see UPC 2-503(a)(3)(A)). 1. What is Reasonable: Witnesses should sign while their recollection of the execution ceremony is fresh enough that they can remember whether the execution ceremony was valid. 2. eg: In NY, the statute allows 30 day delay; UPC allows whatever is reasonable The Meaning of Presence in Will Execution (p. 233) i. What does Presence mean? ii. Line of Sight Test: Presence means that the testator does not actually have to see the witnesses sign, but must be able to see them if he/she looks. 1. Blind Testator Exception: The testator/witnesses pass the line-of-sight test if the testator would have been able to see the witnesses sign from where the testator was standing/sitting if the testator had the power of sight. iii. Conscious Presence Test: The witness is in the presence of the testator if the testator, through sight, hearing, or general consciousness of events, comprehends that the witness is in the act of signing. b.

Chapter 4 Wills: Formalities and Forms

41 of 120

iv. UPC Approach/Modern Trend: UPC 2-502(a) dispenses altogether with the requirement that the witnesses sign in the testators presence. 1. Under this approach, there is only one presence requirement: that the testator sign or acknowledge the will in the presence of the witnesses. c. Strict Compliance i. Strict Compliance (Common Law Approach): Requires absolute strict compliance with each Wills Act requirement, no matter how clear the testators intent that the document be his/her last will. If there is ANY deficiency in the execution ceremony, the document is not a valid will. (see Stevens v. Casdorph, below) 1. In re Groffman (strict compliance) (p. 228) 2. F/P a. Groffman died 3 yrs after executing a will at the home of his friends, the Blocks. If held to be validly executed, Groffmans widow would share the estate with st her daughter from her 1 marriage and with one of Graffmans daughters from his first marriage. But if the will were invalid, his widow would take the entire estate in intestacy. b. Groffmans widow challenged the will. 3. I a. Was the will validly executed? 4. R/A/H a. RULE: English Wills Act of 1837 Will must be 1) in writing; 2) signed at the foot/end by either (A) the testator or (B) someone else in his presence and by his direction; 3) the signature/acknowledgement must be in the presence of 2 or more witnesses present at the same time; 4) the witnesses must attest and must subscribe the will in the presence of the testator b. HELD: The court refused to admit the will to probate (even though the ct was perfectly satisfied that the document was intended by the deceased to be executed as his will) c. Though Groffman had 2 witnesses (Block & Leigh), Groffman did not acknowledge his signature to both witnesses at the same time. Instead, Block attested in Groffmans presence, but without Leigh present; and then Leigh attested without Block present. d. This fails strict compliance 5. Stevens v. Casdorph (strict compliance) (p. 229) 6. F/P a. The Casdorphs took Homer Haskell to Shawnee Bank in Dunbar, East Virgina to execute his will. Haskell asked Debra Pauley a bank employee and public notary to witness the execution. First, Haskell signed the will. Then Pauley took the will to two other bank employees, Ms. Waldron and Ms. Ginn to sign as witnesses. Both ladies testified in their depositions they did not see Haskell sign and Haskell did not go to with Pauley to the ladies work areas. Haskell died on July 28, 1996. Haskell named Paul Casdorph his executor and left the bulk of his estate to the Casdorphs. The evidence at the trial court found that everyone in the bank knew why Haskell was there. Furthermore, the court did not find any evidence of fraud, coercion, or undue influence. The Stevens, Haskells nieces who would share in his intestate estate, filed an action to set aside the will. 7. I a. Is a will is valid where 1) the witnesses do not see the testator sign the will or acknowledge the will, 2) nor does the testator see the witnesses sign the will or acknowledge their signature, even though the execution takes place a one

Chapter 4 Wills: Formalities and Forms

42 of 120

location and the two witnesses are informed that the testator is attempting to execute his will? 8. HELD: NO. A will is not valid if the testator did not sign in it or acknowledge his signature in the presence of two witnesses, who are together, and sign their name or acknowledge their signature on the will. The law favors intestacy over intestacy but this Court has held that a valid will must have testamentary intent and execution in a manner provided by the code, concurrently b. DISSENT (Workman): The majority elevates form over substance. Courts should adhere to the underlying purpose of the statue only to prevent substitution or fraud. Here, strict adherence to formality defeats the purpose of the statue. The Court in Wade held that a narrow rigid construction of the statute should not be allowed to stand in the way of justice. c. The Court will adhere to statute when deciding whether to probate a will. Though the Court did not find any evidence of fraud or undue influence, the evidence did not necessarily rule out the possibility of fraud where the testator was in a wheel chair and escorted to the bank by the beneficiaries in his will. 9. (Interested witnesses omitted from syllabus) d. Curing Defects in the Execution of Attested Wills i. Excusing Execution Defects by Ad Hoc Exception 1. Under Traditional Strict Compliance, almost any mistake in execution invalidates the will. To avoid this harsh result, some courts excuse or correct an obvious execution 2. In re Pavlinkos Estate (p. 246) 3. F/P a. Husband and wife, Hellen and Vasil Pavlinko agreed to leave their property to each other in the event that either of them died. Hellen Pavlinko signed the will that was created for her husband and Vasil Pavlinko signed the will that was created for his wife. b. Hellens will was never offered for probate at her death. The will created for Hellen but signed by her husband was dated March 9, 1949. Vasil Pavlinko died February 8, 1957 and Hellen Pavlinko died before her husband on October 15, 1951. c. The lawyer who created the will and his secretary, Miss Zinkam both signed as witnesses. Miss Zinkam admitted she did not understand the conversation that took place between the Pavlinkos and did not speak their language. The residuary legatee named in the will in question was Helens brother. 4. I a. Whether a will is valid if it is signed by a person other than the indivudal that is referenced to in the will as being the creator of the will. 5. R/A/H a. HELD: No. b. RULE: The Wills Act specifically requires that a will be in writing and signed by the testator. c. Here the will was not signed by the person whom the will describes as being the creator. Even though the facts show that the husband signed his wifes will by mistake and both the spouse and the husband intended to leave their property to each other, the will cannot be probated because it does not meet the requirements. Such a holding would leave room to fraudulent claims. d. The court will strictly adhere to staturory will formation reaquirement that the testator sign his or her own will unless there is evidence to show that someone else is unjustly enriched by the purported error. e. DISSENT (Musmanno): The Pavlinkos wills were identical. They left each other their property and had the same residual legatee. The attorney knew the R/A/H a.

Chapter 4 Wills: Formalities and Forms

43 of 120

language the Pavlinkos spoke in and versed with them in this language. The intent of the testator must be determined by looking at the four corners of the will signed. Because the couple both left their residuary estate to the same person, the will should be admitted for probate. In addition, the residuary clause can stand on its own so the whole will should not be submitted for probate. 6. In re Snide (p. 250) (not assigned) ii. Curative Doctrines: Substantial Compliance and Harmless Error 1. Substantial Compliance: Even if a will is not executed in strict compliance with the jurisdictions Wills Act formalities, the court is empowered to probate the will IF clear and convincing evidence shows both that (1) the testator intended this document to constitute his/her last will and testament, and (2) the will substantially complies with the statutory Wills Act formalities. (see UPC 2-503) a. In re Will of Ranney (p. 253) b. F/P i. During the execution of Russell G. Ranneys will, the two persons serving as witnesses did not sign the will. Instead, they signed an affidavit swearing they had previously witnessed Ranney sign his will. The lawyer executing the ceremony was not aware that the will itself needed an attestation clause. He believed that the affidavit alone without the witnesses signature on the will was sufficient. The Appellate court ruled that the affidavit literally satisfied the statutory requirements. c. I i. ISSUE 1: If a statue allows for an affidavit to be submitted along with a duly executed will, may the will be probated if the will itself is not signed by the witnesses. ii. ISSUE 2: Whether a will if not formally executed in compliance with a statute may be probated if it substantially complies with the statutory requirements? d. R/A/H i. HELD 1: No. The will may not be probated unless it is signed by attesting witnesses, even though the witnesses may have signed the affidavit. Affidavits and attestation clauses serve different purposes. The legislature expressed an intent that the affidavit be submitted along with a duly executed will. A will without an attestation clause is not duly executed. ii. HELD 2: Yes. A will may be probated if the proponents of the document can prove by clear and convincing evidence that the testator intended the document to be his will. Statutory formalities exist for the purposes they serve. The purpose of a signature is to provide reliable evidence that the will and its terms reflect the testamentary intent of the deceased. iii. The court held that under strict compliance, the will was not properly executed, but the court went on to adopt the substantial compliance doctrine and remanded the case for further consideration. iv. RULE: Courts may still probate a will that substantially complies with the Wills Act formalities v. Courts will not allow procedural requirements to defeat the purposes the statutes are intended to fulfill. The purpose of a signature is to provide reliable evidence of testamentary intent and attestation clause help to prevent fraud and undue influence. Here, an affidavit is reliable. It may be admitted to probate without the witness appearing

Chapter 4 Wills: Formalities and Forms

44 of 120

2.

in court. Because it is sworn testimony, persons are deterred from committing fraud because of the penalties for lying under oath. UPC 2-503 Harmless Error (p. 258) a. Although a document or writing added upon a document was not executed in compliance w/ 2-502, the document/writing is treated as if it had been executed in compliance IF the proponent establishes: i. (1) by clear and convincing evidence that ii. (2) the decedent intended the document or writing to constitute 1. (i) the decedents will, 2. (ii) a partial or complete revocation of the will, 3. (iii) an addition to or alteration of the will, or 4. (iv) a partial or complete revival of his/her formerly revoked will or of a formerly revoked portion of a will. b. Note: The Harmless Error rule is also adopted in R.3d Wills 3.3 c. In re Estate of Hall (p. 259) d. F/P i. Testator and his wife (the Halls) went to their attorneys office, modified a draft of their joint will, and signed it on the advice of their atty that it would constitute a valid will until they signed the final version. The Halls executed the modified draft, the atty signed it (there were no other witnesses), and the Halls went home, where they destroyed their original wills. ii. After the husband died, the wife applied to informally probate the Joint Will. The husbands daughters from a previous marriage objected to the informal probate and requested formal probate of the Original Will. e. I i. Whether a proponent establishes by clear and convincing evidence that a decedent intended a second will to be his will where he revokes all prior wills in his second will and told his wife to destroy his first will. f. R/A/H i. RULE: For a will to be valid, typically 2 people must witness the testator signing the will and then sign the will themselves. However, Montana statute provides that the document may still be treated as if it had been executed, given certain circumstances. One circumstance is that the proponent of the document must establish, by clear and convincing evidence, that the decedent intended the document to be the decedents will. ii. HELD: Yes Although the will was not signed in the presence of 2 witnesses, it is still valid under the harmless error doctrine. iii. A proponent (the wife) establishes by clear and convincing evidence that decedent intended a document to be his will where he revoked his first will in second will and instructed his wife to tear up his first will. Testamentary intent may still exists where there is evidence that the testator did not give the will to anyone because he said it was not finished because the testator may have wanted the will to stand until his attorney provided for a final will iv. Clear and convincing evidence of testamentary intent there is evidence that the testator did not want his first will to be valid and attempted to execute a valid second will. Furthermore, the testator gained the attorneys assurance that the second will would be valid as it was.

2.

Revocation of Wills

Chapter 4 Wills: Formalities and Forms

45 of 120

Revocation in IL (755 ILCS 5/4-7): Will may be revoked only by i. burning/obliterating (by testator, or by someone in his presence and by his direction & consent) ii. By execution of a later will (codicil) declaring revocation iii. By later will to the extent that it is inconsistent with the prior will iv. By execution of an instrument declaring the revocation, and signed/attested according to IL will formalities b. Revocation by Writing or Physical Act i. UPC 2-507 Revocation by Writing or Act (p. 286) 1. Wills are ambulatory documents; i.e. subject to modification or revocation by the testator during her lifetime. 2. All states permit revocation of a will in one of two ways: (1) by a subsequent writing executed with testamentary formalities, or (2) by a physical act, such as destroying, obliterating, or burning the will. a. Note: In all states, oral declaration that a will is revokes, with nothing more, is inoperative. 3. If a properly executed will is not revokes in a manner permitted by statute, the will is admitted to probate. 4. Revocation by Writing: A will, or any part of a will may be revoked by executing a subsequent will that revokes the previous will expressly or by inconsistency. (UPC 2507(a)(1)). The subsequent writing must be executed with Wills Act formalities. a. Express Revocation: There is a clear and express statement of the intent to revoke the prior will. A properly executed instrument that does no more than express the intent to revoke a prior will is a valid will. b. Revocation By Inconsistency: The subsequent writing does not expressly revoke the prior will, but makes a complete disposition of the testators estate, as presumptively replacing the prior will and revoking it by inconsistency. (UPC 2-507(a)(1)) c. Will vs Codicil: If the subsequent will completely revokes the prior will (either expressly or by inconsistency), the subsequent will becomes the testators sole will. BUT If the subsequent will does not make a complete disposition of the testators estate, it is not presumed to revoke the prior will, but is viewed as a codicil. The property not disposed of under the codicil is disposed of according to the prior will. i. Codicil Execution: The codicil IS A WILL it must be executed with Wills Act formalities. However, a codicil is a will that merely amends an existing will, rather than completely replacing it. (UPC 2-507(a)(1)). ii. Exception Codicils to Holographic Wills: Handwritten amendments (interlineations) to a holographic will constitute a valid holographic codicil, even if the interlineations do not qualify as a valid holographic will in their own right. iii. Mixed Wills and Codicils: Holographic codicils to attested wills are valid, and attested codicils to holographic wills are valid. iv. Revocation of codicil/will: Revocation of a codicil does not revoke the underlying will. Revocation of a will revokes all codicils to it. v. 5. Revocation by Act: A will or any part of a will is revoked by performing a destructive act on the will, (1) if the testator performed the act with the intent and for the purpose of revoking the will or part OR (2) if another person performed the act in the testators conscious presence and by the testators direction. a. Traditional Approach: Destructive acts had to actually affect the written portions of the will (e.g. you had to rip, burn, etc. the actual text of the will) b. Modern Approach: Destructive Acts need only destroy the document; not required to affect the text portion (Note: Revocatory or Destructive Acts a.

Chapter 4 Wills: Formalities and Forms

46 of 120

6.

7.

include: burning, tearing, canceling, obliterating, or destroying the will or any part of it (not even requiring the destruction to occur to the actual words of the will)). Writing as Revocation by Act: The act of writing on a will can qualify as revocation by act (e.g. the act of writing VOID across the will qualifies as a destructive (revocatory) act. a. Thompson v. Royall (p. 291) b. F/P i. Lou Bowen Kroll asked Judge Coulling and H.P. Brittain to bring her will and codicil to her home. She told them both in the presence of her attorney to destroy them. Coulling suggested that instead of destroying the will and codicil, Kroll should retain the will and codicil in the event that she decided to execute a new will. Coulling wrote on the back of the manuscript cover to the will the words, This will null and void and to be only held by H.P. Brittain instead of being destroyed as a memorandum for another will if I desire to make the same. This 19 Sept., 1932. Crowell then signed the document. The same was written on the back of the codicil except the name S.M.B. Coulling was substituted for H.P. Brittain and signed by Kroll. ii. The trial court admitted the will and codicil and Krolls heirs at law appeal the decision. c. I i. . d. R/A/H i. HELD: The writing did not qualify as a revocation by writing because the notation did not qualify as a valid will (not executed properlyi.e. not attested because not witnessed and not holographic because the material provisions were not in the testators handwriting). ii. HELD: The writing did not qualify as a revocation by act because the handwriting did not touch any of the written portions of the will as required under the traditional common law approach (see above, p. 46) iii. A will is not revoked where the testator fails to execute another valid will or attempt in some way to physically obstruct the will by causing words or marks to come into contact with the will or destroy the will itself. iv. No. A will or codicil is not revoked where the testator attaches a paper to a will writing this will null and void because it was not executed the way a will is required to be executed. v. No. The will was not revoked because the words this will null and void did not physically come into contact with the words of the original will, even though the testator wrote words on a separate paper attached to the will that declared the will null and void. Furthermore, the words themselves did not constitute a validly executed will. vi. Modern Trend Analysis: The writing in Thompson may qualify as a valid revocation by act because the act arguably affected some portion of the will (if the manuscript cover were construed to be part of the will). The writing may also qualify as revocation by writing its possible to have delayed attestation Partial Revocation by Physical Act: Jurisdictions are split some jurisdictions allow partial revocation by physical act. But, in some staes, a will cannot be revoked in part by an act of revocation; it must be revoked by subsequent instrument.

Chapter 4 Wills: Formalities and Forms

47 of 120

8.

Rationale for requiring partial revocation by writing: 1) Canceling a gift to one person necessarily results in someone else taking the gift, and the new gift can only be made by an attested writing. 2) Permitting partial revocation revocation by physical act offers opportunity for fraud (e.g. the person who takes the new gift may be the one who made the canceling marks). b. Modern Trend/UPC Approach: Many states and the UPC allow partial revocation by physical act (UPC 2-507), but states are split over how to treat the revoked gift. i. MAJORITY: Permit the revoked gift to fall to the residuary and to increase the residuary, but the partial revocation cannot increase a gift outside of the residuary. ii. MINORITY: Hold that the revoked gift may pass via intestacy only. iii. UPC: The will should be given effect as it reads with the partial revocation by act, regardless of where that means the revoked gift goes. c. Partial Revocation in IL: IL does not specify how partial revocation works. Assume Majority approach? Revocation by Presumption: If a will was last in the testators possession and cannot be found following testators death, a rebuttable presumption arises that the testator revoked the will by act. If the presumption is not overcome, the will is deemed revoked. If the presumption Is rebutted, the will is deemed lost, and extrinsic evidence is admitted to prove its terms. If the terms are established, the lost will is probated (jurisdictions are split as to what the burden of proof should be: clear and convincing vs. preponderance of the evidence; almost any evidence is admissible). a. Rationale for Presumption: Testators know that their will is an important document. If the testator takes the will home with him/her, the presumption is that he/she will safeguard it. If the will is not found after the testators death, the more likely explanation is that the testator revoked it by act, rather than lost it. b. The presumption is weak: The presumption that the testator revoked the will is rather weak. If those challenging the will offer a more plausible explanation for why the will is found, the issue becomes one for the trier of fact. c. Duplicate Originals: Duplicate originals are multiple originals of the same will; each one must be properly executed. A photocopy of an executed will is not a duplicate originalthe testator must properly execute each version of a duplicate original. i. Revocation by act or by writing: Affirmative evidence that the testator properly revoked one duplicate original by act or by writing automatically revokes ALL duplicate originals. ii. Revocation by Presumption: The jurisdictions are split over whether the presumption doctrine applies to revoke all duplicate originals if the one the testator took home is not found, but a different duplicate original is found. 1. Revokes all duplicate originals: If the presumption doctrine applies to one duplicate original, it applies to all duplicate originals. The reason for this is that revocation by presumption is a subset of revocation by act. Valid revocation by act revokes all duplicate originals, so valid revocation by presumption of one duplicate original revokes all duplicate originals, even if another duplicate original is found. 2. Does NOT revoke all duplicate originals: The presumption doctrine does not revoke duplicate original wills unless NONE a.

Chapter 4 Wills: Formalities and Forms

48 of 120

d. e.

f.

g.

of the duplicate originals are found following the testators death. The reason is that revocation by presumption is based on the assumption that testators who take their wills home with them take care to safeguard them. But If there is a duplicate original (say at the attys office), the testator is less likely to safeguard the duplicate original he/she takes home. If the will the testator took home is not found, it is just as likely that the testator lost it as that he/she destroyed it with the intent to revoke. Harrison v. Bird (p. 287) F/P i. Daisy Virginia Speer executed a will and named Katherine Crapps Harrison as the main beneficiary of her estate. The original will remained with her attorney and a duplicate was given to Harrison. A few years later, Speer called her lawyer and told him that she wanted to revoke her will. Then the lawyer or his secretary, in each others presence, tore the will, put the torn pieces in an envelope along with a letter informing Speer that her will had been revoked. Harrison attempted to probate her copy of Speers will but the trial court did not admit it to probate. The Circuit Court affirmed the decision I i. Whether a presumption arises that a testator revoked her will if (1) she had possession of the will before death, and (2) the will was not found among her personal effects at her death, even though a duplicate of the will exists. R/A/H i. HELD: Yes, the presumption arises, and the presumption of revocation was NOT rebutted. The evidence provided by Harrison was not sufficient to rebut the presumption that Ms. Speer destroyed her will w/ the intent to revoke it. (Trial Ct Affirmed) ii. RULE: If there is clear and convincing evidence that the testator had possession of the will before death, but the will is not found among the testators personal effects after death, a presumption arises that the testator destroyed the will. iii. RULE: Furthermore, if the testator destroys the copy of the will in her possession, a presumption arises that she has revoked her will and all duplicates, even though a duplicate exists that is not in her possession. This presumption of revocation is rebuttable, and the burden of rebutting the presumption is on the proponent of the will. iv. A presumption arises that a testator revoked her will even if another person has a duplicate copy of the will, if there is evidence she had it before she died and it is not found among her personal effects at death. Though a person may have left copies of his or her will with other people, the law does not require that the testator insure that each copy is destroyed in order to revoke a will. The law only requires evidence that the testator revoked the will itself.

3.

Revival of Wills a. IL Law (755 ILCS 5/4-7(c)): A will that is totally revoked cannot be revived other than by (1) its reexecution or (2) by an instrument declaring the revival that complies with IL will formalities. i. Revival of Partially Revoked Will: If a will is partially revoked by an instrument, which is itself revoked, then the revoked part of the will is revived (it takes effect as if there was never a revocation). ii. See also IL Statutes

Chapter 4 Wills: Formalities and Forms

49 of 120

4.

1. 755 ILCS 5/4-1 to 15 (Wills) 2. 755 ILCS 5/5-1 to 3 (Place of Probate of Will or of Administration) 3. 755 ILCS 5/6-1 to 21 (Probate of Wills and Issuance of Letters of Office) 4. 755 ILCS 5/7-1 to 6 (Probate of Foreign Wills and Estates of Nonresidents) 5. 755 ILCS 10/0.01 to 10 (Uniform International Wills Act) b. Revocation by Operation of Law: Change in Family Circumstances i. Divorce: i.e. the testator, married, executes a will, and then gets a divorce 1. IL Approach: Upon divorce, the testator revokes EVERY legacy interest or power of appointment given to the former spouse; the testator also revokes any fiduciary office of the testators former spouse (755 ILCS 5/4-7(b)) 2. Majority State Law Approach: Divorce revokes any provision in the decedents will for the divorced spouse 3. Minority State Law Approach: Divorce revokes the will only if the divorce is accompanied by a property settlement 4. Note Revocation Statutes Apply to Wills Only: These revocation statutes ordinarily apply only to wills, not to life insurance policies, pensions plans or other nonprobate transfers 5. Modern Trend/UPC Approach: Revocation applies to wills AND other nonprobate transfers 6. UPC 2-804 Revocation of Probate and Nonprobate Transfers by Divorce; No Revocation by Other Changes of Circumstances (p. 306) a. Divorce revokes any revocable will/portion of a will (i) disposition/appointments of property; (ii) powers of appointment (of the spouse or any relatives of the spouse); (iii) nominations of the spouse or any relative of the spouse to serve in any fiduciary/representative capacity; AND b. Severs property interests (the spouse no longer has a joint tenancy or a right of survivorship c. Effect of Revocation: Treat the spouse as though the spouse (and any relatives) disclaimed all provisions revoked by this section of the UPC ii. Marriage (spouse not in premarital will): i.e. the testator (unmarried or married, I suppose) executes a will, and then gets married 1. Majority Approach/UPC: The spouse is entitled to his/her intestate share, unless it appears from the will that (1) the omission was intentional or (2) the spouse is provided for in the will or by a will substitute with the intent that the transfer be in lieu of a testamentary provision (see also UPC 2-301) a. Effect = Revocation: In effect, this kind of statute revokes the will to the extent of the spouses intestate share 2. Minority Approach: The pre-marital will is revoked entirely by marriage. The testator would need to write an entirely new will. iii. Marriage (spouse IN premarital will): Spouse may be able to take an elective/forced share of the decedents property in separate property states (see below, p. 69) c. Birth of a Child: i.e. child born after will executed i. Majority/UPC Approach: Almost all states have pretermitted child statues these give a child born after the execution of a parents will, and not mentioned in the will, a share in the parents estate (see UPC 2-302) 1. Effect of Statutes: Pretermitted child statutes result in a revocation of a parents will to the extent of the childs share. ii. Minority/Common Law Approach: Marriage followed by birth of children revokes a will executed before marriage (this position is disappearing) Components of a Will a. Integration of Wills i. Introduction: Wills are often written on more than one sheet of paper. The threshold issue is: what constitutes the pages of a will?

Chapter 4 Wills: Formalities and Forms

50 of 120

ii. Integration: Doctrine under which all papers present at the time of execution, intended to be part of the will, are integrated into the will. (R.3d Wills 3.5) b. Republication by Codicil i. Introduction: Publication of a will is the testators statement to the witnesses, by words or by action, that a document is the testators will. ii. Republication by Codicil: A will is treated as re-executed (re-published) as of the date of the codicil. 1. RULE: A will is treated as if it were executed when its most recent codicil was executed, whether or not the codicil expressly republishes the will, UNLESS treating as such would be inconsistent with the testators intent. (R.3d Wills 3.4) iii. Potential Conflicts: Republication by Codicil can have unintended consequences 1. Example: Testator revokes Will 1 by Will 2, but then executes a Codicil to Will 1. Then Will 1 is republished, and Will 2 is revoked by implication (squeezed out) 2. Policy: Because of this, the doctrine of Republication by Codicil should be applied only where updating the will carries out the testators intent (i.e. not automatically) iv. Requires Existing Valid Will: Republication applies only to a prior validly executed will. 1. Compare to Incorporation by Reference (below, p. 51): IBR can apply to incorporate into a will language or instruments that have never been validly executed. 2. Exception: A few jurisdictions (e.g. NY do not recognize Incorporation by Reference in those states, some cts will stretch Repub by Codicil to re-execute and re-publish an invalid underlying will, but only if that will went through a valid execution ceremony but is invalid for some other reason v. c. Incorporation by Reference i. Definition: UPC 2-510 Incorporation by Reference: A writing in existence when a will is executed may be incorporated by reference if the language of the will (1) manifests this intent and (2) describes the writing sufficiently to allow it to be identified ii. IL Law (755 ILCS 5/4-9): Unless made by either (1) the testator or (2) someone in the testators presence and by his direction/consent, an (A) addition to a will or (B) an alteration, substitution, interlineation, or deletion of any part of a will, which does not constitute revocation, has no effect. 1. Note: Alterations to wills MUST follow IL will formalities. iii. Clark v. Greenhalge (Incorporation by Reference) (p. 310) iv. F/P 1. Nesmith executed a will in 1977 that named Frederic T. Greenhalge as the executor. The will also made Greenhalge her principal beneficiary to all of their tangible personal property upon her death except the items that she designated by a memorandum left by her and known to Greenhalge, as in accordance with her known wishes to be given to others at her death. Nesmith kept a plastic covered notebook in her drawer and named it List to be given Helen Nesmith 1979. One of the entries among others read, Ginny Clark farm picture hanging over fireplace. Several times Nesmith told her home care nurses that the painting should go to Virginia Clark. Nesmith also told Clark that she wanted her to have the painting. 2. Nesmith executed two codicils to her 1977 will, one on May 30, 1980 and one on October 23, 1980. The codicils amended certain bequests and deleted others while ratifying the will. Nesmith also created a document called MEMORANDUM with Greenhalge in 1972 and identified it as a list of items of personal property prepared with Miss Helen Nesmith upon September 5,1972 for the guidance of myself in the distribution of personal tangible property. 3. Greenhalge was the executor for Nesmiths estate. As executor he distributed Nesmiths property in accordance with the will as amended, the 1972 memorandum as amended in 1976, and certain provisions of the notebook. Greenhalge did not give the painting to Clark because he wanted to keep it. Greenhalge was aware of the notebook and its

Chapter 4 Wills: Formalities and Forms

51 of 120

contents but made no attempt to determine the validity of the gift of her farm scene painting to Clark. Nesmiths notebook was in existence at the time of the execution of the 1980 codicils. The probate judge ruled that the notebook was a part of the will and Greenhalge appealed. v. I 1. May a document be incorporated by reference into a will if the will refers to the document, even though it may not be in the same form as stated in the will, but serves the same function as the document stated in the will, and was in existence at the time the codicils to the will were created?

vi. R/A/H 1.

vii. viii.

ix.

x.

HELD: Yes. A notebook that gives the executor guidance in distributing the testators estate may be incorporated by reference to a will that includes the language of a memorandum that serves as a guide to the executor on the distribution of her estate. 2. RULE: A document may be incorporated into a will by reference if the will makes reference to the document, the document was in existence at the time that the will was created, and is the document is sufficiently identifiable in the will. 3. The cardinal rule in the interpretation of wills is that the intention of the testator shall prevail. 4. It appears from the language used in her will that Nesmith intended to retain the right to alter and amend her will without formally amending the will itself. 5. The language in the will, a memorandum does not preclude the existence of more than one memorandum. It also does not preclude the existence of a document in the form of a notebook from being included in the will. The fact that it was not labeled as such does not mean that it was not intended by the testator to be an instruction as to how to distribute her property at death. Since the testator retained the right to amend and alter her will after execution, the notebook is sufficiently described since it guides the executor in distributing her estate at death and the notebook was in existence at death. 6. A document may be incorporated by reference if the will refers to the document and the document was in existence with codicils were made to the will, even if it did not exist at the time the original will was created. Also, the item is sufficiently described in the will even if it is not specifically referred to if it serves the same purpose as is indicated in the will. Simon v. Grayson (p. 315) F/P 1. Grayson executed a will making reference to a document that would be dated March 25, 1932 and would tell his executors how to give away a gift of $4,000. The letter was found at Graysons death but it was dated July 2, 1933. The letter instructed the executors to give $4,000 to Ester Cohn. Grayson also executed a codicil dated November 25, 1933. I 1. Whether a document may be incorporated by reference to a will if was created after the will but before a later codicil was executed. 2. Whether a document is sufficiently identified by a will if it bears a date differenet from the date that is referenced to in the will R/A/H 1. HELD 1: Yes. A document may be incorporated by reference in the will where it is in existence at the date of a codicil to the will. 2. HELD 2: Yes. A document written in July of 1933 can be validly incorporated into a will and codicil with its latest date at November of 1933 since it informs the testators executors how to pay $4,000 and the document tells the executors to give $4,000 to a beneficiary named Ester Cohn.

Chapter 4 Wills: Formalities and Forms

52 of 120

RULE: A document may be incorporated into a will that was in existence at the time a later codicil was made but not in existence at the time the will was written. A will makes sufficient reference to a document outside of the will to be incorporated by reference even though the will states the date on the document is different to the date referenced in the will as long as the document serves the same function as the one stated in the wil. 4. Because a codicil republishes a will, a document meets the time requirements of the incorporation by reference doctrine if it was in existence at the time the codicil was written. Furthermore, the document referred to does not have to specifically match the description in the will, as long as the will describes the substance of the document and its purpose. xi. UPC 2-513 Separate Writing Identifying Devise of Certain Types of Tangible Personal Property 1. Introduction: UPC Tangible Property List allows a testator to give away his/her tangible personal property via a list not executed with Wills Act formalities, even if the list is created AFTER the will is executed, as long as the will expressly states such intent. 2. Explanation: The doctrine modifies incorporation by reference by waiving the requirement that the document be in existence at the time the will is executed, as long as the document only disposes of the testators tangible personal property. 3. Johnson v. Johnson (p. 317) (not assigned)

3.

Chapter 5 Construction of Wills


1. Mistaken or Ambiguous Language in Wills a. Introduction: Goal of construing wills is to give effect to the testators intent (R.3d Wills 10.1) b. Admissibility of Extrinsic Evidence (Validity vs Construction): i. Evidence Offered to Prove VALIDITY of Will: Generally admissible ii. Evidence Offered to Prove CONSTRUCTION (Interpretation) of Will: Generally inadmissible c. Traditional Approach: No Extrinsic Evidence, No Reformation i. Majority Approach: 2 rules bar admission of evidence to vary the terms of the will 1. Plain Meaning Rule (aka No Extrinsic Evidence Rule): Extrinsic evidence may be admitted to resolve some ambiguities, but the evidence cannot be introduced to argue that a different meaning was intended, apart from the plain meaning of the words of the will 2. No Reformation Rule: Reformation is an equitable remedy that, if applied to a will, would correct a mistaken term in the will to reflect what the testator intended to say. The No Reformation Rule bars this. a. Policy: Not allowing reformation forces the court to interpret the words that the testator actually used, not the words that the testator is purported to have intended to use. ii. Mahoney v. Grainger (Construction of Wills) (p. 336) iii. F/P 1. Sullivan executed a will and instructed her attorney to leave all of her real and personal property to about twenty-five first cousins equally. Her prior will left her assets to two of her first cousins. The will was duly executed. The residuary clause read, All the rest and residue of my estate, both real and personal property I give, devise, and bequeath to my heirs at law living at the time of my decease, absolutely , to be divided among them equally, share and share alike The trial judge ruled that the term heirs at law only applied to her maternal aunt, Frances Hawkes Greene and not her twenty-five first cousins. Certain of the first cousins appealed form the decree disimissing the petition for distribution to them. 2. Issue. Whether extrinsic evidence that a testator intended to dispose property to beneficiaries not named in the will may be admitted when a beneficiary can be ascertained from the face of the will? iv. I

Chapter 5 Construction of Wills

53 of 120

1.

May extrinsic evidence that a testator intended to dispose property to beneficiaries not named in the will be admitted when a beneficiary can be ascertained from the face of the will?

HELD: No. RULE: Extrinsic evidence is not admissible when the beneficiary of a will can be identified on the face of the will. (No Extrinsic Evidence Rule) 3. The words used in the will, heirs at law living at the time of my decease undoubtedly refer to the testators aunt and not her cousins. The testator only heir at law was her aunt. Extrinsic evidence would only be admissible to help to determine the meaning of testamentary language that its not clear in its application to the facts. 4. The Court does not want to redraft a testators will. Therefore, it will not allow extrinsic evidence to change the plain face meaning of a disposition under a will. vi. How to Handle Ambiguities (Traditional Approach) 1. Introduction: 2 kinds of ambiguities 2. Patent Ambiguities: Appears on the face of the will. a. Example: Clause 1 in Ts will leaves the disposable portion of my estate to Ts daughter, A. Clause 2 in Ts will leaves my entire estate to Ts daughters, A and B. b. RULE: (Note this is still the Traditional View for Modern view, see below): Extrinsic Evidence is not admissible to clarify a patent ambiguity. Court must use the 4-corners approach, even if as a result, the will/devise fails, and the property passes by intestacy. 3. Latent Ambiguities: Manifests itself only when the terms of the will are applied to the testators property or designated beneficiaries. a. Type 1 (Equivocation): Occurs when a will clearly describes a person or thing, and two or more persons or things exactly fit that description i. E.g. To my niece, Alicia, if the testator has 2 nieces named Alicia ii. RULE: Extrinsic is admissible (and necessary) b. Type 2: No person or thing exactly fits the description, but two or more persons or things partially fit. i. RULE: Extrinsic evidence is admissible (and necessary) 4. Personal Usage Exception: (A form of latent ambiguity) If the testator has always referred to a person by a name other than the persons true name (e.g. nickname), and the testator uses that name in the will, extrinsic evidence is admissible to show that (1) the testator always called the person by that name and (2) the person called by the nickname is the person who is supposed to take the gift a. The court treats the nickname that the testator gave to the object/person as if it were the proper name d. Modern Trend: Correcting Mistakes Without The Power To Reform Wills i. The Plain Meaning Rule is repudiated: Extrinsic evidence is admissible to establish the circumstances around the testator at the time he/she executed the will; this helps the ct determine testators intent ii. How to Handle Ambiguities (Modern Approach) 1. No Distinction Between Patent and Latent Ambiguities: Extrinsic evidence is allowed to help interpret ambiguities, regardless of form iii. Reformation Is NOT Allowed, but correction is: Courts are allowed to correct mistaken terms to conform the will to the actual intent of the testator. iv. Policy Underlying Modern Trend: Allowing evidence makes room for the doctrines of undue influence, testamentary capacity (including insane delusions), duress, and fraud. v. Arnheiter v. Arnheiter (Reformation of Will/Correction of Mistakes) (p. 343) vi. F/P

v. R/A/H 1. 2.

Chapter 5 Construction of Wills

54 of 120

1.

Guterl executed a will and directed her executrix to, sell my undivided one-half interest of premises known as No. 304 Harrison Avenue, Harrison, New Jersey, and use the proceeds to establish a trust for each of the decedents two nieces. Guterl did not own a home at 304 Harrison Avenue but at 317 Harrison Avenue. Is a testators description of her property in a will sufficient where she names the street, city and state where her house is located but does not identify the correct street number?

vii. I 1.

e.

HELD: Yes. Here the testators disposition is valid because even though the street number was not correct, she owned one-half one piece of property on the street. Because she only devised one-half of the property, which was what she owned, the property is sufficiently identified 2. RULE: Mere erroneous description does not vitiate. The court may use evidence to interpret mistaken terms in the will, to give effect to the testators intent. However the court does not have power to correct or reform a will. 3. The identification of property in a will does not have to be exact, but only needs to clearly fit the actual property that exists. The disposition will be valid if despite a description that does not fit the remaining description clearly identifies the property. Openly Reforming Wills for Mistake i. Introduction: The No Reformation Rule is under attack (in Erickson v. Erickson, not assigned, the court explicitly rejected the rule). ii. UPC 2-805 Reformation to Correct Mistakes (p. 351) 1. The court may reform the terms of a governing instrument, even if unambiguous, to conform the terms to the transferors intention 2. It must be proved by clear and convincing evidence that the transferors intent and the terms of the governing instrument were affected by a mistake of fact or law, whether in expression or inducement. iii. Langbein Curing Execution Errors and Mistaken Terms in Wills (p. 351) 1. R.3d Wills 12.1: Allows courts to reform mistaken terms in a will 2. Policy Reasons: (1) The rise of the nonprobate system; (2) Experience in other jurisdictions; (3) growing embarrassment that failure to cure well-proved mistakes inflicts unjust enrichment; and (4) concern to spare lawyers from needless malpractice liability. iv. Fleming v. Morrison (p. 356) v. F/P 1. Butterfield created a will that disposed all of his personal and real property to Mary Fleming. Before Sidney S. Goodridge signed his will as a witness, Butterfield told him that the will was fake, and that he only created the instrument to undue Fleming to sleep with him. Under statutory law, a will must be attested by three witnesses. Two other witnesses had already attested the will. 2. The Probate Court admitted the will to probate. vi. I 1. May extrinsic evidence be admitted to contradict that the will has been signed by persons signing as witnesses? vii. R/A/H 1. HELD: Yes. And also, the will was invalid. Extrinsic evidence is admissible to prove that one witness was not truly signing as a witness to a will. The testator told a witness that the will was fake and composed for another motive. Because a will in this jurisdiction requires three witnesses and one of them did not truly believe that the testator was signing his will, the will is not valid.

viii. R/A/H 1.

Chapter 5 Construction of Wills

55 of 120

2.

RULE: Extrinsic evidence is admissible to contradict the statements in a will that it is a will, that it had been signed by a person named as the testator, and attested and subscribed by persons signing as witnesses. 3. The Court admitted the evidence because it related to the belief of one of the witnesses, not the validity of the provision to the beneficiary. Therefore the will was not admitted to probate. The issue was not on concerning the testators intent to make a gift to a beneficiary. Death of Beneficiary Before Death of Testator a. Introduction: If a devisee does not survive the testator, the devise lapses (fails). b. IL Law (755 ILCS 5/4-11 Legacy to a Deceased Legatee): (See blue sup p. 13) c. Default rule = All gifts made by will are subject to requirement that the devisee survive the testator, unless the testator specifies otherwise. d. Anti-lapse Statutes: Most states have anti-lapse statutes that, under specified circumstances, substitute another beneficiary for the predeceased devisee (see Antilapse Statutes below, p. 57) e. Common Law Rules Re Lapsed Devises (Default Rules): i. Specific or General Devise: (1) If a specific or general devise lapses, the devise falls into the residue. ii. Residuary Devise: (1) If a residuary devise lapses, heirs of the testator take by intestacy. (2) If only a share of the residue lapses (e.g. when one of two residuary devisees predeceases the testator), at common law, the lapsed residuary share passes by intestacy to the testators heirs rather than to the remaining residuary devisees. 1. Note: This rulethe no residue of a residue ruleis followed in Estate of Russell (see below, p. 56 2. But note: In most states, this rule has been overturned by statute or judicial decision. iii. Class Gift: If the devise is to a class of persons, and one member of the class predeceases the testator, the surviving members of the class divide the gift. iv. Void Devise: Where a devisee is already dead at the time the will is executed, or the devisee is a pet or some other ineligible taker, the devise is void. 1. The same default rules govern the disposition of a void devise as govern a lapsed devise. v. Estate of Russell (p. 359) vi. F/P 1. Thelma Russell executed a valid holographic will that said in pertinent part, I leave everything I own Real & Personal to Chester H. Quinn & Roxy Russell. Russell also disposed of jewelry and money to Georgia Nan Russell Hembree. Russells heirs offered extrinsic evidence to prove that Roxy Russell was a dog. 2. The ct ruled that the fact that Roxy Russell was a dog was a latent ambiguity, and extrinsic evidence was admissible to establish that fact. The trial court held that Russell intended Quinn to receive all of her personal and real property and that the gift to the testators dog merely reflected a wish that Quinn care for the dog. Hembree appeals the decision. vii. I 1. Whether a gift lapses under the anti-lapse statute if the gift is void? 2. Whether extrinsic evidence is admissible because the language of a will could reasonably signify two or more meanings? viii. R/A/H 1. RULE: No-Residue of a Residue (see above) 2. HELD: Dogs are not eligible takers, so the gift to Roxy failed. The gift to Roxy was in the residuary clause; the court applied the no-residue-of-a-residue rule, and held that Roxys half fell to intestacy to the testators heirs. 3. A gift lapses under an anti-lapse statute if the gift is void. Testamentary gifts to animals are void. The gift to the dog lapses under the anti-lapse statute. 4. Note: Extrinsic evidence is not admissible to prove the testators intent because the language of the will is not reasonably susceptible to one or more meaning. Here the

2.

Chapter 5 Construction of Wills

56 of 120

f.

g.

testator left her property to a person and to her dog. The language did not state that the testator was making a gift to a person for the benefit of the dog. The will on its face makes a gift to the dog. The language is not precatory in nature. Extrinsic evidence is not admissible to prove the testators intent. BUT it IS admissible to correct the interpretation of who Roxy was (latent ambiguity) Modern Trend/UPC Approach (Lapsed Devises): As long as any part of the residuary clause is valid, that part catches whatever part of the residuary clause fails i. Policy: The Modern Trend reasons that if the testator included a residuary clause, the testators intent was for ALL of the testators property to pass via the will, and for nothing to pass through intestacy. (UPC 2-604; R.3d Wills 5.5, cmt. O) Antilapse Statutes i. Introduction: Anti-lapse statutes do not prevent lapse; they merely substitute other beneficiaries (usu. Descendants) for the dead beneficiary if certain requirements are met. ii. Typical Antilapse Statute: If a devisee is (1) of a specified relationship to the testator and (2) predeceases the testator, but is survived by descendants who survive the testator, then the descendants are substituted for the predeceased devisee UNLESS the will expresses contrary intent iii. Scope of Anti-Lapse Statutes: 1. Traditional/Majority Approach: Anti-lapse statutes apply to wills only 2. Modern Trend/Minority: Anti-lapse doctrine applies to wills, as well as some will substitutestrusts, insurance policies, and POD contracts, but NOT JOINT TENANCIES iv. Policy/Theory (Presumed Intent): The idea is, for certain predeceasing devisees, the testator would prefer a substitute gift to the devisees descendants rather than for the gift to lapse. v. Contrary Intent: The presumption of testators intent is rebuttable (i.e. the anti-lapse statutes yield to a contrary expression of the testators actual intent). But the will must explicitly provide for the testators intent if it is contrary to the default anti-lapse rules. 1. Words of Survivorship are not (on their own) enough to establish contrary intent: e.g. words of survivorship, e.g. in a devise to an individual if he survives me, or in a devise to my surviving children are NOT a sufficient indication of contrary intent, in the absence of additional evidence. a. Policy: By not specifically providing a substitute gift to the devisees descendants, the testators intent is deemed to be that the gift should pass through anti-lapse. (UPC 2-603; R.3d Wills 5.5, cmt h) vi. UPC 2-605 Antilapse; Deceased Devisee; Class Gifts (p. 365) 1. Devisee must meet the following rules: a. Devisee must be a grandparent or lineal descendant of a grandparent of the testator b. Devisee must either (1) be dead at the time of execution of the will, (2) fail to survive the testator, or (3) be treated as if he predeceased the testator. 2. If the conditions in step 1 are met, then the issue of the deceased devisee take (inherit) in place of the deceased devisee, if they meet the following conditions: a. Issue must survive the testator by 120 hours 3. How much do the issue take (Requisite Degree of kinship)? a. Issue take equally if they are all of the same degree of kinship to the devisee b. If they are of unequal degree to the devisee, then those of more remote degree take by representation. 4. One who would have been a devisee under a class gift if he had survived the testator is treated as a devisee for the purposes of this section, whether his death occurred before or after the execution of the will. vii. Enforcement of Anti-lapse Statutes 1. Common Law: The anti-lapse doctrine applied only to lapsed gifts, not to void gifts

Chapter 5 Construction of Wills

57 of 120

3.

Modern Trend/UPC: Anti-lapse doctrine applies to any qualifying beneficiary who predeceases the testator, regardless of whether the beneficiary dies before or after execution of the will (see UPC 2-605) viii. Ruotolo v. Tietjen (Anti-Lapse Statute) (p. 367) ix. F/P 1. Testator devised half of his property to Hazel Brennan of , if she survives me. Hazel was his stepdaughter, a beneficiary covered by the state anti-lapse statute. She died 17 days before the testator, survived by a daughter. x. I 1. Did the language if she survives me constitute an express contrary intent to the application of the anti-lapse statute? xi. R/A/H 1. HELD: Standing alone, the bare words-of-survivorship language (if she survives me) was NOT enough to establish contrary intent 2. RULE: UPC approach (i.e. anti-lapse statute applies here) see UPC 2-605 above 3. The anti-lapse was adopted to overcome the harsh effects of lapse and therefore should be applied broadly and liberally, placing the burden of proving contrary intent on those who oppose the application (i.e. the drafters of the will). 4. Such language is merely boilerplate in many wills; there was no express gift-over in the event the beneficiary predeceased. xii. Words of Purchase, Words of Limitation, and the Meaning of And vs Or (p. 374) Changes in Property After Execution of a Will a. Introduction: Scenario = A will includes a specific devise of an item of property, but the testator sells or gives the item away before dying. The question is, what, if anything, should the beneficiary take? b. Types of Devises: i. Specific Devise: A disposition of a specific item of the testators property 1. Eg: My 3-carat diamond ring given to me by my Aunt Jane; or Blackacre farm ii. General Devise: Testator intends to confer a general benefit and not give a particular asset 1. Eg: A legacy of $100,000 to A. If there is not $100,000 in cash in the testators estate at death, the legacy is not adeemed (presumed revoked) other property must be sold to satisfy As general legacy iii. Demonstrative Devise: Hybrida general devise, yet payable from a specific source 1. Eg: to B, The sum of $100,000 to be paid from the proceeds of sale of my Apple stock. If the testator owns sufficient Apple stock at death, the executor must sell the stock to get the money to give to B. But if testator doesnt have $100,000 worth of Apple stock, the devise is not adeemed; other property must be sold to raise the full $100,000. iv. Residuary Devise: Conveys the portion of the testators estate not otherwise effectively devised by other parts of the will. 1. Eg: A devise to A of all the rest, residue, and remainder of my property and estate. c. Ademption by Extinction i. Specific devises of real property are subject to the doctrine of ademption by extinction. ii. Ademption by extinction applies ONLY to specific devises, not to general, residuary, or demonstrative devises. iii. Identity Theory: If a specifically devised item is not [found or identified as being] in the testators estate, the gift is extinguished (the item is adeemed). The court will NOT take any extrinsic evidence as to why the item cannot be found. 1. Exceptions: iv. Intent Theory: If the specifically devised item is not in the testators estate, the beneficiary my nonetheless be entitled to the replacement for, or cash value of, the original itemif the beneficiary can show that this is what the testator would have wanted. Otherwise, the item is adeemed. v. UPC 2-606 Nonademption of Specific Devises; Unpaid Proceeds of Sale, Condemnation, or Insurance; Sale by Conservator or Agent (p. 387) 2.

Chapter 5 Construction of Wills

58 of 120

A specific devisee has a right to the specifically devised property in the testators estate at death (etc) (UPC 2-606) see CB p. 387) 2. If specifically devised property is sold or mortgaged (e.g.), then the specific devisee has the right to a general money devise equal to the net sail price, the amount of unpaid loan, condemnation award, or basically, whatever. d. Stock Splits and the Problem of Increase (CB p. 390) e. Satisfaction of General Pecuniary Bequests i. Satisfaction (Ademption by Satisfaction) applies when the testator makes a transfer to a devisee after executing the will (CB p. 390) f. Exoneration of Liens (CB p. 391) g. Abatement i. Intro: If testator gives away more in his/her will than he/she has to give, the doctrine of abatement provides for which gifts are to be reduced first ii. General Approach: Based on presumed testators intent, residuary clause is reduced first, then general gifts, and specific gifts last 1. Criticism: Reducing the residuary gift first arguably is against the testators intent. Often the testator presumes that the residuary clause will be the biggest gift and saves it for the most important beneficiary. If testator is married, usually residue is left to surviving spouse. iii. Minority/UPC: Adopt the general approach but also include a provision giving the courts flexibility to alter the order of abatement where it appears inconsistent w/ the testators overall testamentary wishes. (UPC 3-902) iv. IL Law (755 ILCS 5/24-3): Abatement = Specific legacies pro rata, then general legacies pro rata

1.

Chapter 6 Nonprobate Transfers and Planning for Incapacity


1. Introduction to Will Substitutes a. Langbein The Nonprobate Revolution and the Future of the Law of Succession (p. 394) i. Will substitutes do not pass through the probate system. They are the core of the nonprobate system ii. The Will Substitutes (4 Main Substitutes) 1. Pure Will Substitutes: When pure will substitutes are created, they are functionally indistinguishable from a will. Each reserves to the owner complete lifetime dominion, including the power to name and change beneficiaries until death. a. Life Insurance b. Pension Accounts c. Bank, Brokerage, and Mutual Fund Accounts d. The Revocable Inter Vivos Trust 2. Imperfect Will Substitutes: Also serve to transfer property at death without probate; (but they are functionally distinguishable from wills?) a. Common-law Joint Tenancy b. Joint tenancies in real estate c. Joint tenancies in securities iii. The Hidden Causes of the Nonprobate Revolution 1. Primary Cause = avoidance of probate process iv. Traditional/Common Law Approach to Will Substitute Formalities: Because will substitutes are governed by other areas of law (e.g. contract law, property law, etc), the rules and doctrines of those areas of law control (and not the law of wills) v. Modern Approach to Will Substitute Formalities: Modern trend is to subject the will substitutes to wills-related construction doctrines. 1. Policy: People use will substitutes to avoid the costs, hassles, and delays of probate, not to avoid the wills-related formalities. The wills formalities are better suited than other

Chapter 6 Nonprobate Transfers and Planning for Incapacity 59 of 120

2.

to handle issues that arise as far as creating a document that purports to transfer property at the time of death. Will Substitutes and the Wills Act a. Revocable Trusts/Inter-Vivos Trusts i. Introduction: A trust is nothing more than another way to make a gift. (Conceptually, the key to understanding a trust is to remember that it is a bifurcated gift see Trusts in Chapter 8, starting on p. 69) ii. Parties to a Trust: There are 3 parties to a trust: A Settlor (a.k.a. grantor); a trustee (or numrerous trustees?); and the beneficiaries. 1. Title to the Property is Bifurcated: Legal title is given to the trustee, who holds and administers the property for the benefit of the beneficiaries. Equitable title is given to the beneficiaries. The donor is called the settlor. 2. RULE: The trustee can be one of the beneficiaries, but the same person cannot be the sole trustee and also the sole beneficiary. Policy =If the sole trustee were also the sole beneficiary, then the trustee would owe no duties to anyone but himself. iii. Inter Vivos vs Testamentary Trusts: Inter Vivos Trust = The trust is created during the settlors life. Testamentary Trust = the trust is created by will (see 755 ILCS 5/4-4) 1. Inter Vivos Trusts May be Revocable or Irrevocable: Thats all 2. A Testamentary Trust is Always Irrevocable: Nuff said 3. A Revocable Trust is Always Inter Vivos: This is because, for a trust to be revoked, the grantor must be alive. iv. Creation of Revocable Trusts: 1. Via Deed of Trust: Settlor transfers the property to be held in trust to the trustee. When the settlor dies, the trust property is then distributed or held in further trust, depending on the terms of the instrument. a. Traditional Approach: A revocable trust created by deed of trust requires with Wills Act Formalities, or else it is ineffective. b. Modern Approach: All jurisdictions allow a revocable trust created by deed of trust to create a nonprobate transfer on death. (see R.3d Trusts 25) 2. Via Declaration of Trust: Settlor declares himself to be the trustee of certain property for the benefit of himself during his life, with the remainder to pass to others at his death. While alive, the settlor may revoke the trust and the right to the trust income; and as trustee 3. Farkas v. Williams (Validity of Inter Vivos Trust Created by Declaration) (p. 398) 4. F/P a. Farkas purchased stock on four different occasions, each time taking title in his name as trustee for Richard J. Williams. Concurrently with each purchase, Farkas signed four declarations of trust, where he conveyed himself the life interest, remaninder to Williams, and retained power to revoke by selling the stock. b. Farkas died intestate. His heirs claimed the inter vivos trusts were invalid testamentary dispositions that failed to comply with Wills Act formalities. 5. I a. Should the creation of trust by declaration, that doesnt comply with Wills Act formalities, be held valid? 6. R/A/H a. HELD: Yes the trust is valid b. It appeared as though Farkas did intend to give Williams an interest in the property 1) he set himself up as trustee of the stock for the benefit of Williams.; The stock was registered in his name as trustee for Williams. 2) He manifested an intention to bind himself to having his property pass upon his death to Williams, unless he changed the beneficiary or revoked the trust.

Chapter 6 Nonprobate Transfers and Planning for Incapacity 60 of 120

Some interest passed inter vivos to Williams even though the trusts were revocable, and in the alternative, the process Farkas went through to create the inter vivos trusts adequately served the functions underlying the Wills Act formalities. 7. Theoretical/Policy Perspectives on Inter Vivos Trust Creation (Wills Formalities Required?): a. Classic Trust: Assume the settlor, trustee, and beneficiaries are all different people. Also assume the trust is irrevocable. Then the settlor must transfer title to the trust inter vivos (during iife). The trust then holds legal title to the property, not the settlor. v. UTC 603 Settlors Powers; Powers of Withdrawal (p. 403) 1. (a) While a trust is revocable (and the settlor has the capacity to revoke the trust), rights of the beneficiaries are subject to the control of, and the duties of the trustee are owed exclusively to, the settlor. 2. (b) During the period the power may be exercised, the holder of a power of withdrawal has the rights of a settlor of a revocable trust under this section to the extent of the property subject to the power. 3. Standing a. Linthicum v. Rudi (Beneficiaries Standing To Challenge Revocable Trust Amendments During Settlors Life) (p. 403) b. F/P i. The settlors revocable trust named her brother and sister-in-law as successor trustees on her death or incapacity and also made them the primary beneficiaries on her death. The settlor then amended the trust naming her late husbands nephew as successor trustee and sole beneficiary. Brother and sister-in-law sued to set aside the amendment on grounds of lack of capacity and undue influence. c. I i. Issue: whether revocable trust beneficiaries have the right (standing) to challenge amendments to trust, when made by settlor during settlors lifetime. d. R/A/H i. HELD: Beneficiaries of a revocable trust have no standing to contest amendments to the trust made during the settlors life ii. At best, the beneficiarys interest is contingent and unenforceable during the settlors lifetime. The beneficiaries interest in the trust property vests only at the settlors death. vi. Revocability: 1. Traditional/Majority Approach: Inter vivos trusts are presumed to be irrevocable unless the terms of the trust expressly state that the trust is revocable a. Policy: Gifts are irrevocable, and because trusts are simply another way of making a figt, trusts should be irrevocable unless expressly state otherwise. 2. UTC/Modern Trend: (Reverses the traditional approach) A settlor may revoke or amend an inter vivos trust, unless the trust expressly states that it is irrevocable a. Policy: UTC approach is arguably more consistent w/ a typical settlors intent; but the traditional rule is true to the origin of trust lawthe law of gifts. vii. Revocation by Particular Method Expressed in Trust Instrument: When a trust sets forth an express, particular method of revocation, ONLY that method of revocation is valid 1. Policy: The settlors intent controls. If the settlor sets forth a particular method of revoking a trust, it is presumed that the settlor intends that to be the only way to revoke the trust.

c.

Chapter 6 Nonprobate Transfers and Planning for Incapacity 61 of 120

2.

3.

4.

Subsidiary Law of Wills: The revocable trust is the most will-like of all will substitutes. Courts have applied subsidiary rules from the law of wills, such as abatement or ademption. a. Abatement: Where there is not enough trust property to satisfy the provisions calling for distribution on death of the settlor (abatement). b. Ademption: the trust does not include a specific item of property that is to be distributed to a particular beneficiary (ademption) Traditional Approach: a. In re Estate and Trust of Pilafas (Revocation of Trust by Express Method) (p. 414) b. F/P i. Pilafas created a trust in which eight nonprofit organizations were to receive a portion of the trust property upon his death. The remaining portion was to go to his wife and other stated relatives. He amended the trust twice and simultaneously executed a will at the same time that he amended the second trust. ii. The attorney did not retain the originals of Pilafass will but to the best of his knowledge, gave the originals of the trust agreement, the amendments and the will to Pilafas. iii. Pilafas kept meticulous records but his will and the trust with amendments could not be found among his personal things at death. Pilafas had a room filled with important documents including photographs and old divorce papers. After Pilafas died, his son sought a determination that Pilafas revoked his trust agreement and will. iv. The trial court held that Pilafas had revoked his will and trust. The remainder beneficiaries appealed the decision of the trial court that Pilafas revoked his inter vivos trust and will and died intestate. c. I i. Because the will and trust documents were last known to be in decedents possession, but the documents could not be found at his home, would a presumption of revocation apply to one or both documents? d. R/A/H i. HELD: The inter vivos trust was not revoked, and remained valid. ii. RULE: A will is presumably revoked if it was in the testators possession, but not found among his personal effects at his death. iii. RULE: If a settlor reserves the power to revoke a trust in a particular manner and under certain circumstances, he may only revoke the trust in the stated manner and under the same circumstances as stated in the trust. iv. The trust instrument expressly specified the manner in which decedent could revoke his trust (only through a document in writing given to the trustee). Because the decedent did not issue a document to the trustee, which would revoke/amend the trust, the trust is still valid. UTC Approach/Modern Trend: UTC provides that where the trust sets forth a particular method of revocation, it should NOT be construed as the EXCLUSIVE method UNLESS the trust provision EXPRESSLY makes it exclusive. Substantial compliance with the particular method of revocation is sufficient. (see UTC 602(c)(1) (2)). A will executed after the trust, which specifically refers to the trust or the power to revoke, can revoke a revocable trust IF the trust terms do not specify an exclusive method of revocation AND if the will is not revoked. (UTC 602(c)(1)).

Chapter 6 Nonprobate Transfers and Planning for Incapacity 62 of 120

viii. Revocation No Particular Method Expressed in Trust Instrument: If the trust is revocable, but silent as to the method of revocation, the power may be exercised in any manner that adequately expresses the intent to revoke. The trust can be revoked (1) by writing (even if the writing does not qualify as a will), (2) by act (destructive act + intent), (3) by presumption (arguably), and even (4) orally (UNLESS real property is involved). 1. Divorce: a. Traditional Approach: An inter vivos revocable trust is not revoked by divorce, whilc a will is automatically revoked by operation of law (e.g. divorce). b. Modern Approach: Treat inter vivos revocable trusts like wills, and apply the revocation by operation of law doctrine and automatically revoke the provisions in the trust that favor the ex-spouse. c. Note: The UTC seems to have implicitly adopted the modern trend through its rules of construction. ix. Rights of Creditors of the Settlor: General Rule = one should not be able to shield ones assets from ones creditors. If one has a property interest in a trust, ones creditors should be able to reach that property interest. (Note: The rights of creditors of beneficiaries other than the settlor are different, and are covered in Chapter 8 Trusts). 1. State Street Bank and Trust Co. v. Reiser (p. 416) 2. F/P a. Dunnbeier created a revocable inter vivos trust and reserved the power to amend and revoke, and to direct the disposition of principal and income during his lifetime. He transferred to the trust the stock of 5 closely held corporations. After that, Dunnebier obtained an unsecured loan for $75,000. During the loan application process, Dunnebier represented to the bank that he held controlling interest in the 5 closely held corps. (The court found that he was not fraudulent in this representation). b. Shortly afterward, Dunnebier died. His probate assets wer insufficient to pay his creditors. The bank sued to reach the assets in the revocable inter vivos trust. 3. I a. Whether a creditor may reach the assets of a deceased settlors trust if he created a trust during his lifetime and reserved the right to amend and revoke, or direct payments to himself, even though the trust has living beneficiaries. 4. R/A/H a. HELD: To the extent that Dunnebier had power over the assets in the revocable inter vivos trust during life time, those assets should be available to creditors following his death. i. The court required the creditors to exhaust the decedents probate assets first. b. RULE: Creditors may reach a deceased debtors trust income if he created a trust during his lifetime and reserved the right to amend and revoke, or to direct disposition of principal and income. The creditors may recover an amount that is not satisfied by the estate, and not greater than that which the settlor could have used for his own benefit during his lifetime. c. A creditor may reach the property of a settlros trust if he reserved the rights to amend and revoke, or direct payment to himself during his lifetime. It violates public policy for an individual to have an estate to live on but not an estate to pay his debts with. The creditors may reach the assets of the trust to the extent that the debt is not satisfied by the estate. The creditors may not reach any amount that the settlor could not have used for his personal benefit during his lifetime.

Chapter 6 Nonprobate Transfers and Planning for Incapacity 63 of 120

Where there is evidence that a settlor creates a trust that is not solely intended for a purpose other than the settlors own gain, the court will not allow that trust to be used to avoid paying a debt. 5. Notes on Creditors Rights to Non-Trust Nonprobate Assets: a. Creditors of Joint Tenants: Cannot reach the jointly held property after the joint tenants death because the decedents interest is treated as having vanished. b. Creditors of Life Insurance Policy Holders: Usually cannot reach the proceeds if they are payable to a spouse or child. c. Creditors of Retirement Account Holders: Same as for Life Insurance. b. Payable on Death Contracts and Other Nonprobate Transfers i. Life Insurance Introduction: POD contracts (most commonly life insurance) are contracts by which the insured pays into a policy, and the insurance company agrees to pay the benefits under the policy to a beneficiary designated in the policy if the insured dies while covered. Life Insurance Policies ii. Creation (Wills Act Formalities Required?): POD contracts essentially transfer property from the transferor (decedent) to a beneficiary. So theyre like wills. Life insurance policies need not be created with Wills Act Formalities. 1. Common Law: Under common law, life insurance was the ONLY POD contract that could be created w/o Wills Act formalities. a. In re Estate of Atkinson (Pay-On-Death Contracts Common Law Approach) (p. 407) b. F/P i. Decedent made a deposit in a local bank and took the certificate of deposit as follows: Walter S. Atkinson, P.O.D. Mrs. Patricia Burgeois. He created 2 other accts w/ the same language, but designating two other P.O.D. beneficiaries. ii. After his death, his wife claimed her statutory share of his probate estate and argued that the monies in all three accounts should be included in his probate estate as invalid attempts at non c. I i. Are the P.O.D. clauses on the C.O.Ds valid nonprobate transfers? d. R/A/H i. HELD: The monies in the accounts were invalid attempts at nonprobate transfers. (i.e. this conforms to the common law nonlife insurance POD contracts are invalid transfers). ii. Even though the contracts clearly intended to transfer money to the named beneficiaries, the state did not recognize such POD accounts as valid nonprobate transfers. 2. Modern Trend/UPC Approach: Any & all POD contracts/instruments may be treated as will substitutes a. Estate of Hillowitz (Pay-on-Death Contracts Modern Approach) (p. 409) b. F/P i. Hillowitz was a partner in an investment club. A provisioning in the partnership agreement transferred his partnership interest to his wife in the event of his death. The executors of Hillowitzs estate challenged the agreement claiming that it was an invalid testamentary disposition. The District Court held that the agreement was valid but the Appellate reversed. Hillowitzs widow appealed. c. I i. Whether a partnership agreement that transfers a partners interest to a beneficiary upon his death is valid even though it does not comply with the statute of wills. Chapter 6 Nonprobate Transfers and Planning for Incapacity 64 of 120

d.

3.

4.

R/A/H i. HELD: Yes. Partnerships are third party beneficiary contracts. They are similar to other instruments which are contractual in nature but need not conform to the statute of wills to dispose of gifts at death. Examples of such agreements include (1) a contract to make a will, (2) an inter vivos trust in which the settler reserves a life estate, and (3) an in insurance policy. ii. RULE: A partnership agreement that provides for a payment of the partners interest to a beneficiary upon his death is valid even though it does not comply with the statue of wills. iii. The courts will enforce agreements where the agreement is not solely testamentary. The Court is likely to enforce contractual agreements where the parties undertake a business venture separate from the testamentary disposition, and beneficiaries have reason to expect a gift. Revocability: The transferor who creates the POD clause is presumed to have the right to cancel or change the POD clause (i.e. Beneficiaries of a POD clause do not receive an irrevocable property interest inter vivos). Construction (under the Law of Wills): The modern trend/UPC approach generally applies the wills-related rules to will substitutes, including life insurance policies. The wills-related rules are primarily the rules of construction that arise out of changes that can occur btwn when the instrument was created and when the transferor/testator dies. a. Cook v. Equitable Life Assurance Society (Revocation of POD by operation of law Traditional Approach) (p. 420) b. F/P i. Douglas Cook purchased a life insurance policy and named the appellant (Doris) as the beneficiary. Approximately two years later, Cook and Doris (appellant) divorced, and he married Margaret. ii. The insurance policy expressly provided that the owner of the policy may change the beneficiary by written notice to the company. Douglas never gave written notice to the ins co to remove Doris, but after marrying Margaret, he executed a holographic will that expressly stated that he was giving the ins policy to his wife Margaret and his son. c. I i. May a testator change the beneficiary of his life insurance policy through a will even though it does not comply with the prescribed method in the insurance policy? d. R/A/H i. HELD: No The divorce did not revoke the contractual provisions in favor of Doris (ex-spouse) ct awarded the life insurance proceeds to Doris as the contractual beneficiary designation controlled. ii. RULE: Beneficiaries of a life insurance policy may not be changed by a will if the policy contract provides a specific method for changing beneficiaries. iii. Strict compliance with insurance policy requirements is necessary to change a beneficiary under the policy. The insurer, the insured, and beneficiary should be able to rely on the certainty that policy provisions relating to the naming and changing of beneficiaries will control iv. Courts will protect the expectation interest of a beneficiary under a policy. Because the testator remarried, his first wife would not have

d.

Chapter 6 Nonprobate Transfers and Planning for Incapacity 65 of 120

c.

known that he had changed her as the beneficiary because he changed it in his will and not with the Society e. Common Law: Revocation by operation of law applies to wills only f. UPC/Modern Trend: Revocation by operation of law applies to all will substitues, including life insurance. g. UPC/Modern Trend Survival Requirement: While the UPC applies an express survival requirement to life insurance contracts, it is silent w.r.t. general contracts with P.O.D. clauses. (UPC 2-104, 2-702, 6-101). iii. Langbein The Nonprobate Revolution and the Future of the Law of Succession (p. 410) iv. UPC 6-101 Nonprobate Transfers on Death (p. 411) 1. A provision for nonprobate transfer on death in an insurance policy, contract of employement, etc. is NONTESTAMENTARY 2. Transfer on Death Deeds: New development in lawsome states allow Transfer on Death Deeds; there isnt yet a uniform/model law for it. Key characteristics are: (1) Deed must be executed and recorded inter vivos, but it does not take effect until the grantors death; (2) Deed is revocable during grantors life (usu by recording another deed that revokes the initial deed); (3) transfer is effective immediately upon the grantors death and avoids probate. v. Superwills: If a will is permitted to change the terms of a will substitute, the will is called a superwill. 1. General Rule: Most jurisdictions have rejected the idea of a superwill 2. UPC: The UPC adopts the superwill doctrine only if the contract permits the beneficiary of the policy to be changed by a subsequently executed will (UPC 6-101). The UPC is silent as to what the rule should be if the K does not addres the issue 3. Restatement: See Casebook p. 423. Pension and Retirement Accounts i. Introduction: Pension plans vary greatly, but they typically involve the creation of a property right in a fund of money, to be used by the retiree upon retirement. The plans usually allow the retiree to designate a third party beneficiary who shall receive whatever is left, if the retiree should die before the plan proceeds are exhausted. ii. Langbein The Twentieth-Century Revolution in Family Wealth Transmission (p. 424) iii. Defined Benefit vs Defined Contribution Pension Plans (p. 425) 1. Defined Benefits Plan: Usually, defined benefits plans are funded by employers; there is no individual account; the employee is entitled to receive a fixed benefit (e.g. a percentage of their highest annual salary) for the remainder of his/her life an annuity. a. Annuities: An annuity is a stream of income for the remained of ones lifetime, paid monthly at a fixed amount. Annuities can be purchased separately or are an option as to how one can receive benefits from a pension plan. i. Annuities shift the risk of long life (i.e. long-time payouts) to the company/provider. This is the flip of how it is w/ life insurance. This is the common plan for govt providers (but it is declining in popularity among the private sector). b. Joint and Survivor Annuity: Guarantees fixed payments not only for the life of the employee, but also for the life of the spouse. 2. Defined Contribution Plan: The employer, or employee, or both make contributions to a specific pension account for the employee (e.g. 401(k)). Upon retirement, the individual has rights to withdraw from the account, subject to various distribution rules. Because the employee owns all of the funds in the account, the dcp often leads to lumpsum payouts on the death of the worker and her spouse. 3. Federal Regulation: Private pension plans are heavily regulated by federal statutes most notably the Employee Retirement Income Security Act of 1974 (ERISA). a. General Rule: Federal Law pre-empts state law on time-of-death issues. b. Egelhoff v. Egelhoff (Pension Plans) (p. 426)

Chapter 6 Nonprobate Transfers and Planning for Incapacity 66 of 120

c.

F/P i. The Petitioner was married to David A. Egelhoff. Egelhoff was employed by Boeing who gave him a life insurance policy and pension plan that were both governed by ERISA. The Petitioner was designated as a beneficiary under both plans. Afterwards, they divorced, and the husband Egelhoff died in an automobile accident two months later. Egelhoffs children of a previous marriage challenged her status as beneficiary because a state law revokes all nonprobate testamentary gifts to former spouses.

d.

i. Does ERISA preempt a state statute that revokes the payment of a non-probate asset to a former spouse? e. R/A/H i. HELD: Yes The federal Employee Retirement Income Security act preempts a state statute which revokes the payment of a non probate asset to a former spouse because the Washington statute interferes with the federal statutes s goal to administer a nationally uniform plan. ii. RULE: Wash Statute: Divorce automatically revokes the beneficiary designation in favor of an ex-spouse in all revocable nonprobate arrangements. iii. RULE: ERISA expressly pre-empts all state laws insofar as they may relate to any employee benefit plan. iv. DISSENT (Breyer): The state law imposes a mere administrative burden on the ERISA statute at the expense of other substantive state goals. This Court has held that the fact that state law poses some burden on the administration of ERISA plans does not necessarily require pre-emption. ERISAs ultimate goal is to protect employee benefits and the state law seeks to transfer an employees pension in the manner they wanted to receive them. In this case the Court permits a divorced wife to receive a windfall at the expense of the testators children. The logic of this Court would also extend to state cases involving slayer statues that prohibit a husband who kills a wife from receiving benefits as a result of a wrongful death. v. The ERISA statute commands that a plan shall, specify the basis on which payments are made to and from the plan. If administrators are forced to act in accordance with the state statute, they will have to comply with the varying statutes of all 50 states and wait on litigation before processing a payment. This delay conflicts with the legislatures goal of minimizing the administrative and financial burdens placed on beneficiaries. vi. The state law directly interferes with the purpose of ERISA because it would cause beneficiaries to endure lengthy litigation before receiving their payments and the administrators would have to comply with several states with conflicting laws in order to distribute payments. d. Multiple-Party Bank and Brokerage Accounts i. Introduction: The various types of multiple-party bank and brokerage accounts include (1) Joint and Survivor, (2) POD, (3) Agency or Convenience, and (4) saving (Totten trust) accounts. ii. Problem with Multiple-Party/Brokerage Accounts: Sometimes, depositors intend for some other type of account than Joint & Survivor; but banks used to only allow people to open Joint & Survivor accounts. So the courts have to figure out what kind of account was actually intended

Chapter 6 Nonprobate Transfers and Planning for Incapacity 67 of 120

e.

Definition (Joint & Survivor): With joint & survivor accounts, either A or B has the right to draw on the account, and the survivor solely owns the balance of the account, which is non-probate. 2. POD disguised as Joint & Survivor: A might open a joint acct with B, intending only that B receive the balance upon As death. a. Common Law: Recall that at common law, POD accounts were invalid as nonprobate transfers (UNLESS they were life insurance policies) b. Modern Trend/UPC: POD accts are one of many POD arrangements permitted as valid nonprobate transfers. 3. Agency or Convenience Acct disguised as Joint & Survivor: A might intend that B have power to draw on the account during As life only for the convenience of A (e.g. to help A pay bills), but not for other purposes, and not for B to receive anything at As death. iii. Extrinsic Evidence to Prove Depositors Intent: Because banks and brokerage houses routinely forced depositors interested in created various types of multiple-party accounts to use only one type (the joint tenancy account), even if that wasnt the depositors intent, courts allow extrinsic evidence to prove depositors intent. 1. Burden of Proof: Rule: If the depositor executes paperwork that expressly states that the account is a jt tenancy account, the paperwork creates a presumption that the account is a true jt tenancy account. To overcome the presumption, move jurisdictions require clear and convincing evidence of a different intent. 2. Varela v. Bernachea (Multiple-Party Accounts Depositors Intent) (p. 432) 3. F/P a. Bernachea, a retired atty, fell in love w/ Varela. After a year of traveling together, Varela moved into his condo in Fla, and he paid all of her expenses. Bernachea added Varela to his bank account as a joint tenant w/ right of survivorship. Varela received a check card for the account, which she used freely. Bernachea then suffered a heart attack, and while he was in the hospital, Varela wrote a check that transferred the entire $280k in the acct into an account in her name alone. b. When Bernachea returned from the hospital, he demanded the bank return the money to the original acct. The bank complied and initiated proceedings to settle ownership status. Bernachea alleged that he did not have donative intent, and that Varela had restricted access to the acct because she only had a fcheck card and not paper checks. 4. I 5. R/A/H a. HELD: Varela was entitled to half of the money in the acct. b. When a jt bank acct is established with the funds of one person, a true joint tenancy is presumed and may be rebutted ONLY by clear and convincing evidence to the contrary. c. A check card with no limit on its withdrawal abilities had the same significance as a paper check, so Varelas access to the acct was unrestricted. d. Bernachea had claimed that he did not understand the meaning of a right of survivorship (even though he was an attorney)this fails to provide clear and convincing evidence that Bernachea did not have donative intent. iv. UPC Approach: The UPC provides that, inter vivos, it is presumed that the parties to a multiple party account own in proportion to their contributions, and that upon the death of any party, it is presumed that there is a right of survivorship. 1. The Presumptions control: The presumptions control the distribution of the money in the account unless clear and convincing evidence of a contrary intent exists. 2. (See UPC 6-201 6-227) Joint Tenancies in Realty 1.

Chapter 6 Nonprobate Transfers and Planning for Incapacity 68 of 120

3. 4.

i. Introduction: Joint Tenancy and Tenancy by the Entirety are common/popular methods of avoiding the cost and delay of probate. Upon the death of one joint tenant or tenant by the entirety, the survivor owns the property absolutely, free from the decedents interest in the property. ii. Three Key Features of Joint Tenancy: 1. Equal Interests: The creation of a jt tenancy in land gives the joint tenants equal interests upon creation. a. Unlike jt tenancies in personalty (e.g. bank/brokerage accts), jt tenancies in land require the agmt of all jt tenants to take most important actions. b. A person who transfers land into a jt tenancy cannot, during like, revoke the transfer and cancel the interest given the other jt tenant. (In this sense, a jt tenancy is an imperfect will substitute because it is not revocable). 2. No Devise of Shares By Will: A joint tenant cannot devise her share by will. If a joint tenant wants someone other than the other joint tenant to take her share at death, she must sever the joint tenancy during life, converting it to a tenancy in common. This is because, under the common law vanishing theory of joint tenancy, once the decedent dies, there is no interest for the decedents will to operate on. 3. Creditors Rights: A creditor of a joint tenant generally must seize the joint tenants interest, if at all, during the joint tenants life. In almost all states, the joint tenants interest vanishes at death, leaving nothing for the creditor to reach. Will Substitutes and the Subsidiary Law of Wills a. R.3d 7.2 Application of Will Doctrines to Will Substitutes (p. 413 not assigned, but cover it anyway) See also IL statutes a. 755 ILCS 25-1 to 2 (Lifetime Transfer of Property Act) b. Skim 755 ILCS 27/1 to 100 (Illinois Residential Property Transfer on Death Instrument Act) c. 755 ILCS 30/1 to 2 (Third Party Beneficiary Contract Act) d. 735 ILCS 5/2-1403 (Judgment debtor as beneficiary of trust) e. 205 ILCS 625/1 to 5, 10, 15 (Illinois Trust and Payable on Death Accounts Act) f. 760 ILCS 25/1 (Disclaimer Under Nontestamentary Instrument Act)

Chapter 7 Restrictions on the Power of Disposition: Protection of the Spouse and Children
1. Introduction to Marital Property Systems a. Separate Property (English Common Law): i. Property: Husband and wife own separately all property that each acquires (including his/her earnings). If one spouse is the wage earner, while the other works in the home, the wageearning spouse will own all of the property acquired during marriage (but gifts to the homespouse belong to him/her). ii. Gifts: Gifts to either spouse are theirs separately iii. Death of One Spouse: Upon the death of one spouse, under the elecetive share doctrine, the surviving spouse has a right to claim a share of the deceased spouses property, regardless of the terms of the deceased spouses will. b. Community Property: i. Property: While property acquired before marriage (and gifts acquired during marriage) by either spouse are each spouses separate property, all earnings acquired (and any property acquired w/ such earnings) during marriage by either spouse are community property. Each spouse has an undivided interest in each community property asset. ii. Gifts: Gifts to either spouse are their separately. iii. Death of Spouse: If one spouse dies, the community is dissolved each community property asset is divided in half. The surviving spouses half is his/hers immediately and outright (this ensures that each spouse has a share of the marital property regardless of which spouse

Chapter 7 Restrictions on the Power of Disposition: Protection of the Spouse and Children 69 of 120

2.

acquired it). The deceased spouses half goes into probate where he or she can devise it to whomever she/he wishes. iv. Policy: The spouses are considered partners any property acquired as a result of time, energy, and/or labor of either spouse is considered owned by the partnership. Rights of Surviving Spouse to Support (i.e. Right to Receive Support from Deceased Spouse): a. Social Security: All workers in the USA have to pay into Social Security. SS pays benefits to (1) survivors of predeceased workers, (2) the dependents of Social Security beneficiaries, and (3) (less importantly for the purpose of wills & trusts), people who have already retired or are disabled. Only a surviving spouse can receive the workers survivors benefit (a stream of income for life). The worker spouse cannot transfer the benefit to anyone else. i. Waiver: Surviving spouse cannot waive ii. Divorce: A divorced former spouse of the worker has a right to benefits if the marriage lasted 10 years or longer b. Private Employee Pension Plans: Under ERISA, a surviving spouse must have survivorship rights in the worker spouses retirement benefits (usually an annuity; i.e. a stream of income for lifethat is, support). Unlike social security, under ERISA a surviving spouse can waive his or her rights in the worker spouses private pension plan. i. Waiver: Surviving spouse can waive, but waivers are not favored ERISA has strict rules, e.g. pre-nuptial agreements cannot waive ERISA-covered pension rights. c. Homestead: Homestead Law is designed to secure the family home to the surviving spouse and minor children, free of the claims of the decedents creditors. i. Probate Homestead: Surviving spouse usu has right to occupy the family home (or perhaps family farm) for his lifetime. This right is usu in addition to any other rights the surviving spouse has in the decedents estate. In some states, the homestead must be established by the decedent during life, usu by filing declaration of homestead in some public office; in others, probate ct has power to set aside real property as a homestead. d. Personal Property Set-Aside: The surviving spouse is entitled to claim certain tangible personal property items, regardless of the deceased spouses attempts to devise them (i.e. if set-aside meets limitations & conditions, the decedent usu has no power to deprive the surviving spouse of the exempt items). Items are free from creditors claims (and usually include household furniture and clothing; may include a car and farm animals). Details of the right vary state to state (some states have a statutory list of tangible personal property to which the surviving spouse is entitled; other states have a monetary limit on how much the surviving spouse may claim). i. UPC 2-403: Sets the limit at $15,000, subject to the cost of living adjustment formula in 1-109 e. Family Allowance: Family allowance = statutorily authorized allowance for maintenance and support of the surviving spouse (and often of dependent children). In some states, max allowance is fixed by statute. In others, a reasonable allowance tied to the spouses standard of living is allowed. (See also UPC 2-404 and 2-405) see p. CB 475 f. Dower and Curtesy: At common law, the main method of providing spousal support was either dower or curtesy i. Dower: A widow had dower in all of her husbands qualifying land. Dower entitles the widow to a life estate in 1/3 of her husbands qualifying land. 1. When Dower Attaches: Dower attaches at the LATER of 1) the moment the husband acquires title to land or 2) upon marriage. 2. When Dower Vests (completes): Dower is inchoate (incomplete) during the husbands life, and becomes complete at husbands death. Once dower has attached (even inchoate), the husband cannot unilaterally terminate it by transferring the land. No purchaser (bona fide or not) can cut off the wifes dower without her consent. ii. Curtesy: At common law, the husband had a support interest in his wifes lands, called curtesy. It was the same as dower, but the husband was given a life estate in the entire parcel, not merely 1/3. 1. When Curtesy Attaches: Curtesy only attached if the husband and wife had children

Chapter 7 Restrictions on the Power of Disposition: Protection of the Spouse and Children 70 of 120

3.

iii. Modern Trend: Virtually all jurisdictions have abolished dower and curtesy in favor of the elective share. iv. IL Law IL doesnt have Dower/Curtesty (ILCS 755 5/2-9) v. Note on Transferring Real Property: To avoid any possible claims, both spouses should sign any deed transferring real property, even if the title is held in one spouses name, to ensure that no dower or curtesy interest may be asserted after the transfer. Rights of Surviving Spouse to a Share of Decedents Property a. The Elective Share: All but one of the separate property states give the surviving spouse an elective share of the decedents property (a.k.a. a forced share). It is called elective share because typical statutes provide the surviving spouse w/ an election either A) take under the decedents will or B) renounce the will and take a fractional share of the decedents estate. b. A Typical Elective Share: Typical elective share is 1/3 of all of the decedent spouses probate property, plus certain nonprobate transfers (i.e. less of a share than in the community property approach). c. Policy Issues to Consider: (1) WHAT credit, if any, should the non-wage earning spouse receive for contributing to the partnership and enabling the wage-earning spouse to focus on earning money, and (2) WHEN should that credit be recognized? d. Policy Justifications for elective share: i. #1 (the Partnership Theory): The surviving spouse contributed to the decedent's acquisition of wealth and deserves to have a portion of it 1. Did they, though?? -- what if the higher-earning spouse earns WAAAAYYYY more than the other? ii. #2 (the Support Theory): Provide the surviving spouse w/ adequate support 1. e.g. possible to get a really low amount of support? e. Personal Right: Is the election personal to the surviving spouse? i.e. can someone else (e.g. personal representative) make the election? i. Example Scenario: H dies, leaving a will that excludes W. Before W exercises her right of election, she dies. Should Ws personal rep be allowed to renounce Hs will and take a forced share? If yes, then Ws elective share would pass to Ws heirs/devisees. If no, then all of Hs property will pass to Hs devisees. ii. Majority/UPC Approach: Election right is personal e.g. personal rep cannot act on surviving spouses behalf. (see UPC 2-212(a)) iii. IL Law (755 ILCS 5/2-8) (blue sup p. 10) iv. Incompetent Spouse: If the surviving spouse lacks the capacity to decide whether to exercise the elective share, a guardian of the spouse can decide in the best interests (with the probate courts approval). UPC Approach to Incompetent Spouse: If the elective share is exercised for an incompetent spouse, the share of the elective share that exceeds the share the spouse was taking under the deceased spouses will is placed in a custodial trust, with the surviving spouse having a life estate, and the remainder in the devisees under the will f. Life Estates: It is not uncommon for the first-spouse-to-die to leave the surviving spouse only a life estate in all or part of his/her property. Many reasons for thisone major reason is tax benefits (see p. CB p. 482) i. Estate Taxes: The decedents estate tax depends on the size of his/her estate. 1. Example Scenario: Husband is the wage earner; he dies first with virtually all of the property in his name. 2. Community Property Approach: The husband owned only half of the property, resulting in a lower estate tax. 3. Separate Property Approach: The husband owned nearly all the property, resulting in a higher estate tax. a. Policy: Even if the husband left all of his property to his wife, the wife is worse off under separate property because the after-tax amount passing to her is less ii. Marital Deduction: Congress adopted the estate tax marital deduction to help the surviving spouse (typically the wife) under the two property approaches.

Chapter 7 Restrictions on the Power of Disposition: Protection of the Spouse and Children 71 of 120

g.

Definition: Marital deduction provides that no estate taxes apply to any and all transfers from the deceased spouse to the surviving spouse, even if the transfer is the whole estate. 2. Rule: The decedent must transfer a life estate interest (typically this is done in trust and is called a QTIP trust). 3. Policy: This protects the spousal transfer and also prevents states from earning easy income from such transfers iii. Same-Sex Couples: To date, no court has extended the elective share doctrine to include samesex couples who lived in a spousal-like relationship. But with states increasingly recognizing same-sex marriages, such partners should have the right to claim the elective share. What Property Is Subject to the Elective Share: Elective share statutes gave a surviving spouse a fractional share in the decedents estate. The question is: does estate also include nonprobate property? i. Traditional Approach: The definition of estate in elective share meant probate estate only ii. Modern Doctrinal Considerations: Should nonprobate property be included in the elective share? 1. The Illusory Transfer Test: The most widely adopted judicial response to the nonprobate avoidance problem. Ask whether the inter vivos property arrangement that permits the property to avoid probate is really an inter vivos transfer, or whether the deceased spouse retained such an interest in the arrangement that the transfer is more testamentary than inter vivos. If the transfer is deemed illusory, then the transfer is valid, but the property in question is included in the decedents estate subject to the elective share 2. The Intent to Defraud Test: Focus on the deceaseds state of mind: Did the deceased spouse intend to defraud his/her surviving spouse by creating the nonprobate property arrangement in question? a. Subjective Approach: Did the deceased actually intend to defraud the surviving spouse of his/her right to an elective share in the property by creating the nonprobate transfer? b. Objective Approach: Focus on a variety of factors: (1) The amount of property in question relative to the partys overall property; (2) when the nonprobate arrangement was created relative to the partys death and relative to the partys marriage; (3) how much of an interest the deceased spouse retained; etc. 3. The Present Donative Intent Test: Focus on whether the deceased spouse really had a present donative intent at the moment he/she created the nonprobate transfer. (One could argue this is very similar to the intent to defraud test, just worded differently) 4. Sullivan v. Burkin (Elective Share and Trust Property) (p. 488) 5. F/P a. Sullivan executed a deed of trust where he transferred to real estate to himself as the sole trustee. The net income of the trust was payable to Sullivan during his lifetime and the trustee could pay to him all part of the principal of the estate as he might request by writing. Sullivan retained the right to revoke the trust at any time. b. At his death, the successor trustee was to pay the principal and any undistributed income equally to defendants, George Fr. Cronin, Sr., and Harold J. Cronin if they should survive him. The beneficiaries survived Sullivan. c. Sullivan executed a will in where he stated that he intentionally neglected to make any provision for his wife, appellant, Mary Sullivan, and his grandson, Mark Sullivan. He directed the residue of the trust be paid to the trustee of the inter vivos trust. 1.

Chapter 7 Restrictions on the Power of Disposition: Protection of the Spouse and Children 72 of 120

d. 6. I a.

When Sullivan died, the appellant sought a determination by the court that the trust property should be considered a part of his estate. Whether an intervivos trust with a remainder interest is an invalid testamentary disposition if the settlor retained broad power to modify or revoke the trust, receive income and invade principal during his lifetime?

7.

h.

HELD: The court is a valid inter vivos trust. But assets in an inter vivos trust created during marriage would be subject to the elective share if the deceased retained a power to revoke or general power of appointment. rd 8. Bongaards v. Millen (Elective Share and Trust Created by 3 Party) (p. 492) 9. F/P a. Jean Bongaards was the life tenant of a trust established by her mother. The trust consisted of a 1.4 million apartment building. had the power to terminate the trust at any time during her life and the corpus would have been paid to her. Ten days before her death, Jean appointed the trust remainder to her sister Nina. Bongaards specifically disinherited her husband in her will. Bongaards husband petitioned the court to include the trust property in his wifes estate. 10. I a. Whether a surviving spouse may invade a deceased spouses trust and take their elective share, if the trust was established by a third party? 11. R/A/H a. HELD: No, a surviving spouse may not invade a deceaseds spouses trust and take their elective share if the trust was created by a third party. A third party has no obligation sot support someone elses property. Property owned by a third party has never been a part of someone elses spouses elective share estate. b. RULE: A surviving spouse may include the property of a trust within their deceased spouses estate if the trust was created by third party for the benefit of the deceased spouse c. Most trusts are upheld that make testamentary dispositions at death but do not comply with the Statue of wills on a theory that the beneficiaries have a legitimate expectancy interest. However surviving spouses do not have a legitimate expectancy interests in their deceaseds spouses trust benefits. 12. Revocable Trusts: In most states, a revocable trust created by the decedent spouse is included in determining the surviving spouses elective share (R.3d Wills 9.1) 13. Conflict of Laws: What to do if the decedents elective share property is in a different state? a. UPC: The laws of the state where the decedent was domiciled at the time of death control whether the surviving spouse is entitled to receive an elective share in real property located in another state (UPC 2-202(d)) Statutory Responses to Elective Share (Expanding the Definition of Estate): i. 1969 UPC Amendments - Augmented Estate: The surviving spouse is entitled to the decedent spouses Augmented Estate 1. Augmented Estate: Augmented estate includes (1) decedents probate estate and (2) certain nonprobate and gratuitous inter vivos transfers made during marriage (but NOT life insurance proceeds): a. any transfers where the deceased spouse retained the right to possession or income from the property b. any transfers where the deceased spouse retained the power to revoke or the power to use or appoint (dispose of) the principal for his/her own benefit c. any joint tenancies with anyone other than the surviving spouse

R/A/H a.

Chapter 7 Restrictions on the Power of Disposition: Protection of the Spouse and Children 73 of 120

4.

gifts to 3 parties within 2 years of the deceased spouses death in excess of $3,000 per done per year, AND e. property given to the surviving spouse either inter vivos or via nonprobate transfers (including life estates in trusts). f. Note: UPC approach includes a community property component ii. 1990 UPC Amendments - Marital Property Approach: The augmented estate combines the property of both spouses and gives the surviving spouse a share of the combined, augmented estate that depends on the duration of the marriage 1. Sliding Scale: 1990 UPC Amendment abandons fixed share approach the elective share percentage increases depending on the length of marriage. The surviving spouse starts out entitled to 3% of the couples marital property. The share increases roughly 3% each year until the spouse is entitled to 50% of the couples marital property after 15 years of marriage. (UPC 2-203) 2. Increase in Scope of Marital Property (vs Community Property): Unlike in community property, the 1990 UPC Amendments include both (1) property acquired before marriage and (2) gifts acquired during marriage in property that may be subject to the elective share a. Under community property, property acquired before marriage, and gifts acquired during marriage are that spouses separate property, and the surviving spouse has no rights in that property (unless the spouse commingles it with community property) i. Must the Surviving Spouse Accept a Life Estate? i. They used to be forced to, but now (1993 UPC Amendments) the surviving spouse is not charged against her elective share if she renounces the life estate Migrating Couples and Multistate Property Holdings a. Introduction: Migrating Couples cause problems for the spousal protection doctrine because of potential conflicts of law: (1) real property is governed by the laws of the state where it is located, (2) personal property is characterized at the time it is acquired as either separate or community property based on the laws of the spouses domicile at the time of acquisition, and (3) the at the time of death spousal protection that a surviving spouse is entitled to depends on the spouses domicile at the time of death of the first spouse b. Moving from Separate Property to Community Property State: The spousal protection scheme is community property, BUT the characterization of the couples already-owned property does not change (from separate to community) because the couple moved into a community property state. i. Rule: The spousal protection scheme is community property. BUT the characterization of the couples already-owned property does not change (from separate to community) because the couple moved into a community property state. ii. Example: H + W live in a separate property state. All of the couples marital property was acquired by H, so the property is Hs separate property. They move to a community property state, and H dies. W is entitled to 50% of their community, but they have no community property because they have not acquired property in the new state. iii. Quasi-Community Property: Quasi-community property is separate property that would have been characterized as community property if the couple had been domiciled in a community property jurisdiction if the couple had been domiciled in a community property state when the spouse acquired the property. 1. Rule: When a spouse with quasi-community property dies, the quasi-community property is treated like community property for distribution purposes. The surviving spouse immediately receives a interest in the quasi-community property that is his or hers outright. The deceased spouse can devise only of QCP. iv. Order of Deaths: QCP gives the non-wage-earning spouse property rights in the property acquired during the marriage by the other spouse only if the wage-earning spouse dies first.

d.

rd

Chapter 7 Restrictions on the Power of Disposition: Protection of the Spouse and Children 74 of 120

5.

QCP is not the same as community property QCP applies only to the property owned at death by the deceased spouse, not by the surviving spouse (i.e. QCP actually applies to separate property). 2. If non-acquiring spouse dies first: he/she has no right to devise any of the surviving (QCP-owning) spouses property (even if the surviving spouse has property that would have been characterized as QCP if he/she had died first). c. Moving from Community Property to Separate Property State: i. RULE: The spousal protection scheme is the elective share. The non-wage earning spouse receives his/her half of the community property outright, and the deceased spouses half goes into probate. Surviving spouse can then claim an additional 1/3 or interest in the deceased spouses probate property ii. Example: Before moving, H + W live in a community property state. All of H + Ws marital property is acquired by and titled in the wage-earning spouses name. But the property is treated as comm prop (each spouse owns share). Couple moves to sep prop state; H dies. W gets by community property and Hs half goes into probate. W is also entitled to an additional 1/3 to through elective share (i.e. W can double-dip in spousal protection). Rights of Spouse Omitted from Premarital Will: a. Introduction: Issue arises when a testator writes a will, then gets married, and then dies without ever adding the spouse to the will b. Omitted spouse presumption: Where the testator (1) marries after executing his or her will and (2) dies w/o revising or revoking the will, a presumption arises that the testator ACCIDENTALLY disinherited the spouse (i.e. testator meant to amend the will to provide for the spouse, but died before doing so). i. Rebuttable Presumption: The presumption is rebuttable to rebut, the challenger must show either (1) the failure to provide for the new spouse was intentional, and that intent appears in the will; (2) the testator provided for the spouse outside the will and the intent that the transfer outside the will be in lieu of the spouse taking under the will is established by any evidence, including oral statements y the testator and/or the amount of the transfer; or (3) the spouse validly waived the right to share in the testators estate ii. In re Estate of Prestie (Omitted Spouse Presumption) (p. 516) iii. F/P 1. Maria and W.R. Prestie were married in Las Vegas in 1987, but they were divorced two years later. Over the years, they maintained a good relationship, and in 2000, when W.R. became ill, Maria moved into his home to take care of him. 2. In 2001, W.R. amended an inter vivos trust he had established in 1994 to provide for a life estate for Maria in his condominium. 3. Shortly thereafter, the couple was married. W.R. passed away nine months later. W.R.'s son, the trustee and beneficiary of the trust, said that W.R.'s amendment to the inter vivos trust rebutted the presumption of revocation of W.R.'s will as to Maria. 4. The ptf (the son and primary beneficiary under the decedents estate plan) asked the court to expand the scope of the doctrine judicially to take into consideration the decedents revocable trust. The decedent amended his revocable trust just a few weeks before he married to grant his new spouse a life estate in his real property. 5. Son argued that the new spouse should not qualify as an omitted spouse because the deceased spouse provided for her in his revocable trust iv. I 1. Whether an amendment to an inter vivos trust could rebut the presumption that a will is revoked as to an unintentionally omitted spouse. v. R/A/H 1. HELD: An amendment to an inter vivos trust does NOT serve to rebut the presumption that a will is revoked as to an unintentionally omitted spouse." 2. "[T]he only evidence admissible to rebut the presumption of revocation for the purposes of NRS 133.110 is a marriage contract, a provision providing for the spouse in the will, or a provision in the will expressing an intent to not provide for the spouse." 1.

Chapter 7 Restrictions on the Power of Disposition: Protection of the Spouse and Children 75 of 120

6.

[citation omitted]. Since there was not a marriage contract, and there was nothing in the will providing for the spouse or expressing an intent not to do so, the will was revoked as to Maria. c. Common Law/Minority: A premarital will was revoked on (1) marriage outright or (2) marriage following the birth of a child. d. Modern Trend/Majority: The premarital will is valid/left intact. However, state statutes give a surviving spouse a spousal intestate share. e. UPC: UPC 2-301 (see casebook p. 518) UPC grants an omitted spouse the right to receive no less than his/her intestate share of the deceased spouses estate from that portion of the testators estate (if any) that is not devised to a child of the testator or the childs descendants (directly or through anti-lapse) if (1) the child is not a child of the surviving spouse, and (2) the child was born before the testator married the surviving spouse. i. Effect of UPC: If the testator devises all of his probate estate to his child or descendants from a prior relationship or marriage, the surviving spouse will not receive an omitted spouses share, despite otherwise meeting the requirements of an omitted spouse. f. Inter Vivos Trusts Treated as Wills for Omitted Spouse Statutes: In some states, the omitted spouse doctrine arises only if the marriage occurs after execution of all the deceased spouses wills AND revocable trusts, and the surviving spouses share is of the property included in the probate estate and revocable trusts. i. E.g. in Estate of Prestie, the court refused to consider the gift in the revocable trust in applying the rebuttable presumption doctrine, because the state statute made express reference only to the decedents will. g. Malpractice Liability (see Emanuels p. 201) Rights of Descendants Omitted from the Will a. IL Approach: By default, if a child is born after the execution of the will, the child is entitled to receive the portion of the estate he would have gotten had the testator died intestate (and all legacies shall abate proportionately for that purpose) i. Exception: If provision is made in the will for a child born after the will is executed, then that provision controls ii. Exception: If, from the will, it appears that the testator intended to DISINHERIT the child, the child gets nothing. IL rolls like that, son. b. Protection from Intentional Omission i. Majority Approach: In all states except Louisiana, a child or other descendant has no statutory protection against intentional disinheritance by a parent c. Protection from Unintentional Omission i. Introduction: The omitted child doctrine applies where a testator executes a will; some time later, has a child; and dies without revising or revoking the will. The issue is whether the testator meant to omit the child or not. Pretermission statutes are designed to prevent the unintentional disinheritance of descendants. ii. If the Will Provides for Children Born After Execution: If the testator writes a clause indicating that children who are not born at the time of execution (but are anticipated/contemplated to be born afterwards), generally those children do not qualify as omitted children (they may be receiving a class gift) iii. Children Alive At Will Execution: Some states extend the omitted child statute to include both (1) children born after the will was executed and (2) children born before the will was executed, but who werent named in the will 1. Affirmative Disinheritance Required: If the omitted child statute covers living children, most courts require affirmative disinheritance specific reference to the child. Blanket statements, that the testator has no children or that no children are to take under the will, usually arent enough to disinherit the child. (see Anna Nicole Smith note on CB p. 534)

Chapter 7 Restrictions on the Power of Disposition: Protection of the Spouse and Children 76 of 120

iv. Omitted Child Presumption: Where the testator has a child after executing his/her will, and then dies w/o revising or revoking the will, a presumption arises that the testator meant to amend his/her will to provide for the new child, but died before getting around to it. 1. Rebuttable Presumption: The presumption that the testator accidentally disinherited his/her child is rebuttable. a. To Rebut the Presumption: Traditional RulePresumption can be rebutted ONLY by showing that either: (1) the failure to provide for the new child was intentional and that intent appears from the will; OR (2) the testator provided for the child outside of the will and the intent that the transfer outside of the will be in lieu of the child taking under the will is established by any evidence, including the amount of the transfer; OR (3) the testator had one or more children when the will was executed and devised substantially all of his/her estate to the other parent of the omitted child b. Two types of omitted (or pretermitted) child statutes: Missouri type and Massachusetts type i. Missouri Type Statute: Under the Missouri type of omitted child statute, the intent to omit the child must be determinable SOLELY from the terms of the will. Extrinsic is not admissible. ii. Massachusetts Type Statute: Extrinsic evidence IS admissible to help determine whether the omission of the child was intentional. v. Omitted Childs Share: If the presumption of unintentional failure to provide for the new child is not rebutted, the typical omitted child statute gives the omitted child his/her share of the testators probate estate. 1. Gray v. Gray (Omitted Childs Share) p. 528 2. F/P a. John Gray executed a will while married to Mary, and although he had two children from a previous marriage, he left all of his estate to Mary. 3 years later, John and Mary had a son, Jack. 5 yrs later, John and Mary divorced. John died w/o changing his will b. Under Alabamas revocation by operation of law doctrine, Mary was treated as though she predeceased John (due to the divorce), so she did not take. c. Jack petitioned the probate court to determine if he was entitled to a share of Johns estate under Alabamas omitted child doctrine. 3. I 4. R/A/H a. HELD: b. RULE: Usually, if a child is born after the execution of a will, and the will fails to provde for the child, the omitted child is entitled to a share of the testators estate. i. An exception exists an omitted child is NOT entitled to a share of the estate if when the well was executed, the testator had one or more children and devised substantially all his estate to the other parent of the omitted child c. Although the presumption arose that the omission was accidental, it could be overcome by the exception that the testator had one or more children when the will was executed, and he left substantially all of his estate to the other parent of the omitted child (i.e. John meant to omit Jack from the will). d. It did not matter that Johns other children were from a different marriage; the exception still applies (Jack gets nothing). e. DISSENT: The exception was meant to apply where the child could look to inherit down the road from the other parent who took from the decedent. Here, where the other parent was ineligible to take (due to divorce), the exception should not apply. Chapter 7 Restrictions on the Power of Disposition: Protection of the Spouse and Children 77 of 120

vi. Overlooked Child vs Omitted Child: Some states have expanded the traditional omitted child doctrine to include a living child who is omitted because (1) the testator does not know about the child, or (2) the testator mistakenly believed the child was dead 1. General Rule: The overlooked child receives his/her intestate share, just like an omitted (pretermitted) child vii. UPC Approach: UPC 2-302 (see CB p. 531): If a testator fails to provide for any of his/her children born or adopted after the execution of the will, the omitted after-born or after-adopted child receives a share in the estate (NOTE: I am not typing out the whole rule. See the book) 1. UPC Applies to both Adopted and After-Born Children: The UPC expressly provides that it applies to children born or adopted after execution of the will (UPC 2-301(a)) 2. Intent to Omit: UPC dictates that evidence that the failure to provide for the child was intentional must come FROM THE WILL (2-203(b)(1)) NOTE: Unlike the UPC omitted spouse doctrine, the UPC omitted child doctrine does not broaden the scope of the evidence that can be used to prove intent to omit a new child. (Compare omitted spouse doctrine above, p. 75) 3. Other Children: UPC 4. The Omitted Childs Share (How Much Do They GET??): The omitted childs share depends on whether the testator has other children living at the time he/she executes the will. a. Testator Had No Children When Executing Will: The omitted child receives his/her intestate share, UNLESS (1) the testator devised all or substantially all of his/her estate to the other parent of the omitted child AND (2) the other parent survives the testator and is entitled to take (in which case the omitted child gets nothing). (UPC 2-302(a)(1)) b. Testator Had One or More Children Living When Executing Will: If the will devised property to one or more of the then-living children, then the omitted childs share (1) is taken out of the portion of the testators estate being devised to the then-living children, and (2) should equal the share or interest the other children are receiving, had the testator included all omitted children with the children receiving shares and given each an equal share. (Gifts to the then-living children are to abate pro-rata (UPC 2-302(a)(2)) c. Overlooked Child: The UPC considers ONLY a child overlooked because the testator thought him/her dead when executing the will. UPC treats those children exactly the same as adopted or after-born children. However, The UPC DOES NOT consider children overlooked simply because the testator did not know (e.g. illegitimate children) (UPC 2-302(c)) 5. Omitted Issue of Deceased Child: Most omitted child statutes cover omitted children only. But some explicitly provide that they apply ALSO to the omitted issue of a child who predeceased the testator. The omitted issue of the child who died before the testator (e.g. the omitted grandchildren of the testator) take their intestate share (does this mean that the issue evenly split the predeceased childs share? Per stirpes? Etc?) viii. Nonprobate Transfers and Pretermitted Child Statutes ix. Introduction: With the increasing popularity of nonprobate modes of transfer, esp the use of revocable inter vivos trusts as will substitutes, a new issue has arisen: Do the pretermitted heir statutes protect a child omitted from a non-probate mode of transfer such as a revocable trust? x. Modern Trend (Minority Approach?): A few states have modified the omitted child doctrine to recognize that increasingly, the inter vivos revocable trust is being used much like a will. In those states, the omitted child doctrine arises only if (1) the birth occurs after execution of all deceased spouses wills and revocable trusts AND (2) the omitted childs share is of the property included in the combined probate estate and revocable trusts. xi. Majority Approach: The pretermitted-heir statutes apply only to wills. xii. Kidwell v. Rhew (Pretermitted Child Statutes and Revocable Inter Vivos Trusts) (p. 536) xiii. F/P Chapter 7 Restrictions on the Power of Disposition: Protection of the Spouse and Children 78 of 120

1.

2.

Irene Winchester created a revocable, inter vivos trust. She transferred real property to the trust, and named her daughter, Margie Rhew, as successor trustee upon Irenes death. Irene never executed a will. Another daughter, Rhenda Kidwell, petitioned the ct to apply Arkansas pretermitted-heir statute to the revocable trust. Kidwell argued that the pretermitted-heir statute should apply to dispositions made by testamentary will substitutes, such as an inter vivos trust. Should the pretermitted-heir statute apply to dispositions made by testamentary will substitutes, such as an inter vivos trust? HELD: No (Majority View). RULE: Pretermitted-heir statutes apply only to wills, not trusts. Kidwell cited no convincing authority that would compel the court to conclude that the pretermitted-heir statutes apply to trusts. A will and a trust are different things legally. A will is a disposition of property to take effect on the death of the testator. A trust is a fiduciary relationship in which one person holds title to a property subject to an equitable obligation to keep/use the property for the benefit of another. The terms are not interchangeable. Thus, the statutes, which speak only in terms of execution of a WILL do not apply to trusts.

xiv. I 1. xv. R/A/H 1. 2. 3. 4.

7.

See Also IL Statutes a. 755 ILCS 5/15-1 to 4 (Spouse and Child Awards)

Chapter 8 Trusts: Introduction and Creation


1. Introduction: A trust is a fiduciary agreement by which a settlor transfers legal/title ownership of trust property (the trust res or trust corpus) to trustees, who hold/manage/use the property for the benefit of the beneficiaries, who have equitable interests in the property. Trust Purpose: Trusts can be used for many purposes (trusts are the most flexible legal instruments available). The main point of trusts is to separate the benefits of ownership (for the beneficiaries) from the burden of ownership (for the trustees). Trust Structure: Prototypical trust structure is that A transfers property to B for the benefit of C (and possibly others). That is a trust. A is the settlor; B is the trustee; and C is the beneficiary a. Example (Gift vs. Trust): Gift A gives B $1,000. This is a classic inter vivos gift. B can do whatever she wants with the $1,000. Trust A gives B $1,000, to use for the benefit of C during Cs lifetime; then on Cs death, any remaining principal is given to D. This is a classic trust. b. 5 Common Trusts: i. Revocable Trusts: O declares herself trustee of property for the benefit of O for life, and then on Os death, to pay the principal to Os descendants. O retains the power to revoke the trust. 1. S Basic Trust Rules: These are the main rules: a. The same party can wear all 3 hats: i.e. One person can be settlor, trustee, and beneficiary, as long as there is another trustee or beneficiary. i. Merger: If the same party is both trustee and beneficiary, and there is no other trustee or beneficiary, the legal title and equitable title are said to merge, and the trust is terminated (i.e. bifurcation of legal and equitable titles is essential to a trust). b. A trust is not created until it is funded: A trust is not funded until property is transferred to the rd trust/trustee. Where the trustee is a 3 party (someone other than the settlor), property MUST be transferred to the trustee, with the intent that the trustee hold and manage it for the benefit of someone else. i. Note: Executing a deed of trust does not constitute funding. c. A trust will not fail for lacking a trustee: Where the settlor clearly expresses the intent to create a trust of provides for funding, a trust will not fail for want of a trustee.

2.

3.

4.

Chapter 8 Trusts: Introduction and Creation

79 of 120

5.

i. Trustee Unable/Declines to Serve, or Settlor Forgets to Appoint Trustee: If a trustee declines to serve (a named trustee is not required to serve but rather must accept the appointment), dies, or is unable to continue, or if the settlor forgot to name a trustee, the court will appoint a successor trustee. ii. If the court appoints a trustee: If the court appoints a trustee, and the trustee accepts the position, due to the fiduciary duties inherent in the office, the trustee can leave the position only with court approval or the consent of all beneficiaries. iii. If the will creates the trust, but fails to name the trustee: The general rule is to appoint the executor as trustee. iv. Exceptions: If the court concludes that the powers given to the trustee were personal, to be exercised only by that particular trustee (e.g. the settlor intended the trust to last only as long as the named trustee actually served as trustee), the court will DECLINE to appoint a successor trustee, and the trust will fail. BUT this exception is narrowly construed, and rarely applied. d. Co-Trustees must agree on action: If a private trust appoints co-trustees, the general rule is that all the trustees must consent to any proposed action. An individual trustee or subgroup of trustees cannot act alone. BUT the trust instrument, however, can provide that the trust can act upon a majority vote of the trustees. i. UTC Approach: The UTC rejects the common law rule and permits action based on the vote of a majority of the co-trustees (no requirement to specify majority vote in the trust instrument). The Parties to a Trust a. The settlor The person who creates the trust. i. Inter Vivos Trust: Settlor creates the trust during his/her life ii. Testamentary Trust: Trust is created by will 1. Note: If a trust is created by will, then of course, the settlor cant be the trustee. iii. Trust Creation: Trusts may be created by 1. Declaration of Trust: Settlor declares that he holds certain property in trust this occurs if the settlor is also the trustee 2. Deed of Trust: Settlor transfers property to another person as trustee a. RULE: If the settlor is not the trustee, then deed of trust or actual delivery of the trust property to the trustee is REQUIRED b. The trustee The person/people who own legal title to the trust property. i. A trust may have one or more trustees ii. TRUSTEE RULES: 1. For the trust to be valid, the trustee must owe equitable duties to someone other than himself/herself (i.e. the same person cannot be the sole trustee and also the sole beneficiary) 2. A trust will not fail for want of a trustee: a. If the settlor intends to create a trust, but fails to name a trustee, the court will appoint one (R.3d 31, 34) b. If a will names someone as trustee, but the named person refuses the appointment or dies while serving as trustee, and the will does not provide for a successor trustee, the court will appoint a successor trustee. 3. The trustee MUST have active duties to perform. If the trustee has no duties at all, the trust is said to be passive or dry, the trust fails, and the beneficiaries get legal title to the trust property (i.e. the beneficiaries take what the trustee had). iii. Bifurcated Ownership 1. The trustee owns legal title to the trust property (a.k.a. trust corpus). The beneficiaries own equitable/beneficial interests. iv. Legal Entity Status of Trustee: The trust is not a freestanding legal entity with the power to sue, be sued, and transact in its own name. Instead, the trustee sues, is sued, and transacts in his capacity as such. v. Rights of Creditors of Trustees

Chapter 8 Trusts: Introduction and Creation

80 of 120

c.

Creditors of the trustee as an individual: Although the trustee has legal title to the trust property, a personal creditor of the trustee has no recourse against the trust property (R.2d Trusts 308). 2. Creditors of the trustee as a trustee: Creditors of the trustee as trustee (i.e. a creditor who interacts with the trustee in regard to trust property) has recourse against the trust property, but not the trustees personal property (UTC 1010) vi. Duty of Trustee 1. Trustee is held to a fiduciary standard of conduct. This includes: 2. Duty of loyalty: Trustee must administer the trust solely in the interest of the beneficiaries (self-dealing is sharply limited and often prohibited altogether) 3. Duty of Prudence: Trustee is held to an objective standard of care in managing the trust property 4. Subsidiary Duties: a. Duty of Impartiality Between Classes of Beneficiary: Treat all classes of beneficiaries the same (e.g. high-income beneficiaries, remainderpersons, etc.) b. Duty Not To Commingle: Dont commingle trust property with trustees own property c. Duty to Inform and Account: To the beneficiaries 5. Penalties for Failing to Meet Duties: A trustee who breaches her fiduciary duties may be denied compensation, subjected to personal liability, and removed as a trustee 6. Rejecting Trustee Appointment: a. Common Law: Once a person accepts appointment as trustee, the person cannot be released from office unless all of the beneficiaries consent, or by court order. b. UTC Approach: Allows for trustees to resign with 30 days notice to all interested parties (UTC 705) vii. Individual vs Corporate Trustees 1. Trustee may be an individual or a corporation 2. Settlors often name co-trustees (e.g. individual and corporate) 3. Delegation: Trustees may delegate to experts on matters outside the trustees competence a. Modern Approach: Delegation is allowed 4. Individual: Usually costs less than corporate trustee, and usually has a strong sense of settlors wishes. 5. Corporate: e.g. banks with a trust dept experienced in portfolio mgmt. and trust administration. Usually higher fees (cost of expertise, deep pockets, and institutional safeguards) 6. Trustee Compensation: Note: The rules governing trustee compensation entitle the trustee to reasonable compensation. a. Right to Contract for Trustee Compensation: The default rules of compensation may be displaced by contrary agreement; corporate trustees usually insist on such agreements b. Traditional Rule of Trustee Compensation: Award the trustee an annual commission set by a statutory formula (usually a % of the trust corpus, or percentage of trust income, or some combo of the two). c. Modern Rule of Trustee Compensation: UTC 708 and R.3d Trusts 38 entitles the trustee to reasonable compensation 7. The beneficiaries the people who own beneficiary title to the trust property i. Rights of Beneficiaries: 1. If the trustees breach duties, beneficiaries have a claim against trustees personally 1.

Chapter 8 Trusts: Introduction and Creation

81 of 120

6.

7.

8.

9.

Wrongful Disposure: If the trustee wrongfully disposes of the trust property, the beneficiary can recover the property (unless the property has come into the hands of a bona fide purchaser for value). b. If the trustee disposes of trust property and acquires other property with the proceeds of sale, the beneficiaries can enforce the trust on the newly acquired property Remedial Trusts: Remedial Trusts arise by operation of law and thus are not subject to the requirements to create a valid trust. They are used as equitable remedies by courts to order one party who is currently holding property to transfer the property to another party whom the court concludes has a stronger equitable claim to the property. a. Resulting Trust: A resulting trust arises when a trust fails on whole or in part. Courts use it to require the party holding the property (usu. The trustee) to return the property to the settlor (or the settlors estate if the settlor is dead). i.e. the Resulting Trust is an equitable reversionary interest. i. Scenario 1: Where an express trust fails or makes an incomplete disposition (R.3d Trusts 8) ii. Scenario 2: Where one person pays the purchase price for property and causes title to the property to be taken in the name of another person who is not a natural object of the bounty of the purchaser (R.3d Trusts 9) b. Constructive Trust: Constructive trusts are used to prevent unjust enrichment. i. Traditional Approach: The courts required (1) a confidential or fiduciary relationship between the transferor and the transferee, (2) a promise (express or implied) by the transferee, (3) that the transferor transferred property to the transferee in reliance on the promise, and (4) that the transferee refuses to honor the promise (which constitutes unjust enrichment) ii. Modern Trend: Courts tend not to emphasize the elements themselves, but they do focus on the equitable notion that the constructive trust is used to prevent unjust enrichment. Trust vs Legal Life Estate a. Legal Life Estate: Life tenant possesses and controls property (i.e. life tenant has legal ownership of property i. Formation: Property to A for life, remainder to As descendants ii. Problems Arise: 1. Cannot Sell Fee Simple: Legal life tenant cannot sell a fee simple unless the life estate instrument explicity grants such power to the life tenant. Otherwise, life tenant must get permission from ALL remainderpersons and reversioners, or get judicial approval. 2. Obtaining A Mortgage or Leasing Property: Potential lenders/lessees will be wary of working with land owned by a life tenant who will eventually die (and then possession of the land will go to someone else. 3. Waste: e.g. cutting timber, taking down a still-usable building on the property may give rise to waste remainderpersons might have rights to injunctions or damages. 4. Risk of Creditors: If the life tenant gets into debt, the creditor can seize the life estate and sell it. b. Trust (aka Equitable Life Estate): Trustee, not the life beneficiary, has legal title to the trust property i. Formation: Property to X in trust for A for life, remainder to As descendants. ii. Trusts Cure/Mitigate the Problems with Life Estates: (see CB p. 555) 1. Risk of Creditors: Trusts can be put out of reach of creditors (unlike legal life estates) Trust vs Debt: A trust is not the same as a debtthe requirement of an identifiable trust res distinguishes a trust from a debt a. Trust: A trustee holds a fiduciary duty to hold a specified property for the benefit of another. The trust property must be kept separately from the trustees own funds b. Debt: Involves a personal obligation to pay a sum of money (not necessarily a specifically identified property) to another. c. Key Analysis: Whether the recipient of the assets is entitled to use them as his own and commingle them with his own assets. (R.3d Trusts 5, cmt k) Creation of A Trust

a.

Chapter 8 Trusts: Introduction and Creation

82 of 120

a.

b.

Overview: 4 core requirements to create trust: (1) settlors intent to create trust; (2) funding property transferred to the trust/trustee; (3) the beneficiaries must be ascertainable; and (4) writing (i.e. the trust act does not require writing, but the law of wills and/or the statute of frauds (if the trust involves real property) requires writings) Intent to Create a Trust: The grantor must manifest an intention to create a trust relationship (UTC 402(a)(2), R.3d Trusts 13). No specific words are requiredonly intent. i. Lux v. Lux (Intent to Create Trust) (p. 557) ii. F/P 1. Philomena Lux died testate devising the residue of her estate to her grandchildren. In the residuary clause of the will, the testator stated that her estate shall be maintained for the benefit of said grandchildren and shall not be sold until the youngest of said grandchildren has reached twenty-one years of age. 2. The Court determined whether the language in the will created a testamentary trust. iii. I 1. Did the language in the will devise the property to testators grandchildren outright, or in trust? iv. R/A/H 1. HELD: The will created a trust. 2. RULE: A trust is created when legal title to property is held by one person for the benefit of another. No particular words are required to create a testamentary trust (the absence of words such as trust or trustee is immaterial where the requisite intent of the testator can be found). 3. RULE: General rule (will executor as trustee): Unless either (1) a contrary intention appears in the will or (2) the appointment of executor as trustee is deemed improper or undesirable, the executor of the will would be named trustee. 4. The language in the will did not give the grandchildren a fee simple title to the realty. The testator, realizing the nature of the bequest and the age of the beneficiaries, intended that someone would hold/manage the property until they were of sufficient age to do so themselves. v. Jiminez v. Lee (Intent to Create Trust/Language) (p. 558) vi. F/P 1. The defendants mother purchased a $1,000 face value U.S. Savings Bond which was registered in the names of defendant, and/or plaintiff, and/or Dorothy Lee, plaintiffs mother. One of the defendants clients made one $500 gift to the defendant and two other $500 gifts to the defendants two other children. The donor deposited $1,500 into a savings account in the name of the defendant and his three children. The defendant cashed the savings bond and invested the proceeds in common stock of the Commercial Bank of Salem, Oregon. The shares were registered as Jason Lee, Custodian under the Laws of Oregon for Betsy Lee. The defendant closed the joint savings account containing the clients gifts to defendants children and $1,000 of the proceeds invested in Commercial Bank stock. The defendant also took title to this stock as custodian for his children. The trial court held that the defendant did not hold the savings bond or the savings account in trust for the benef 2. it of the plaintiff. At trial, the defendant testified that his mother said that she was going to supply a bond to help with his daughters educational needs and that she was naming him and his wife to use the funds as may be conducive to the educational needs of his daughter. The defendant also wrote a letter to his mother in which he stated that his children, Dave and the plaintiff were aware that the defendant held his mothers property in trust for them. vii. I 1. Whether a trust exists where the specific term in trust is not used to transfer the property? viii. R/A/H

Chapter 8 Trusts: Introduction and Creation

83 of 120

HELD: Yes. A trust exists here even though the specific term in trust was not used to transfer the property. 2. RULE: No specific words are required to create a trust, as long as the requisite intent is expressed in the trust instrument. 3. It is sufficient that the defendant testified that he received property for the benefit of his own children and he admitted in a letter that he wrote to the owner that he held property in trust for his children. Even though the defendant purchased bank stock as the custodian for the plaintiff under the Uniform Gift to Minors Act, it was ineffectual to increase the defendants powers over the trust property from that of a trustee to a custodian. 4. The particular facts revealed during the trial showed that the defendant was aware that he was holding property in trust for the plaintiff. ix. Fuzzy Distinctions between Gift and Trust: The intent to create a trust is merely a variation on the intent to create a gift; sometimes it is hard to tell the difference between the two. Two particularly difficult scenarios are: 1. Precatory Trust: i.e. language used such as wish or hope. Precatory language DOES NOT CREATE A LEGAL OBLIGATION that the property be used for the benefit of another. A precatory trust is not a trust at all, but merely a gift with a wish. a. Key Analysis: Whether the language indicates the intent to vest the beneficial rd interest in the 3 party (in which case the transfer is a trust), or whether the language expresses merely the hope or wish that the recipient of the property rd uses it for the 3 partys benefit (in which case, it is a gift to the recipient, who is not actually a trustee). b. Failed Gifts: Note this in in Emanuels, but it wasnt assigned reading, so fuuuuuuuuck it. 2. Unthank v. Rippstein (Transfer of Property to Trust) (p. 569) 3. F/P a. Craft handwrote a letter to Iva Rippstein promising to her that he would make monthly payments in the amount of $200.00 to the appellee for the next five years, if he lived that long. At the end of the letter, he wrote in his own handwriting that he bound his estate to the payments and struck out the words stating provided I live that long. b. The appellee filed an action against the estate of the Craft demanding payments; she argued that the letter constituted a declaration of trust. c. The trial court ruled in favor of the appellee. The Court of Civil Appeals reversed and held that the letter established a voluntary trust. 4. I a. Whether a donor creates a trust by promising to make monthly payments in the future for an indefinite amount of time 5. R/A/H a. HELD: No. A donor does not create a trust by promising to make monthly payments in the future. b. RULE: To prove a trust, the facts must show that the donor intended to act as a trustee or have the property managed by a trustee, not just a sincere desire to make a gift. c. Craft wrote that he bound his estate to a promise to make monthly payments and excluded the clause stating provided I live that long. However this language does not suggest that the donor intended to be a trustee over the payments. His intent was to bind his estate to such payments but not to create a trust. i. He also did not expressly declare that all or any specific portion of his property/assets would constitute the trust corpus.

1.

Chapter 8 Trusts: Introduction and Creation

84 of 120

c.

The Necessity of Funding (Transferring Trust Property): A trust cannot exist without trust property, often called the res or the corpus. The res may be one dollar or one cent or any interest in property that can be transferred (R.3d Trusts 40, cmt b) i. Traditional Approach: The act of funding the trust was viewed as completely separate from creating the trust. Funding was a function of the law of propertythe asset had to be delivered to the trust/trustee, and the Statute of Frauds governed the requirement of written evidence of the transfer/funding. 1. Transfers of Real Property: Required there to be a grant deed, separate from the declaration of trust or deed of trust, which transferred the real property from the settlor to the trust/trustee. 2. Transfers of Personal Property: Required either (1) the item itself to be manually delivered to the trust/trustee or (2) a written instrument manually delivered to the trust/trustee that listed the personal property in question and evidenced the symbolic/constructive delivery of the item(s) to the trust. ii. Modern Trend Approach: The requirements of Creating and Funding the trust can overlap and can be one and the same. 1. Written DECLARATION of Trust and Real Property: i.e. Settlor is the trustee; written declaration of trust IDs the real property the trust holds a. Majority Rule: The declaration of trust also transfers the real property interest into the trust, without a separate writing (UTC 401). b. Minority Rule: Separate writing required to transfer title to the real property from the settlor as nontrustee to the settlor as trustee (this seems pointless) 2. Written DECLARATION of Trust and Personal Property: i.e. Settlor is the trustee; written declaration of trust specifically IDs the personal property the trust holds a. Majority Rule: Writing will be sufficient to simultaneously create the trust, transfer the personal property interests into the trust, and transfer the equitable interest to the beneficiary (UTC 401) b. Minority Rule: If the trust property is stock or other titled property, the settlor is required to change the registration of the trust property from the settlor as nuntrustee to the settlor as trustee to transfer property to the trust. 3. Oral DECLARATION of Trust and Personal Property: i.e. Settlor is the trustee; oral declaration (by settlor) specifically identifies the personal property the trust holds a. Majority Rule: The oral declaration will be sufficient to create the trust and to transfer the personal property interests into the trust. b. Overlap of Funding with Intent: Because the elements of trust creation and funding overlap, evidence of creation and of intent may overlap as well. (See Emanuels p. 222??) c. Note: There is no oral declaration that can specify real property because the Statute of Frauds requires written evidence of the transfer. rd 4. Written DEED of trust and Real Property: i.e. Trustee is a 3 party (not the settlor); written deed of trust IDs the real property the trust holds a. Jurisdictions are split: on whether a reference to the property in a list attached to the deed of trust is sufficient to constitute delivery in all cases or whether the settlor must do more if the property is real property (e.g. execute a separate grant deed transferring title from the settlor to the trust/trustee). rd 5. Written DEED of trust and Personal Property: i.e. the trustee is a 3 party (not the settlor); written deed of trust specifically includes a list that identifies the personal property the trust holds a. Jurisdiction are split: on whether a reference to the property in a list attached to the deed of trust is sufficient to constitute delivery in all cases or whether the settlor must do more if the property is stock (e.g. re-register the stock) or other titled property. rd 6. Oral DEED of trust and Personal Property: i.e. trustee is a 3 party; deed of trust is oral

Chapter 8 Trusts: Introduction and Creation

85 of 120

Majority Rule: Even if the settlor specifically identifies the personal property the trust holds, oral declaration is insufficient to create the trusts and to transfer personal property interests into the trust. There must be some other evidence of delivery, i.e. either actual delivery of the personal property in question to the trust/trustee or deliver of a written list of personal property assets being transferred to the trust/trustee. 7. UTC Approach: The Uniform Trust Code expressly PERMITS oral trusts, UNLESS otherwise prohibited by state law (e.g. Statute of Frauds). The required proof of the terms is clear and convincing evidence. (UTC 407) iii. Grantor TrustsTaxation (see CB p. 576) 1. Grantor Trust: Trusts in which the income is taxable to the settlor (grantor) because the grantor has retained substantial control and is deemed by the Internal Revenue Code to still be the owner of the trust assets. 2. In Brainard v. Commissioner (not assigned), the taxpayer was trying to avoide taxes by shifting the income generated by his stock trading from himself, as settlor, to the income beneficiaries, who were in lower tax brackets. Such tax avoidance is no longer available. a. RULE: Where the settlor/grantor retains sufficient dominion and control over the trust, under what are known as the Clifford Regulations, the trust is known as a grantor trust and the settlor is deemed owner of the income generated by the trust for tax purposes. d. The Necessity of Trust Beneficiaries i. Introduction/RULE: A Trust MUST have one or more ascertainable beneficiaries (UTC 402(a)(3)), R.3d Trusts 44). There must be someone to whom the trustee owes fiduciaries; someone who can call the trustee to account. A private trust must be for the benefit of the beneficiaries. 1. Charitable Trust Exception: A charitable trust (unlike a private trust need not have an ascertainable beneficiary to be valid but notewe skipped charitable trusts on the syllabus, so. Fuuuuuuckit!! 2. Unborn Children Exception: The beneficiaries of a private trust may be unborn or unascertained when the trust is created. a. Eg: A trust created by O, who is childless, for the benefit of her future children would be valid. ii. Beneficiaries MUST be ascertainable (except unborn children): Ascertainable means that you must be able to identify the beneficiaries by name. If their names are not expressly set forth in the trust, the trust must contain a formula or description of the beneficiaries that permits the court to determine by objective means who they are. 1. Policy: The requirement of ascertainable beneficiaries ensures that it is possible to determine who has standing to come into court and hold the trustee accountable. 2. Clark v. Campbell (Ascertainable Trust Beneficiaries) (p. 579) 3. F/P a. A settlor left certain property in trust to my friends, as they, my trustees, shall select. He also wrote, Each of my trustees is competent by reason of familiarity with the property, my wishes and friendships, to wisely distribute some portion at least of said property. The court determines whether the trust must and does sufficiently identify the beneficiaries of the trust. 4. I a. Whether the bequest for the benefit of the testators friends must fail for the want of certainty of the beneficiaries? b. Does the clause provide for definite and ascertainable beneficiaries so that the bequest can be sustained as a private trust? 5. R/A/H a. HELD 1: Trust fails though the instrument indicated a clear intent to create a trust, the trust failed for want of ascertainable beneficiaries.

a.

Chapter 8 Trusts: Introduction and Creation

86 of 120

6.

7.

8.

HELD 2: Nothe document does not provide an objectively ascertainable formula for identify who the beneficiaries should be. c. RULE: There must be a beneficiary or a class of beneficiaries indicated in the will capable of coming into court and claiming the benefit of the bequest. This principle applies to private trusts, but not to public trusts or charities. d. Even though the settlor described the class of beneficiaries as his friends and stated the trustee would know who they were, there were no guidelines to eliminate separate who might be beneficiaries from those that the settlor intended except what the trustee believed. Therefore the trustee was given wide discretion and the beneficiaries were not identifiable. Identifying Beneficiaries Using Familial Terms: Courts routinely hold that familial terms such as children, issue, nephews, and nieces are objectively ascertainable. Some courts have even held that the terms relatives and relations refer to ones heirs under the states descent and distribution scheme and thus are objectively determinable Power of Appointment: Where there is a transfer in trust for members of an indefinite class of persons (such as to testators friends), no enforceable trust is created, but the transferee (trustee) has a discretionary power to convey the property to such members of the class as he may select. a. Key Differences between Trust and Power of Appointment: (1) with power of appointment, the holder owes no fiduciary duty to the possible appointees (unlike trusts); (2) exercise of the power is purely discretionary; (3) the possible appointees of a power (analogous to trustees) do not have to be ascertainable; they just must reasonably meet the description set forth in the power. i. See 755 ILCS 5/4-2 (blue sup p. 12) b. Transforming a Failed Trust into a Power: Prof. Scott (top trust law scholar) argued that anytime a trust in favor of an unascertainable class failed, the trust should automatically be transformed into a power of appointment. i. Restatement Adopts Scotts View: See R.3d Trusts 46 but note, only a handful of courts have adopted the rule. c. Application to Clark v Campbell: In Clark, the court ruled that use of the terms trustees and in trust evidenced the intent to create a trust, not a power. If the testator had not used those terms, the more likely construction would have been that the testator intended to create a power. Honorary Trusts: In an honorary trust, the transferee is not under a legal obligation to carry out the settlors stated purpose, hence the qualifier, honorary. But if the transferee declines, she is said to hold the property upon a resulting trust, and the property revers to the settlor or the settlors successors. a. Rule: If a trust would otherwise fail for want of ascertainable beneficiaries, but the purpose is specific and honorable (and not capricious or illegal), the trust may continue as long as the trustee is willing to honor the terms of the honorary trust. If the trustee stops honoring the terms of the honorary trust, he/she is not permitted to keep the property. In that case, a resulting trust is imposed, and the property is ordered distributed to the proper takers. b. Trusts for the benefits of a pet, eg: Trusts for the benefits of a pet or to maintain ones gravesites, while honorable, technically fail for want of ascertainable beneficiaries. c. In re Searights Estate (Honorary Trusts) (p. 582) d. F/P i. George Searights (testators) will left his dog, Trixie, to Florence Hand (trustee), and he directed his executor to deposit $1,000 in the bank to be used to pay Florence 75 cents a day to care for the dog for its life. The

b.

Chapter 8 Trusts: Introduction and Creation

87 of 120

e.

e.

i. Is the testamentary bequest for the care of Trixie the dog valid in Ohio as (a) a proper subject of a so-called honorary trust, and (b) not being in violation of the rule against perpetuities? f. R/A/H i. HELD: Yes, the bequest is a valid honorary trust. ii. HELD: No, the trust does not violate the rule against perpetuities. iii. Because the beneficiary is a dog, there is no beneficiary capable of enforcing the trust. However, because the trust was for a specific, honorable purpose (to take care of the specific dog, Trixie), and not a capricious or illegal purpose, the trust was a valid honorary trust, and could continue as long as Florence was willing to honor the terms of the trust (i.e. to use the money to care for Trixie). iv. The trust does not violate the Rule against Perpetuities because the trust principal ($1,000) would be exhausted within 3-5 years. g. The Rule Against Perpetuities: If the administration of the trust can continue for longer than the maximum period allowed under the Rule of Perpetuities, the trust is invalid from the moment of its attempted creation. i. Common Law Rule: An honorary trust for a non-charitable purpose is void if it can last beyond all relevant lives in being at the creation of the trust + 21 years. Even if the trust is for a pet animal, the pets lifetime is NOT considered relevant (e.g. even if a pet bird lives for 80 years, the birds lifetime is not part of relevant lifetimes). ii. Honorable Trusts vs Charitable Trusts: Charitable trusts are NOT subject to the rule against perpetuities. But most Honorary Trusts ARE subject to the rule against perpetuities, because though they are honorable, they are usually not charitable. i.e. Charitable trusts have to be for the good of society, but most Honorary trusts specify a specific animal or a specific gravesite, not all dogs or something like that. iii. Statutory Purpose Trusts (Majority Approach): Most states have statutes that permit a trust for a pet animal or other non-charitable purpose for a given amount of time, and for the perpetual care of a grave site. These statutes are based on the Uniform Trust Code (UTC 408-409) or Uniform Probate Code (UPC 2-907). The Necessity of a Written Instrument: i. Inter Vivos Trusts: 1. Common Law: Under the common law approach, the requirement for a written trust instrument is governed by the Statute of Frauds and the Wills Act formalities not trust law. a. No Orally Created Trusts Involving Real Property: Under the Statute of Frauds, the terms of an oral trust involving real property cannot be enforced against the transferee because oral conditions are not permitted to vary the terms of a deed. 2. Modern/Majority approach: An inter vivos oral declaration of trust is enforceable. R.3d of Trusts 20. An oral trust of personal property need not be evidenced by a trust instrument, but the creation of an oral trust and its terms may be established only by clear and convincing evidence. (UTC 407) a. Real Property: Restatement imposes a constructive trust and orders the purported trustee to distribute the real property to the intended beneficiaries outright (often the settlor is the intended beneficiary, so the constructive trust ends up looking like a resulting trust).

Chapter 8 Trusts: Introduction and Creation

88 of 120

Remedial Trusts: The Statute of Frauds does not apply to the remedial trusts (constructive or resulting trusts) because they are not true trusts in the full sense of the word, but rather are judicial remedies that arise by operation of law. Under the remedial trusts, the original trust is not upheld and enforced; the purported trustee is simply ordered to transfer the real property in question immediately to the appropriate party or parties. 4. Unclean Hands: If the settlor created the oral trust for real property for improper reasons (the settlor was trying to hide assets from creditors or from an impending divorce, etc.), even under the Modern Trend, the courts generally will not help one who has unclean hands. The purported trustee will be permitted to keep the real property free of any trust. ii. Testamentary Trusts: Testamentary trusts (usually created in a testators will) must be in writing pursuant to the Wills Act formalities. Where a testamentary trust fails for want of a writing, the issue is whether the relief should be a constructive trust or a resulting trust. The answer turns on whether the failed testamentary trust is deemed a secret trust or a semi-secret trust. 1. Testamentary Trusts vs Inter Vivos Trusts: a. RULE: A trust where the trustee is to deliver personal property at the settlors death, if funded inter vivos, is a an inter vivos trust and does not have to be in writing. 2. In re Estate of Fournier (Oral Creation of Trust for Personal Property) (p. 589) 3. F/P a. Fournier gave $400,000 in cash to a couple he was friends with and asked them to hold it until he died and then to deliver it to his sister Fogarty. He had two other sisters, Flanigan and Rose, but he explained to the couple that Fogarty needed the money and the others did not. b. He died testate, with a will leaving the residue of his estate equally to Fogarty, Flanigan, and his nephew, Curtis King. Fogarty met with the couple and received the money. Fogarty petitioned the court for declaratory judgment to establish that during his lifetime, Fournier had created an oral trust for her benefit. c. The trial court reasoned that because Fournier had told Flanigans daughter about the money, he must have intended it to pass through probate, and that Fogarty was to take the money as personal representative of Fournier. 4. I a. Had the decedent created an inter vivos oral trust for the benefit of Fogarty? 5. R/A/H a. HELD: Fournier had created an oral trust for the benefit of Fogarty, in which the couple was supposed to hold money for Fogarty until Fourniers death. b. RULE: Although a trust need not be in writing, the creation of an oral trust must be established by clear and convincing evidence. c. The court ruled there was clear and convincing evidence he intended to create an inter vivos trust for Fogarty. Primarily, the testimony of the couple showed that Fournier intended for Fogarty to take the money. d. EPILOGUE: After the initial ruling, Flanigan found a handwritten note in Fourniers hosue, written by Fournier and dated after he gave the money to the couple, that referenced the $400,000 and provided that the money was to reimburse Flanigan, Fogarty, and King but there was a line through Kings name. The court ruled that the note tracked the decedents residuary clause, and the note sufficiently evidenced that the inter vivos trust was for all three. 6. Secret Trust: A secret trust is a testamentary trust that fails because, on the face of the will, the secret trust looks like an outright gift to a deviseeit does not indicate intent to create a transfer to the devisee as a trustee, for the benefit of someone else. 3.

Chapter 8 Trusts: Introduction and Creation

89 of 120

7.

8.

Evidence: Courts admit extrinsic evidence that the devisee was to take property as a trustee, not as a devisee b. Majority/Traditional Remedy: The courts remedy unjust enrichment by imposing a constructive trust for the benefit of the intended beneficiary (see also R.3d Trusts 18, R.3d Restitution 46) Semi-Secret Trust: A semi-secret trust is a testamentary trust that fails because, although it identifies a trustee (i.e. it expressly states that the devisee was not intended to take the property for his/her own benefit), it does not identify a beneficiary. a. Evidence: Courts DO NOT admit extrinsic evidence to identify the intended beneficiaries b. Majority/Traditional Approach: Courts impose a resulting trust and give the property back to the settlor/testator. Typically, the property falls into the residuary clause of the will. But if the failed testamentary trust was the residuary clause, then the property falls to intestacy. c. Minority/Modern Approach: Constructive trust should be imposed in favor of the intended beneficiary (see also R.3d Trusts 18, R.3d Restitution 46) d. Oliffe v. Wells (Secret/Semi-Secret Trusts) (p. 593) e. F/P i. Ellen Donovan created a will leaving the residuary estate to the defendant and wrote, to distribute the same in such manner as in his discretion shall appear best calculated to carry out wishes which I have expressed to him or may express to him. Wells (defendant) was also named as executor. Donovans heirs brought suit, claiming that the residue should be distributed to them. ii. The defendant stated in his answer to the lawsuit that Donovan had orally expressed to him before and after the execution of the will that her estate be used for charitable purposes. He also stated that he desired and intended to distribute the residue of the estate for those purposes. f. I i. Did the will in question create a valid trust, or did the trust fail (leaving a secret or semi-secret trust)? g. R/A/H i. HELD: The trust is too indefinite to enforce; the will indicates that the devisee take property as trustee, but the beneficiaries cannot be identified. ii. RULE: A trust that is not sufficiently declared on its face to be a trust cannot be used to defeat the rights of heirs at law by extrinsic evidence of a trust iii. On the face of the will, the residuary bequest to the defendant gives him no beneficial interest (i.e. the heirs have the beneficial interest the will does not exclude them). As for Wells duty as a trustee, the will declares a trust too indefinite to be carried out, and the testatrixs next of kin must take by way of resulting trust. Oral Inter Vivos Trusts of Land: Where S conveys land to T upon an oral trust to pay the income to B for life, and upon Bs death to convey the land to R, the Statute of Frauds in virtually every state prevents enforcement of the express trust. a. Remedy for Transfers Wrongly Obtained: If the transfer was wrongly obtained by (1) fraud or duress; (2) where the transferee was in a confidential relationship with the transferor; (3) where the transfer was made in anticipation of the transferors death, then the courts will impose a constructive trust for the beneficiaries. b. See also Hieble and Pappas, p 596 (casebook) a.

Chapter 8 Trusts: Introduction and Creation

90 of 120

10. See Also IL Statutes a. 760 ILCS 5/1 to 21

Chapter 9 Rights to Distributions from the Trust Fund


1. Rights of the Beneficiary to Distributions a. Mandatory Trust: Trustee must make specified distributions to an identified beneficiary i. Overview: Mandatory trusts and mandatory interests cause relatively few problems because the trustee either performs pursuant to the mandatory terms or the trustee does not. b. Discretionary Trust: Trustee has discretion over distributions. The beneficiary has no right to receive payments of income and/or principalany such payments are at the discretion of the trustee (typically according to a standard set forth in the express terms of the trust). The trustees discretion can also be limited by coupling it with an express standard (e.g. a discretionary support trust). i. Settlors Responsibilities: Settlor can postpone and delegate to the trustee the decisions of to whom to make distributions, in what amounts, and when. 1. Settlors Purpose: Settlor may provide or purpose or standard that the trustee must keep in mind when exercising his/her discretion. a. RULE: The standard MUST be set forth in the express terms of the trust. b. Implications for Trustee: The trustees performance is measured against the standard set forth in the will. ii. Trustees Duties: 1. Fiduciary Obligations: (See Chapter 10, p. 105) 2. Duty to Decide: The trustee MUST exercise his/her discretion pursuant to the terms of the trust, even if that decision is not to make a payment to the beneficiary. In assessing the trustees decision-making process, the courts have kept in mind the fiduciary duty that the trustee owes the beneficiary 3. Duty to inquire: Consistent w/ the fiduciary duty the trustee owes the beneficiary, before the trustee can exercise discretion wrt whether to make a payment to the beneficiary, the trustee has a duty to inquire as to the beneficiarys status and needs. This includes a duty of due diligence in attempting to gather the relevant information, and a duty to follow up on unsuccessful attempts. If the trustee fails to inquire, the trustee is deemed to have breached his/her fiduciary duty to the beneficiary. 4. Duty to Act Reasonably: Objective standard to act as a reasonable trustee would act. a. Extended Discretion: Some trusts authorize the trustee to act in his/her sole discretion, or sole and absolute discretion. But courts construe this as not TRULY meaning sole/absolute discretiontrustees MUST still have a fiduciary duty. W/o fiduciary duty, the trust would fail. Such language virtually eliminates the duty to act responsibly, but the trustee must still act in good faith. (UTC 814) 5. Duty to Act in Good Faith: Subjective standardthe trustee acts in good faith as long as he/she honestly thought that he/she was acting in the best interests of the beneficiaries and the trust in making decisions. 6. Duty to Consider a Beneficiarys Other Resources: Whether a trustee should/must consider a beneficiarys other resources in deciding whether to make a payment to the beneficiary is a question of settlors intenti.e. did the settlor intend to provide a minimum level of income for the beneficiary, regardless of the beneficiarys other sources of income, or did the settlor merely intend to provide a safety net? a. Presumption of Steady Income: Where the settlors intent is unclear, most courts presume that the settlor intended to provide for beneficiary regardless of the beneficiarys other resourcesthe trustee is not to consider other resources UNLESS the trust expressly authorizes it. 7. Marsman v. Nasca (Discretionary Trusts Trustees Breach of Duty) (p. 598)

Chapter 9 Rights to Distributions from the Trust Fund

91 of 120

8.

F/P a. Sara Marsman created a testamentary trust that provided that the trustees were to pay the income to her husband (Cappy) at least quarterly, and after having considered the various available sources of support for him, my trustees shall, if they deem it necessary or desirable, in their sole and uncontrolled discretion, pay over to him, such an amount of the principal thereof as they deem advisable for his comfortable support and maintenance. During Saras life, Sara and Cappy lived well. After Sara died, Cappy lost his employment, and his standard of living fell substantially. The principal of the trust was $65,000. When Cappy brought his plight to the trustees attention, the trustee gave Cappy a minimal distribution of principal ($300) and asked Cappy to explain in writing the need for the principal. Cappy failed to reply, and the trustee failed to follow up. For approximately ten years, the trustee never inquired into the plaintiffs financial state and only paid him $300 during the nearly 10 year time span. The plaintiff experienced financial difficulty due to his failing business during the nearly 10 year period Whether a trustee that holds discretionary power to pay principal for the comfortable support and maintenance of a beneficiary, has a duty to inquire into the financial resources of the beneficiary to determine his needs?

b.

9.

I a.

HELD: The trustee had breached the duty to inquire into/follow up on the financial status of the beneficiary. b. The will that gave the trustee the power to pay principal for the support and maintenance of the beneficiary had a duty to inquire into the financial resources of the beneficiary. Despite the broad discretion in the trust, Cappys standard of living had been reduced substantially. In light of the settlors intent that the principal was to be used to maintain Cappys comfortable support and maintenance, the trustee had breached the duty. c. A trustee who is given discretionary power to pay the principal for the beneficiarys support and maintenance must inquire into the financial resources of the beneficiary 11. Exculpatory Clauses: Discretionary clauses often include an exculpatory clause protecting the trustee against liability for breach of trust absent willful neglect or the like. a. Enforceability of Exculpatory Clauses: i. Abuse of Confidential Relationship: If the ct concludes that the exculpatory clause was put in the trust because of the trustees overreaching or abuse of fiduciary or confidential relationship, the clause will be deemed null and void. ii. Intentional, Bad Faith Breaches: If the ct concludes that the breach of trust was intentional, in bad faith, or in reckless disregard for the beneficiarys interest, the clause will not be enforced. iii. Traditional/General Rule: A beneficiary challenging the validity of an exculpatory clause bears the burden of proof (i.e. burden of proving intentional or bad-faith breach) iv. UTC Approach: If the trustee drafted the exculpatory clause or caused it to be drafted, the clause is presumed to be invalid. 1. Rebuttable Presumption: If the trustee proves the exculpatory clause is fair under the circumstances and that its existence and contents were adequately communicated to the settlor, then the clause is valid (UTC 1008(b)).

10. R/A/H a.

Chapter 9 Rights to Distributions from the Trust Fund

92 of 120

2.

v. New York Approach: Exculpatory clauses granting immunity to trustees to failure to exercise reasonable care are void, because they violate public policy. vi. Example of Enforceability: In Marsman above, the TRUSTEE inserted an exculpatory clause into the trust, protecting the trustee as long as he didnt commit willful neglect or default. The trustee testified that he discussed the clause with the settlor during the drafting process, and the settlor approved the clause. 1. Valid Exculpatory Clause: though the trustee drafted the clause and suggested the clause, the ct held that the clause was valid (UTC approach). The ct said the trustees conduct constituted a breach of duty, but the breach was not intentional, in bath faith, or in reckless disregard for the beneficiarys interest. 12. Mandatory Arbitration Clause: Increasingly, trusts contain mandatory arbitration clauses a. Policy: Minimizes administration fees and expedites the resolution of disputes b. Rule: Cases are scarce Schoneberger v. Oelze (p. 609) allowed the beneficiary to litigate in court in spite of the trusts mandatory arbitration provision i. But An argument can be made that the clauses should be subject to the same analysis as exculpatory clauses, and should not be per se invalid. 13. Sprinkle/Spray Trust: A sprinkle or spray trust requires the trustee to distribute the property in question to a group of individuals, and the trustee has discretion as to whom to make the payments and how much each receives. a. Hybrid Trust: To the beneficiaries, the trust is discretionary (they dont know who will receive how much). To the trustee, the trust is mandatory (they MUST distribute the property; but as to whom and how much, the trust is discretionary). 14. Unitrust: Under a unitrust, a life beneficiary is given a fixed annual percentage interest in the total worth of the trust, regardless of whether the property needed to satisfy that fixed interest comes from the income or the principal. The trustee is then free to pursue any investment that he/she thinks will produce the greatest benefit for the trust, regardless of the amount of income the investment produces, because the trustee has the power to disburse income and principal. 15. Perpetual Dynasty Trust: An increasing number of jurisdictions are abolishing the Rule against Perpetuities. This permits a trust to last forever. Where a settlor creates a trust that will last forever for the benefit of ones issue, the trust is called a Perpetual Dynasty Trust. a. Distribution: The trustee has discretionary powers over both the income and the principal; thus the trustee may manage the trust to allow the corpus to remain intact, if not grow, while creating a stream of income for the settlors descendants. Rights of the Beneficiarys Creditors: Some trusts offer asset protection features. 3 main ones: (1) Discretionary trusts, (2) spendthrift trusts, and (3) self-settled asset protection trusts. a. Creditors Rights Generally: A creditor can reach a debtors property as long as the property interest in question is transferable. Absent a special provision in a trust, generally a beneficiarys interest in freely transferable, whether the beneficiarys interest is beneficiary or mandatory. i.e. the creditor can reach a beneficiarys interest in the trust. i. Scope of Creditors Rights: Generally, a creditor only receives whatever interest the beneficiary has in the trustno more, no less.

Chapter 9 Rights to Distributions from the Trust Fund

93 of 120

b.

Mandatory Trusts: The creditor can force the trustee to distribute the income to the creditor pursuant to the terms of the trust, just as the beneficiary could have. Discretionary Trust: Two types of discretionary trusts: (1) Pure Discretionary Trusts, and (2) Support Trusts i. Pure Discretionary Trust: Trustee has absolute, sole, or uncontrolled discretion over distributions to the beneficiary 1. Traditional Approach: Creditor has no recourse against the beneficiarys trust interest. Creditor cannot compel the trustee to pay him. a. Policy: Because the beneficiary has no right to compel distribution, neither does the beneficiarys creditor (R. 2d Trusts 155, cmt b) b. But Court Order: Even though the creditor cannot compel the trustee to satisfy a debt of the beneficiary if the trust is discretionary, the creditor MAY be entitled to a court order directing that when the trustee does decide to pay the beneficiary, the trustee must pay the creditor before making any paying the beneficiary. (Hamilton v. Drogo, p. 610) 2. Modern Approach: Both the R.3d Trusts and UTC collapse the distinction between discretionary trusts and support trusts. (see below, p. 94) ii. Support Trust: Trustee is obligated to make distributions as necessary for the beneficiarys needs (e.g. support/education). 1. Traditional Rule: The beneficiary of a support trust cannot alienate her interest. Therefore, creditors of the beneficiary cannot reach the beneficiarys interest. a. Basic Necessities Exception: Suppliers of necessaries may recover through the beneficiarys right to support (R.2d Trusts 154). b. Child Support and Alimony Exception: Children and spouses may enforce claims for child support and alimony against a beneficiarys interest in a support trust (R.2d Trusts 157). 2. Construction of Support Trust Language: The key to classifying a trust as a support trust is the formula that controls how much the trustee can distribute to the beneficiary, not the use of the word support per se. a. Excample: Where the payment IS limited to the amt necessary for support/education, the trust qualifies as a support trust. If the trustee is required to distribute ALL of the income to the beneficiary for his/her support, the trust is not a support trust because the amount being paid out is not limited to the amount necessary for the beneficiarys support. 3. Discretionary Support Trust: Not recognized as a separate category, but still common: Combines an explicit statement of unfettered discretion with a distribution standard (e.g. see Marsman, above p. 91). a. Creditors Rights: Wrt to creditors rights, courts tend to treat these more as pure discretionary trusts than as support trusts. iii. Modern/UTC Approach to Creditors Rights: Modern trend and UTC is to abolish the distinction btwn discretionary and support trusts. Creditors may reach the beneficiarys trust interest, but the Restatment and UTC take different approaches. 1. R.3d Trusts 60: if the terms of a trust provide for a beneficiary to receive distributions in the trustees discretion, a transferee or creditor of the beneficiary is entitled to receive or attach any distributions the trustee makes or is required to make in the exercise of that discretion. 2. UTC 504: Subject to an exception for claims by children and spouses for child support and alimony, a creditor of a beneficiary cannot compel a discretionary distribution, even if the beneficiary could compel such a distribution. (see casebook p. 612) iv. Protective Trust: A protective trust is a hybrid asset-protecting trust. The beneficiarys interest is mandatory until a creditor attaches a beneficiarys interest (i.e. because beneficiary falls into debt). At that point, the beneficiarys interest automatically becomes discretionary (to protect it against the claim of the creditor). 1.

Chapter 9 Rights to Distributions from the Trust Fund

94 of 120

c.

Favorable in Jurisdictions that dont recognize Spendthrift Clauses: Protective trusts are popular in jurisdictions that dont recognize spendthrift clauses, and they are the norm in England (where spendthrift clauses are not recognizedsee p. 614). Spendthrift Trust: A spendthrift clause bars a beneficiarys ability to transfer his/her interest voluntarily (by sale or gift) or involuntarily (by creditors reaching it). (R.3d Trusts 58) i. Policy Issue: The spendthrift trust arises from the settlors right to condition the terms of her transfer. The issue becomes whether or not to allow the dead hand to restrain alienation of the beneficial interest. ii. Majority Rule: A trust is not spendthrift (nontransferable) unless the settlor expressly inserts a spendthrift clause iii. New York Rule: A beneficiarys interest is presumed spendthrift (nontransferable) unless the trust expressly provides otherwise. iv. England Rule: England dont be recognizin spendthrift clauses at all, yo. Word, son. v. UTC Rule: A spendthrift provision is valid ONLY IF it restrains both voluntary AND involuntary transfer of a beneficiarys interest (UTC 502(a)) vi. Enforceability of Spendthrift Trusts: Generally, spendthrift clauses are valid and enforceable, even as applied to remaineder interests in trust vii. Exceptions: Although spendthrift clauses are generally valid, exceptions apply: 1. Judicial Exceptions: Public Policy certain categories of creditors are not subject to spendthrift clauses: (1) ex-spouses entitled to spousal support (alimony); (2) children entitled to child support; (3) creditors who provide basic necessities; (4) tax claims by the state or federal govt a. But Tort Creditors Are Screwed in Most Jurisdictions: Majority of jurisdictions still apply spendthrift clauses to tort creditors 2. UTC Approach: UTC limits exceptions to (1) children entitled to child support pursuant to a judgment or court order; (2) spouses/ex-spouses entitled to spousal support (alimony/maintenance) pursuant to a judgment or court order; (3) a claim by a state/federal govt; and (4) a judgment creditor who has provided services for the protection of a beneficiarys interest in the trust. (UTC 503, see CB p. 620) a. See also R.3d Trusts 59(a): Agrees w/ UTC 503. b. Minority (substantially small) Approach: Spouse/child cannot reach a spendthrift trust to satisfy a judgment for support. 3. Scheffel v. Krueger (Spendthrift Trusts and Tort Creditors) (p. 616) 4. F/P a. Kruegers grandmother set up a trust for his benefit. He had a mandatory interest in the income, discretionary interest in the principal (for his maintenance, support, and education), and the trust had a spendthrift clause. b. Some time later, Krueger sexually assaulted a minor, videotaped it, and broadcast it over the internet. He was found liable for several sexual assault charges and also faced criminal charges. He also had a tort judgment of over $500,000 entered against him. 5. I a. Whether a trust purpose for support and maintenance may still be fulfilled where the beneficiary faces a criminal sentence for sexual assault. b. Whether a statute that bars creditors from claiming an interest in a beneficiarys trust makes an exception for tort creditors. 6. R/A/H a. HELD 1: Yes the purpose of support and maintenance trust may still be fulfilled while the beneficiary is incarcerated and after he is released. b. HELD 2: Yes the spendthrift clause barred reaching Kruegers interest in the trust. c. RULE: A statute that bars creditors from claiming an interest to a beneficiarys trust does not make an exception for tort creditors. 1.

Chapter 9 Rights to Distributions from the Trust Fund

95 of 120

d.

Shelley v. Shelley (Spendthrift Trusts and Child Support Creditors) (p. 618) F/P a. The settlor created a trust for the benefit of his son. The sons interest in the income was to be mandatory (income to be paid for life). The trustee was to begin distributing corpus to the son after he reached age 30 in amounts that the trustee (and other named persons) deemed the son to be able to invest properly. The trust also authorized the trustee to disburse principal to the son, or his children, in an emergency. The trust had a standard spendthrift clause. b. The son owed both alimony and child support payments to his ex-spouses and children. When the son disappeared, the ex-spouses and children sued seeking to reach the sons interest in the trust. 9. I a. Whether a spendthrift provision bars a beneficiarys former spouse and child from collecting alimony and child support? 10. R/A/H a. HELD: No The creditors were not subject to the spendthrift clause. The children were entitled to payments of the principal in their capacity as beneficiaries of the trust, not as creditors. b. RULE: A spendthrift provision does not bar the claims of a beneficiarys children and former spouse for child support and alimony in regards to the income of a trust. However such claims are barred in regards to discretionary payments from the trust corpus. c. The spendthrift clause does not bar the ex-spouses and children from reaching the sons mandatory interest in the income. d. Moreover, the ex-spouses and children were also entitled to reach the sons interest in the principal (corpus). But because the sons interest was discretionary, they could not force the trustee to make disbursements to them as creditors. However, because the children were beneficiaries of the principal in their own right, the trustees failure to disburse principal to them as beneficiaries was an abuse of discretion viii. Statutory Limitations on Creditor Protection: Some states statutorily limit the amount of the beneficiarys interest that can be protected against creditors claims by a spendthrift clause. 1. There are 3 statutory main approaches: a. (1) Limit the amt of a beneficiarys interest that can be shielded from creditors claims by a spendthrift clause to the amount necessary for the beneficiarys support and education; allow the creditor to reach the rest b. (2) Permit a creditor to reach a fixed percentage (usually less than 1/3) of a beneficiarys interest in the income; c. (3) Fixed dollar amount cap on the amt of money that can be shielded from creditors claims by a spendthrift clause; allow the creditor to take the rest 2. Pension Trusts and the Employee Retirement Income Security Act (ERISA): ERISA requires that each pension plan covered by the act provides that may not be transferred. Current, non-family creditors MAY NOT reach the beneficiarys interest. However, the benefits MAY be reached for child support, alimony, or marital property rights. 3. Bankruptcy Law and Trust Asset Protection: The Bankruptcy Code provides that a beneficiarys interest in a trust passes to the bankruptcy trustee ONLY IF the beneficiarys interest is transferable. If the trust has a spendthrift clause, the beneficiarys interest is not reachable in bankruptcy. Creditors Rights when Settlor is the Beneficiary (Self-Settled Asset Protection Trusts): The above listed creditors rights scenarios assumed that the beneficiary was a different person than the settlor. Here, the beneficiary IS the settlor (and the trustee is someone else)

7. 8.

Chapter 9 Rights to Distributions from the Trust Fund

96 of 120

i. Traditional Rule = the settlor cannot use a trust to shield his own assets from his own creditors. Even if the trust is discretionary, spendthrift, or both, the settlors creditors can reach the maximum amt that under any circumstances the trustee could pay to the settlor or apply for the settlors benefit. (Traditional Rule is carried forward in R.3d Trusts 58(2), 60, cmt f) and UTC 505). 1. Mandatory Interest: If the settlor retained a mandatory interest in the trust, creditors of the settlor can reach the mandatory interest in the trust. If the trustee fails to make the payment to them, the creditors can compel the trustee to make the payment to them 2. Discretionary Interest: If the settlor retained a discretionary interest in the trust, creditors of the settlor can reach the discretionary interest to the full extent that the trust permits the trustee to use the trust for the benefit of the settlor 3. Spendthrift Clause: Generally, spendthrift clauses are null and void as applied to creditors of a beneficiary who is also the settlor. It is against public policy to permit one to shield ones assets in a spendthrift trust. ii. Modern Trend: In an attempt to attract trust business some states have adopted statutes authorizing self-settled Asset Protection Trusts (APTs) 1. Alaska/Delaware: Settlors creditors have no recourse against the settlors interest in a self-settled spendthrift trust, as long as the initial transfer was not fraudulent a. Rules: APTs are permitted in favor of a beneficiary who is also the settlor if (1) the trust is irrevocable, (2) the trust interest is discretionary, and (3) the trust was not originally created to defraud creditors b. Locale: Settlors in any state can take advantage of these exceptions as long as the trusts are created in the states that allow such trusts. c. This approach is followed in 11 states: AK, DE, MO, NH, NV, OK, RI, SD, TN, UT, WY i. ILLINOIS DOES NOT ALLOW APTs 2. Offshore: e.g. Bahamas, Barbados, etc. islands allow self-settled trusts against which settlors creditors have virtually no recourse iii. Federal Trade Commission v. Affordable Media, LLC (Self-Settled Asset Protection Trusts) (p. 628) iv. F/P 1. Denyse and Michael Anderson created a telemarketing scheme that constituted a Ponzi scheme to defraud investors of millions of dollars, which they tucked away in a Cook Islands trust (an offshore self-titled spendthrift trust). The Andersons were co-trustees, along with AsiaCiti Trust Ltd. 2. The FTC brought suit against the Andersons to recover as much money as possible for the defrauded investors and was granted a preliminary injunction ordering the Andersons to account for the funds held offshore and to repatriate the funds. The Andersons faxed AsiaCiti for an accounting and to repatriate the funds to the states, but AsiaCiti concluded that the district courts preliminary injunction constituted duress voiding the Andersons request, and under the terms of the trust, authorized AsiaCiti to remove the Andersons as co-trustees. 3. The dist ct held the Andersons in civil contempt for failure to account and repatriate the funds and took them into custody. 4. As a general rule, a partys inability to comply with a judicial order constitutes a defense to a charge of civil contempt. The Andersons argued that the terms of the trust prevented them from complying with the court order. The court expressed skepticism as a legal matter as to the applicability of the defense where the inability to comply is the intended result of the defendants own actions (here, the creation of the offshore APT to protect the assets from the reach of US courts). 5. The court ruled that the Andersons had not met their burden of showing categorically and in detail that they were unable to comply with the court order. The court was not

Chapter 9 Rights to Distributions from the Trust Fund

97 of 120

convinced that the Andersons had put the money beyond their control, particularly because the Andersons retained the position of protectors of the trust. Protectors have significant control of offshore trusts, including the appointment of successor trustees and the power to make the anti-duress provisions of the trust subject to their control. v. I 1. Whether a party demonstrates categorically and in detail that he us unable to comply with the repatriation section of a preliminary injunction to transfer to the United States all assets under their control directly and indirectly because the assets are in trust under a trustee that refuses to relinquish the proceeds?

vi. R/A/H 1.

vii. viii.

ix.

x.

xi.

HELD: No. The defendants were protectors of the trust and could have forced the trustees to turn over the proceeds. Furthermore, the defendants showed they were aware of their ability to force the trustees to transfer the money. After they stated that they could not comply with the order, the Commission revealed that the defendants were the protectors of the trust. Thereafter, the defendants attempted to resign as protectors of the trust. 2. RULE: A party petitioning for an adjudication that another party is in civil contempt does not have the burden of showing that eh other party has the capacity to comply with the courts order, but the party asserting the impossibility defense must show categorically and in detail why he is unable to comply. There is a high burden on the defendant to prove impossibility as a defense to a contempt charge 3. United States courts will penalize trustees of trusts in foreign lands if the trustees are domiciled or residents of the United States and do not comply with orders in regards to the overseas funds. 4. EPILOGUE: The district court freed the Andersons six months after finding them in contempt. The FTC sued AsiaCiti in the Cook Islands for $20 million, but settled for only $1.2 million In re Lawrence (p. 634) F/P 1. Lawrence created and funded an offshore asset protection trust in Mauritius with $7 million dollars. After Lawrence lost a securities law arbitration proceeding, he was punished with a $20.4 million judgment against him. Lawrence filed for bankruptcy and the court ordered him to turn over the bankruptcy trustee assets held in the offshore trust. Lawrence did not comply and the court held him in contempt and jailed him pending compliance with the turnover order. I 1. Whether a person charged with contempt is responsible for his inability to comply with an order to turn over assets in an offshore account if he was removed as a beneficiary under the account because he declared for bankruptcy? R/A/H 1. HELD: Yes. Lawrence is responsible for his inability to comply because even though he was removed as a beneficiary as a result of his bankruptcy filing, he still retained the authority to appoint trustees who could have exercised the authority to reinstate Lawrence as a beneficiary. The new trustees could have distributed the entire trust to Lawrence once he was reinstated. Therefore Lawrence is responsible for his inability to comply. Furthermore, it appears as if the sole purpose of the provision that extinguishes his inters tint the event of bankruptcy is to aid the settlor in evading contempt. 2. The court did not accept impossibility as a defense here because settlor still had the power to transfer the assets if he appointed new trustees. Transfers in fraud of creditors: Asset transfers made with the intent to hinder or defraud a creditors claim (as opposed to transfers made before a creditors claim arises) constitute actual fraud and are recoverable under the widely adopted Uniform Fraudulent Transfer Act (UFTA).

Chapter 9 Rights to Distributions from the Trust Fund

98 of 120

3.

4.

xii. Professional Responsibility: Lawyers have an ethical obligation not to counsel a client to engage, or assist a client, in conduct that the lawyer knows is criminal or fraudulent (Model Rule of Professional Conduct 1.2(d)). Lawyers must be careful in creating an asset protection trust that would delay, hinder, or defraud a clients existing or foreseeable creditors. Trusts for the State Supported (eg Public Health Benefits): Public health benefits, such as Medicaid and statesponsored health programs, are usually limited to people who cannot pay for their own health services. People have to use trusts (esp discretionary trusts) to shield their assets from their medical expenses, thereby qualifying for public assistance. a. Issue: Whether such trusts are successful turns mainly on whether the applicant who is the beneficiary contributed to the creation of the trust or whether someone other than the applicant/beneficiary created the trust. b. Applicant-Created Trusts (self-settled trusts): i. Definition: For Medicaid purposes, a trust is self-settled if (1) assets of the individual were used to form all or part of the corpus of the trust and (2) the trust was established by the individual, the individuals spouse, or by a person or court w/ legal authority to act on behalf of, or by request of, the person or spouse. ii. General Rule: Rules favor including the trust property among the applicants resources for determining the applicants eligibility for Medicaid and state-sponsored health programs. iii. Revocable Trusts: The whole trust is considered the applicants property for the purposes of determining eligibility iv. Irrevocable Trusts: The trust property is considered the applicants property to the full extent that any part of the trust could be used for the applicants benefit. v. Exceptions: Two important exceptions to this rule: 1. Testamentary Discretionary Trust: Testamentary discretionary trusts created by the applicants spouse (by will) are not included as individual assets for Medicaid 2. Trusts Established for a Disabled Individual by a parent, guardian, or court: If (1) the trust is established for a disabled individual from the individuals property, by a parent, grandparent, or guardian of the individual or by a court, and (2) the trust provides that the state will receive all unreimbursed medical costs upon the applicants death. c. Third-Party Created Trusts: If the applicant had no role in creating the trust, the beneficiarys interest in the trust is considered part of the applicants resources only to the extent the beneficiary could compel the trustee to make a payment of income or principal (typically in a mandatory trust, but not in a discretionary trust). i. Spendthrift Trusts: As a provider of basic necessities, the govt generally is not subject to a spendthrift clause. ii. UTC Approach: Under UTC, providers of basic necessities generally have no right to reach a beneficiarys interest in a trust, BUT if the State is the plaintiff, then it can. (UTC 503(b)(3)) rd iii. Discretionary Trusts: When a trust is a discretionary trust created by a 3 party, the states ability to reach the beneficiarys interest turns on the settlors intent generally. 1. General Rule: The state cannot reach discretionary trusts, nor consider them when determining eligibility for public benefits. 2. BUT Hybrid Discretionary Trusts w/ Support Purposes: Where the discretionary trust is a hybrid with a support standard under which the beneficiary could force distribution, the State can also force distribution 3. Supplemental Needs Trusts: Where the settlors intent was to provide benefits only that the state is unwilling or unable to provide, the state cannot reach the beneficiarys interest. iv. See also IL Statutes 1. 760 ILCS 35/1 (Trusts and Dissolutions of Marriage Act) 2. 760 ILCS 15/1 to 17 (Principal and Income Act) 3. 760 ILCS 30/1 (Instruments Regarding Adopted Children Act) Modification and Termination of Trusts

Chapter 9 Rights to Distributions from the Trust Fund

99 of 120

a.

b.

c.

Introduction: A trust ends naturally pursuant to its terms. A trust ends when all of the trust res is completely disbursed. A trusts terms will provide for when the trust res (principal) is to be disbursed. i. Issue: If the settlor is dead or does not consent, when may the trust beneficiaries modify or terminate the trust prematurely? ii. Irrevocable Trusts: If the settlor and all beneficiaries consent, an irrevocable trust may be modified or terminated. The trustee cannot object. Such a right exists even if the trust contains a spendthrift clause. iii. Revocable Trusts: If the trust is revocable, the settlor can single-handedly terminate the trust. The power to terminate implicitly includes the power to modifysettlor can revoke the trust and create a new trust with modified terms and conditions. 1. Note: The settlor must comply with the requirements for revoking the trust. The doctrines of trust modification and termination presume an irrevocable trust. Interest in Trust (and ability to modify/terminate trust): i. All three parties consent: if all three parties to a trust (settlor, trustee, beneficiaries) agree to modify or terminate the trust, the trust can be modified or terminated. If any of the parties later changes his/her mind and sues any of the other parties, the suing party will be estopped based on his/her initial consent. ii. Settlor and Beneficiaries Consent (but not trustee): Even if the trustee objects, the trust can be modified/terminated. 1. The trustee must assert the settlors intent. The trustee has no beneficial interest in the trust. iii. Trustee and Beneficiaries Consent (but not settlor): Assuming the settlor has no interest in the trust (an irrevocable trust), if all the beneficiaries consent and the trustee consents, the trust can be modified/terminated. 1. The beneficiaries will be estopped from suing later for breach of duty, because they consented earlier to the change/terminate the trust. iv. Beneficiaries Consent, Trustee Objects (Settlor is Dead): If all the beneficiaries consent, but the trustee objects and the settlor is dead, the jurisdictions are split. 1. English Approach: If all the beneficiaries consent, the trust is modified or terminated regardless of the terms of the trust or the trustees objections a. Dead hand control generally is not permitted. After the death of the settlor, the beneficiaries are deemed the owners of the trust property for purposes of modification and termination of the trust. 2. Traditional American Approach: The trustee has the right (and to some degree, the duty) to object to a modification or termination. If all the beneficiaries consent, the trust is modified/terminated regardless of the terms in the trust or the trustees objections. a. Rationale: By protecting a deceased settlors intent as expressed in the trust, future settlors will be encouraged to create trusts, because they know that as a general rule, the courts will protect and uphold their intent even after their death (if the trustee objects). Trust Modification (Deviation and Changed Circumstances): i. Settlors Intent: Under the common law doctrine of modification, the assumption is that the modification is to further the settlors intent. ii. R.2d Trusts 167 The court may permit the trustee to deviate from a term of the trust if circumstances not known to the settlor and not anticipated by him would defeat or substantially impair the accomplishment of the purposes of the trust iii. Unforeseen Change: Fact-intensive inquiry. 1. Common Law: Courts tend to: Courts protect settlors intent, even against attempts at modification. Courts apply a rather high threshold for what constitutes an unforeseen change in circumstances

Chapter 9 Rights to Distributions from the Trust Fund

100 of 120

Modern Trend: There is a noticeable shift toward giving the beneficiaries greater control over the property in the trust after the settlors death (i.e. lower threshold for what constitutes an unforeseen change in circumstances) a. Eg: An unusually high rate of inflation or increased medical costs can be enough to constitute an unforeseen change 3. Beneficiarys Advantage: The mere fact that the proposed modification would be more advantageous to one or more beneficiaries is not enough to warrant modifying a trust, even if all beneficiaries agree. a. In re Trust of Stuchell (p. 643) b. F/P i. The Petitioner sought to modify a trust in regards to one of the remainder beneficiaries because he is mentally repaired. Without the amendment, if the disabled beneficiary survived the other two lifeincome beneficiaries, he would receive the remainder of funds outright. The petitioner, with the consent of all of the other income beneficiaries and remainder men, sought to modify the trust so that it would not be distributed directly to him if he survived the others, but rather continue to exist as a secondary source of funds to supplement the money he receives from pubic assistance. c. I i. Whether a trust may be amended where all but one of the beneficiaries approves and the trust as amended, but only makes the trust more advantages to the beneficiaries. d. R/A/H i. HELD: No. The trust may not be amended where the proposed amendment would only be more advantageous to the beneficiaries. Here the proposed amendment would allow the trust to continue only as a secondary supplement to the current income and benefits that the beneficiary receives from public assistance. ii. RULE: A court will not permit or direct the trustee to deviate from the terms of the trust merely because such deviation would be more advantageous to the beneficiaries than a compliance with such direction. iii. Here the modification only benefited the other beneficiaries. Even though the handicapped beneficiary would not receive less money than he was currently receiving through public assistance, the other beneficiaries would receive more money from the trust. iv. Substantially Impair: Whether an unforeseen change in circumstances defeats or substantially impairs the settlors intent is a very soft, fact-sensitive inquiry 1. Common Law Approach: Protect settlors intent: Apply a high threshold for finding that the change (in circumstances) defeats or substantially impairs settlors intent. 2. Modern Trend/UTC Approach: Favors granting beneficiaries greater power over the trust (low threshold for what constitutes defeating/substantially impairing settlors intent. a. UTC: Ct may modify the administrative or dispositive provisions of a trust if, because of circumstances not anticipated by the settlor, modification would further the purposes of the trust. v. Beneficiaries Consent: General Rule: Even if an unforeseen change in circumstances defeats or substantially impairs the settlors intent, before a court directs or permits modification, ALL of the beneficiaries must consent. There are doctrines to get consent from beneficiaries who lack the capacity to consent or from future beneficiaries who might not be born yet. 2.

Chapter 9 Rights to Distributions from the Trust Fund

101 of 120

Guardian ad litem: Method of getting consent from minors or unborn beneficiaries petition the court for an appointment of a guardian ad litem to represent the interests of the minor or unborn beneficiaries a. Traditional Approach: Guardians ad litem take a strict/conservative approach to representing the minor or unborn beneficiary, asking only whether the proposed modification would increase or decrease the economic value of the interest the guardian was appointed to protect. b. Modern Trend: The courts have encouraged guardians ad litem to take into consideration of non-economic factors, such as family harmony and the settlors apparent primary intent to take care of other family members. 2. Virtual Representation: Some courts and the UTC held that under the doctrine of virtual representation, if the interests of the minor or unborn beneficiaries are virtually identical to those of living adult beneficiaries are deemed to speak not only for themselves, but also for the interests of the minor or unborn beneficiaries by virtual representation. 3. Modern Statutory Trends: Make it easier to get consent of all beneficiaries by reducing the pool of beneficiaries who have to consent or by permitting the court to order modification, even in the absence of all the beneficiaries consenting. a. UTC: UTC requires the consent of only qualified beneficiaries for the removal of a trustee. A qualified beneficiary is one who would be entitled to receive property if the trust was terminated on the day the petition was filed. i. UTC Also authorizes the COURT to order modification or termination, without requiring the consent of all beneficiaries if (1) the trust could have been modified if all the beneficiaries had consented, and (2) the interests of the non-consenting beneficiaries are adequately protected. (UTC 411(e)) b. Modifying Trusts to Achieve Tax Benefits: A handful of states, and both the R.3d of Property, Donative Transfers and the UTC authorize modification of trusts to further a settlors apparent tax minimizing objectives. (see CB p. 651) vi. Administrative Modification: Generally, under the unforeseen change in circumstances doctrine, courts are more willing to modify administrative provisions than they are to modify distributive provisions. 1. Rationale: Modifying administrative provisions changes only how to administer the settlors intent, while still honoring that intent. Modifying distributive provisions may go against the settlors intent. 2. UTC: UTC authorizes a court to modify the administrative provisions of a trust if continuing the current administrative procedures would be impractical or wasteful or impair the trusts administration. (UTC 412) see CB p. 645 3. Restatement: R.3d Trusts 66 authorizes courts to modify administrative AND distributive provisions (see CB p. 645). 4. In re Riddell (Court-ordered Trust Modification) (p. 645) 5. F/P a. George and Irene Riddell created multiple trusts to benefit their only son Ralph and his wife Beverly; and upon the death of the latter of them, to provide benefits to Ralphs children until they reach the age of 35, when the principal would be distributed outright to them. Ralph had 2 kids (Donald and Nancy), both of whom were over age 35. Nancy suffered from schizophrenia and lives in a state hospital George and Irene were not aware of this. b. Ralph was the trustee. Ralph filed a petition to consolidate the trusts and create a special needs trust for Nancy (instead of disbursing her payment outright) that would (1) manage Nancys funds for her benefit, (2) avoid the state seizing the funds to be reimbursed for medical expenses, and (3) avoid Nancys mismanagement of the money. 1.

Chapter 9 Rights to Distributions from the Trust Fund

102 of 120

c. 6. 7. I a. R/A/H a. b.

Trial granted motion to consolidate trusts, but not to modify Should the trustee be allowed to modify the trusts?

d.

HELD: Ralph (trustee) should be allowed to modify the trusts. RULE: Equitable Deviation (R.3d Trusts 66): The court may modify an administrative or distributive provision if (1) because of circumstances not anticipated by the settlor and (2) the modification or deviation will further the purposes of the trust. c. The court found that (1) Nancys special needs constituted circumstances not anticipated by the settlors because George and Irene were unaware of them, (2) the special needs modification will further the purposes of the trust, and (3) the modification was not against public policy. d. George and Irenes intent was not to lose the money to the state and prevent Nancy from passing the money to her son. vii. Trust Protectors: A Trust Protector can be given powers and control over a trust similar to those held by the settlor of a revocable trust (e.g. protector may terminate or modify the administrative or distributive provisions of the trust in response to changed circumstances). 1. Fiduciary Duty: Whether the Trust Protector owes the beneficiaries a fiduciary duty is up for debate a. UTC 808(d): A person who has the power to direct the trustee is presumptively a fiduciary with a duty i. HOWEVER, this rule is not included in the schedule of mandatory rules in UTC 105, so it may be overridden by the terms of the trust b. R.3d Trusts 75: Trust protector is generally a fiduciary c. Alaska State Law: Trust protector is NOT presumptively a fiduciary d. Delaware State Law: Trust Protector is presumptively a fiduciary, but the trustee has no duty to monitor the protector or to notify the beneficiary if the trustee disagrees w/ protectors judgment. Trust Termination: Premature termination where all the beneficiaries consent, but the trustee objects, troubled courts. On one hand, courts wanted to protect settlors intent on the theory that the trust, not the beneficiaries, owned the trust property. On the other hand, the courts were suspicious of the trustees objections because of the trustees vested interest in receiving trustee fees. i. The Claflin Doctrine: Doctrine handles conflict of interest between beneficiaries and trustee. 1. Policy: Favors settlors intent more than beneficiaries rights. 2. Rule: If continuance of the trust without modification or termination is necessary to carry out a material purpose of the settlor, then the beneficiaries cannot compel modification or termination. However, if there is no remaining unfulfilled purpose, and all the beneficiaries, they can compel modification/termination. 3. Unfulfilled Material Purpose: What constitutes an unfulfilled material purpose is a question of fact centers on the language and apparent purpose of each trust. a. Some trusts inherently have unfulfilled purposes: (1) discretionary trusts, (2) spendthrift trusts, (3) support trusts, (4) trusts where the property is not to be disbursed until the beneficiary reaches a specific age. b. If the court determines that the dispositive provisions of the trust constitute merely a succession of interests that have no material purpose, premature termination is ordered, if all the beneficiaries consent. c. In re Estate of Brown (Trust Termination Claflin Doctrine/Material Purpose) (p. 653) d. F/P i. Brown created a trust to be used for the education of the children of his nephew, Woolson S. Brown. Upon completion of that purpose, the trust income and principal were to be used for the care, maintenance,

Chapter 9 Rights to Distributions from the Trust Fund

103 of 120

and welfare of his nephew, Woolson S. Brown and his wife Rosemary Brown, so that they would be able to live in the style and manner to which they were accustomed for the rest of their lives. Upon the death of the survivor, the trust res was to be distributed to their thenliving children equally. ii. When the educational purpose had been fulfilled, all the beneficiaries to terminate the trust early. The trustee objected. e. I i. Whether a trust is a support trust where the trustee must distribute all of the remainder income to specified beneficiaries after the initial purpose of the trust is fulfilled? ii. Whether the material purpose of a trust was fulfilled after the education of the settlors nephews children? R/A/H i. HELD: Nocourt declined to terminate the trust. ii. RULE: The Claflin rule (see above, p. 103) iii. The trustee must pay an amount to the remainder beyond the extent necessary for their support; the trustee must pay the amount of the remainder of the trust income to Woolson and Rosemary Brown as is as needed for them to live in the style and manner to which they are accustomed for the remainder of their lives. iv. The material purpose of a trust that provides for the education of a beneficiarys children ,and then for the beneficiary and his wife to live in a lifestyle to which they were accustomed, is not satisfied after the beneficiarys childs education is complete. The settlor did not merely name successive beneficiaries, but expressed intent to provide for the lifelong income of the beneficiaries. Therefore this the second purpose is material Modern Trend/UTC Approach: i. State Law: Some states have relaxed conditions under which a trust may be terminated/modified by request of the beneficiaries ii. R.3d Trusts 65: Beneficiaries may compel the modification or termination if all of the beneficiaries consent. The modification must be consistent with a material purpose of the trust; otherwise, the beneficiaries need (1) the settlors consent, or (2) if the settlor is dead, court authorization. (see CB p. 656) iii. UTC 411: Court may order 1. (1) termination if all beneficiaries consent and continuance of the trust is not necessary to achieve any material purpose of the trust; 2. (2) modification if all beneficiaries consent and modification is consistent with a material purpose; 3. (3) termination or modification without the consent of all beneficiaries if: a. The trust could have been terminated if all the beneficiaries had consented AND b. The interests of the non-consenting beneficiaries are adequately protected Spendthrift Trusts and Material Purpose: In UTC 411(c) and R.3d Trusts 65, spendthrift clauses do NOT give rise to material purpose. Spendthrift clauses merely provide some indication that settlor had a material purpose, but they are not of themselves enough to establish material purpose.

f.

g.

h.

Chapter 9 Rights to Distributions from the Trust Fund

104 of 120

e.

Termination of Trusts because of Probate Settlement: Where there is litigation during probate, and the heirs and trust beneficiaries reach a settlement that includes terminating the trust: i. Majority Approach: Most (but not all) courts enforce the settlement and terminate the testamentary trust, despite its terms (even if there is an unfulfilled material purpose). ii. Minority Approach: Book didnt say probably honors the trust j. Trust Revocability vs Irrevocability Default Rule: i. Majority Rule: A trust is IRREVOCABLE unless the trust expressly provides otherwise ii. Minority/UTC Rule: The trust is REVOCABLE, unless the trust expressly provides otherwise (UTC 602) k. Trust Revocation by Will: Question arisesMay an inter vivos trust be revoked by a will (especially if the inter vivos trust is used as a will substitute?) i. Traditional Approach: Noa will cannot revoke an inter vivos trust unless the trust expressly provides otherwise. ii. UTC/Restatement Approach: UTC 402, R.3d Trusts 63, R.3d Wills 7.2 expressly authorize a subsequent will (a.k.a. codicil) to revoke a revocable trust, or a provision in a trust. 1. Rule: The trust may be revoked (in full or in part) by a codicil where (1) the will expressly devises the property that otherwise would have passed under the trust, (2) the trust does not provide (i) for how it is to be revoked, or (ii) that the method provided is not stated to be exclusive Removal of Trustees: Traditional Approach to removing trustees is that the settlors intent controls. If the settlor selected a particular trustee, that trustee cannot be removed, even if all the beneficiaries consent, unless the trustee is unfit to serve or commits a serious breach of trust. i. UTC Approach (UTC 706): UTC expands on Traditional Approach. (see p. 660) 1. Under UTC, a settlor, a co-trustee, or a beneficiary may request the court to remove a trustee 2. A trustee can be removed if a. (1) there is a material breach of trust; b. (2) infighting among co-trustees substantially impairs the trusts administration; c. (3) the trust has underperformed persistently and substantially, relative to comparable trusts; OR d. (4) there has been a substantial change of circumstances OR all beneficiaries request a change of trustee, and the court finds that the removal best serves the interests of all the beneficiaries and is not inconsistent with a material purpose of the trust. ii. Davis v. U.S. National Bank Association (p. 660) not assigned!! HaHA, bitch. I aint readin that. i.

Chapter 10 Trust Administration: The Fiduciary Obligation


1. Introduction: The trustee has fiduciary obligations to the beneficiary if the trustee manages the trust property poorly, the beneficiaries bear the loss. a. Historically: Trusts were funded primarily w/ real propertythe primary duty of the trustee was to preserve the real property for the beneficiaries. The trustee had no inherent powers over the trust rd property; 3 parties who dealt with the trustee had a high duty to inquire into the propriety of transactions b. Modern Trend: Trusts are typically funded w/ intangible assets (stocks, bonds, etc)the primary duty of the trustee is to manage the fund of wealth. The trustee is granted by law all the powers a reasonable rd person would need to manage the trust, and 3 parties interested in dealing w/ the trustee have no duty to inquire absent suspicious circumstances.

Chapter 10 Trust Administration: The Fiduciary Obligation 105 of 120

2.

3.

Agency Theory: The risks inherent in a trust are analogous to the risks inherent in the use of an agent. It is impossible for the parties (principal and agent; settlor and trustee) to anticipate and contract with respect to every conceivable scenario so the relationship is governed primarily by the more flexible fiduciary principle. But in the trust context, the beneficiaries do not have the same control over the trustee that a principal typically has over an agent. So, traditionally, courts were stricter in their application of fiduciary principles to trustees. Trustees Powers: a. Common Law: At common law, trustee has no inherent powers. Trustee possesses only those powers either expressly granted in the terms of the trust or those necessarily implied in light of the trust purposes. b. Judicial Authorization: A trustee can petition a court of equity for authorization to undertake an action not expressly or implicitly authorized under the terms of the trust. In essence, the court may grant the trustee the requested additional power. c. Modern Trend: The modern trend is to facilitate the granting of powers to the trustee. Two approaches: i. Statutory List: Jurisdiction adopts a statute that sets forth a long list of powers it presumes a trustee would need. Thus, the drafter may specify the trustees powers in the trust instrument by referring to the statutory list of trustees powers (i.e. incorporation by reference) (see also p. 674, for UTC 815, UTC 816) ii. Inherent Powers: Statutorily grant the trustee a broad set of basic powers, unless the settlor expressly provides that the trustee is NOT to have one or more of the granted powers. (Typically the statute provides that a trustee is presumed to have all the powers a reasonable person would need to perform the acts necessarily in light of the purposes of the trust) d. Third Parties Liability: A trustees powers to act are only effective if third parties are willing to deal w/ a rd trustee. Because 3 parties may be held liable for participating in a breach of trust, the protection given rd 3 parties who deal w/ a trustee affects the practical scope of the trustees powers. rd i. Common Law: Imposes virtually strict liability on 3 parties if the transaction constitutes a breach of trust. 1. Policy: Generally presumed that the purpose of the trust is to preserve trust property. rd If the 3 party knows or should know he/she is dealing w/ a trustee, the party has a duty to inspect the trust instrument to see if the transaction is authorized and is charged w/ proper interpretation of the trust rd ii. Modern Trend: The trustee has greater powers, and 3 parties dealing with a trustee have greater protection. 1. Uniform Trustees Powers Act: Eliminates the duty to inquire into the terms of the trust rd and protects 3 parties unless they have actual knowledge that the transaction constitutes a breach of trust. (UTPA 7, see p. CB 674) rd 2. UTC: Requires 3 parties to act in good faith and give valuable consideration. (UTC 1012(a)-(b), see CB p. 675) The Duty of Loyalty a. Introduction: The most fundamental principle of the fiduciary obligation in trust law is the duty of undivided loyalty to the beneficiary. The trustee must administer the trust SOLELY in the interest of the beneficiary. i. Test/Application: The duty of loyalty breaks down into 2 parts: 1. Duty to act Reasonably: Objective standardpermits judicial review and supervision of the trustees actions even where the trustee acted in good faith. 2. Duty to act in Good Faith: Subjective standardaddresses the trustees state of mind; the trustee must have thought that what he/she was doing was in the beneficiaries best interests b. Duty Against Self-Dealing: Self-dealing arises where the trust and the trustee engage in a transaction. The trustee has a conflict of interesthe has a personal interest in the transaction while at the same time the trustee has a duty to act only in the best interests of the beneficiary. The duty against self-dealing is usually construed broadly to include transactions involving other members of the trustees family (spouse, children, parents). c.

Chapter 10 Trust Administration: The Fiduciary Obligation 106 of 120

i. Presumption of Breach of Trust (The No Further Inquiry Rule): If a trustee engages in selfdealing, an IRREBUTTABLE presumption of breach of the duty of loyalty arises. NO FURTHER INQUIRY OF GOOD-FAITH OR REASONABLENESS IS NECESSARY/APPROPRIATE ii. Hartman v. Hartle (Trustees Duty Against Self-Dealing) (p. 675) iii. F/P 1. The testatrix had 5 children. Her will appointed two of her sons-in-law executors of her estate and directed that her real property was to be sold and divided equally among the five children. The land was sold for $3,900 at public auction, and one of the testatrixs sons bought it for his sister, who was the wife of one of the executors. Two months later, the sister sold the land for $5,500. iv. I 1. Whether a trustee may sell property to his wife without permission by an order of the court? v. R/A/H 1. HELD: No. A trustees wife may not sell property from himself at his own sale. Nonetheless, a resale may not be ordered because the property is currently owned by innocent purchasers. Instead, the executors hold an account for the complainants onefifth share of the profits made on the resale of the property. 2. RULE: A trustee nor his wife may purchase property from himself at his own sale unless leave to do so has been previously obtained under an order of the court. 3. The duty against self-dealing applied to the spouse of the fiduciary. Absent court approval of the transaction, the sale was inappropriate. The sale could not be rescinded because of the subsequent sale to a bona fide purchaser w/o notice of the breach of trust, but the sister was forced to share 1/5 of the profit upon resale with the complaining beneficiary. 4. Because the executor sold the property to his wife, the beneficiaries did not receive a fair amount of the testators estate. vi. In re Gleesons Will (p. 676) vii. F/P 1. In 1950, Mary Gleeson leased 160 acres of farm land to the petitioner. In 1951, Gleeson renewed the lease with the petitioner for another year. Gleeson died just two weeks before the lease was to expire in 1952. Gleeson devised the land to the petitioner, as trustee, for the benefit of her three children. After Gleesons death, with the expiration of the second lease imminent, the petitioner remained on the land for another year until March 1, 1953. He increased his rent payments from $6 per acre to $10 per acre plus a share of the crops. He leased the land to another tenant after the holdover year. In the preceding fall of 1951, the petitioner sowed part of the 160 acres in wheat to be harvested in 1952. The petitioner held over in an open manner. viii. I 1. Whether the petitioner breached his duty of loyalty by holding over before the date that the lease expired to a year later? ix. R/A/H 1. HELD: Yes. The petitioner dealt individually with the farm land as a tenant and breached his duty of loyalty to the trust. An exception is not made for the petitioner because he held over shortly before the lease expired and farm land tenant are not easy to find. Though the petitioners holdover did not damage the property, he is still liable for breach. The petitioner must turn over the profits he made while he remained a tenant at the time that he was the trustee over the farm land, to the trust. 2. RULE: A trustee may not deal in his individual capacity with the trust property. 3. The fact that the petitioners actions did not harm the land did not absolve him of his duty to refrain from self-dealing.

Chapter 10 Trust Administration: The Fiduciary Obligation 107 of 120

c.

x. Traditional Exceptions to the Duty Against Self-Dealing: The duty against self-dealing can be waived either by (1) the settlor in the terms of the trust or (2) by all the beneficiaries, following a full disclosure of the proposed transaction 1. Judicial Review: Even where the self-dealing is authorized, the transaction must still be reasonable and fair. If it is not, the trustee is liable for breaching the duty of loyalty. (UTC 802(b)) xi. Modern Trend Exceptions to the Duty Against Self-Dealing: Many states have statutes permitting a bank trust dept to deposit trust assets in its own banking department; institutional trustees are increasingly authorized to combine separate trust accounts into a common trust fund or mutual fund; trustees are authorized to charge a reasonable compensation. (UTC 802(f), 802(h); R.3d Trusts 78 cmts 4, 6, 8) xii. Trust Pursuit Rule (Remedy): One of the remedies available to trust beneficiaries in equity for a breach of trust is the trust pursuit rule. (see p. 679) 1. Rule: If the trustee, in wrongfully disposing of trust property, acquires other property, the beneficiary is entitled to enforce a constructive trust on the property so acquired, treating it as part of the trust. rd 2. Rule: The trust pursuit rule also applies where property ends up in the hands of a 3 rd person, UNLESS the 3 person is a bona fide purchaser for value and w/o notice of the breach of trust Duty to Avoid Conflicts of Interest: A conflict of interest arises where the trust deals with another party w/ whom the trustee has an interest that may affect the trustees assessment of the proposed transaction. If the transaction involves a possible conflict of interest, but not self-dealing, the no further inquiry rule does NOT apply. Instead, the transaction is assessed to see if it is reasonable and fair under the circumstances. i. In re Rothko (Trustees Duty to Avoid Conflicts of Interest) (p. 679) ii. F/P 1. Rothko was an abstract expressionist painter who had an international reputation of greatness. He died on February 25, 1970. His will was admitted to probate on April 27, 1970. Reis, Stamos, and Morton Levine were the executors. Within a period of three weeks, the executors hastily dealt with Rothkos 798 paintings. By a contract of sale, the executors agreed to sell to Marlborough A.B. (hereinafter MAG) 100 Rothko paintings and to Marlborough Gallery, (hereinafter MNY) 700 paintings. The petitioner attempted to remove the executors, enjoin MNY and MAG from disposing of the paintings, and to rescind the agreements between the executors and said corporations, for a return of the paintings still in possession of those corporations, and for damages. Reis was the director, treasurer, and secretary of MNY, the consignee gallery. The testator had a 1969 inter vivos contract with MNY to sell Rothkos work at commission of only 10 percent. Reiss family had an extensive art collection through the Marlborough interests. Stamos was an unsuccessful artist under contract with Marlborough. Marlborough purchased a Stamos painting from a third party for $4,000 during the week in May 1970 when the estate contract negotiations were pending. Levine was aware of the transactions. iii. I 1. Whether executors fail to act unfairly in the transactions they entered into on behalf of the estate? 2. Whether an executor who acting prudently on the advice of counsel may be liable for the co-executors breach of trust? 3. Whether an executor who is liable for making an improper transfer where he had duty to retain property but chose to sell the property, is liable for appreciation damages? iv. R/A/H 1. HELD 1: Yes. The executors not only held an interest that conflicted with the interests of the estate, but they acted unfairly because their interests conflicted with the interests of the estate. Reis was induced to act in the MNYs favor in conducting the transactions

Chapter 10 Trust Administration: The Fiduciary Obligation 108 of 120

d.

e.

with the estate. As the director, secretary, and treasure of MNY, Reis was induced to favor the interests of MNY, including his own financial aggrandizement of status and financial advantage through sales of almost one million dollars for items from his own collection and his familys extensive private art collection by the Marlborough interests. Stamos was also induced to act in favor of the Marlborough interests because he was an artist under contract with Marlborough and the latter bought one of Stamoss paintings during a week when contract negotiations were pending. Stamos breached his duty not to accept employment from a company that was conducting business with the estate. 2. HELD 2: Yes. Though Levine acted on the advice of counsel, he is liable for damages because he failed to exert efforts directed towards prevention but acceded to the breaches. 3. HELD 3: Yes. Because the paintings cannot be returned to the trust estate, the estate is entitled to appreciation damages. This case involves wrongful transfers that should make the estate whole. The damage award is not punitive. 4. Executors may be held to the same standard as trustees. Trustees may have an interest in a transaction with the estate but they must not engage in the transaction unless they can show that they will not be improperly influenced by those interests when dealing with the estate. v. Damages: 1. Rule: Where a trustee is authorized to transfer trust property, but improperly sells it for too low a price, the trustee is liable for the difference in the actual sale price and the price that should have been realized. 2. Appreciation Damages: Where a trustee sells property he/she was NOT authorized to sell, appreciation damages are appropriate a. Definition: Appreciation damages are the difference between the sale price and the value of the property as of the date of the courts decree (which puts the beneficiaries back in the position they would have been in but for the unauthorized sale. b. Application to Rothko: In Rothko (above, p. 108), the court imposed appreciation damages on the two executors who acted with the conflict of interest, as well as the art gallery. Co-Trustee Liability: i. Traditional Rule: If there is more than one trustee of a private trust, the trustees must act as a group and with unanimity, UNLESS the trust instrument provides to the contrary. One of Several trustees does not have the power alone to transfer or deal with the property. Since co-trustees must act jointly, a co-trustee is liable for the wrongful acts of a co-trustee to which she has consented, or which by her negligence through inactivity or wrongful delegation, she has enabled the co-trustee to commit. ii. Modern Trend: The traditional rule of unanimity is on its way out. In many states, UTC 703(a) (or other statute) allows a majority of trustees to act if there are 3 or more trustees (see also R.3d Trusts 39). But note: Co-trustees still have a duty to take reasonable steps to prevent a breach of trust by her co-trustees, even by bringing suit if need be. iii. Charitable Trusts: Unanimity of action is not required of the trustees action by a majority is valid. iv. Trustees Right to Contribution from Other Trustees: Because co-trustees are jointly liable, a trustee generally has a right to contribution from co-trustees where he/she is found liable. 1. R.2d Trusts: The right to contribution may be limited if the trustee was either more at fault or benefited personally from the breach; and the right to contribution is eliminated if the trustee acted in bad faith. See also IL Statutes i. 755 ILCS 5/10-1 to 5 (Administrators to Collect) ii. 755 ILCS 5/12-1 to 15 (BondsOathsAcceptance of Office) iii. 755 ILCS 5/16-1 to 3 (Recovery of Property and Discovery of Information)

Chapter 10 Trust Administration: The Fiduciary Obligation 109 of 120

4.

iv. 755 ILCS 5/18-1 to 5 (Claims against Estates) v. 755 ILCS 5/22-1 to 6 (Nonresident Representative) vi. 755 ILCS 5/23-1 to 8 (Resignation and Removal of Representative) The Duty of Prudence: Imposes on the trustee an objective standard of care. a. Introduction: The trustee has a duty the trust with such skill and care as a person of ordinary prudence would use in dealing with his/her own property. The UTC adopts this approach (see UTC 804) b. Trust investmentsoverview: The issue of what constitutes an appropriate trust investment goes directly to the question of what is the purpose of a trust. The rules limiting trust investments to safe investments have given way to permitting an acceptable level or risk to ensure an adequate return on the trust property. i. Background: Historically, the most common approach to what constituted an appropriate trust investment was a judicial and/or statutory list of appropriate investments. The courts and/or legislature would identify categories of investments that were presumptively appropriate, but even then, an investment in a particular entity or activity on the list had to be otherwise reasonable and proper. ii. Common Law Approach: Each investment decision is viewed separately. (if one out of 100 investments is deemed inappropriate, the trustee is liable for any loss caused by the one inappropriate investment. 1. Settlors Authorization Exception: If the settlor expressly authorizes investments that are not on the jurisdictions statutory list, such investments are appropriate as long as they are otherwise reasonable and proper. 2. Judicial construction: Generally, courts tend to construe narrowly provisions authorizing a trustee to invest in otherwise inappropriate investments the trustee must still act reasonably and properly. c. Model Prudent Man Investment Act (The Old Prudent Man Rule): The Model Prudent Man Investment act abolished statutory lists i. RULE: Permits any investment that a prudent man would make. Speculative investments are not allowed. ii. Objective Standard: The most common statement of the prudent person std is that the trustee should invest with the same care as a prudent man would make of his own property, taking into consideration the dual goals of preservation of property (principal) and generation of income iii. Criticism: The prudent man rule has been criticized for putting too much emphasis on how risky investments are. Return on investments corresponds directly with risk. Permitting only nonspeculative investments restricts the potential return for beneficiaries. Moreover, safe investment may have little to no risk of complete loss, but may subject the trust to a substantial risk of inflation in that if inflation exceeds the rate of return, the real value of the trust property will fall. d. Uniform Prudent Investor Act: The UPIA, adopted in 1994, builds on the prudent person approach. The R.3d of Trusts ( 90) and the Trustee Act of 2000 have adopted the prudent investor standard (see CB p. 694 for UPIA) i. Prudent Investor Standard: A trustee shall invest and manage trust assets as a prudent investor would, by considering the purposes, terms, distribution requirements, and other circumstances of the trust. In satisfying this standard, the trustee shall exercise reasonable care, skill, and caution. (UPIA 2(a)) ii. Pooling Trust Funds: 1. The common law rule strictly requires each trust fund to be segregated from both the trustees own funds and other trust funds. 2. The Modern Trend and Majority Rule permits pooling of trust funds to achieve efficiencies of scale and to facilitate diversifying trust investments. The modern trend also permits investment in mutual funds. iii. Portfolio Approach: Trustees investment and management decisions are no longer assessed in isolation, but in the context of the trust portfolio (UPIA 2(b))

Chapter 10 Trust Administration: The Fiduciary Obligation 110 of 120

e.

A key to assessing the propriety of an investment under the portfolio approach is whether it is a compensated or uncompensated risk (Modern Portfolio Theory) 2. Compensated Risks: Compensated Risks are investments that are riskier than others, but have a corresponding higher rate of possible return associated w/ them. The investor is compensated appropriately for the enhanced risk. Compensated risks are appropriate investments under the portfolio approach as long as the overall risk level of the trusts investment portfolio is acceptable relative to the trust purposes. a. Example: Putting an appropriate amount of a trusts funds into a start-up company with great growth potential is an example of a compensated risk. 3. Uncompensated Risks: Uncompensated risks are those investments that are risky and do not have a corresponding market-enhanced compensation to reward the investor for taking the risk a. Example: Putting all of ones investments into one stock (regardless of the level of risk associated w/ the stock) is an uncompensated risk (i.e. lack of diversification = uncompensated risk) 4. Investment Decisions: Arguably the key considerations in assessing a trustees investments under the portfolio theory approach are (1) the trustees investigations and decision-making process in determining the trusts acceptable level of compensated risk, and (2) how that level is achieved through the combination of trust investments. 5. Duty to Delegate: The UPIA permits trustees to delegate investment and management functions. Delegating the investment process to an expert is viewed with favor, though the trustee still has a duty (1) to properly investigate to whom the power should be delegated, (2) to consult with the agent to ensure that he/she properly understands the trusts terms, purposes, and acceptable level of compensable risk, and (3) to monitor the activities and decisions of the investment agent. iv. Duty to Diversify: The trustee must diversify the investments of the trust, UNLESS the trustee reasonably determines that, because of special circumstances, the purposes of the trust are better served without diversifying (UPIA 3) 1. A well-diversified portfolio spreads the risk of loss across all the investments so that the aggregate level of risk is acceptable in light of the trust purposes. Adequate Diversification: The question of how much diversification is necessary is not addressed in the Act. (it is a fact-sensitive issue to be determined on a trust-by-trust basis, considering the purpose of the trust and the particular investments in question. i. In re Estate of Janes (Risk, Return, and Diversification in Practice) (p. 702) ii. F/P 1. The testator died (May 1973) with a probate estate of approximately $3.5 mil, approx. $2.5 mil of which was held in stock. 71% of that $2.5 mil (approx. $1.79 mil) consisted of 13,232 shares of Kodak common stock. His will bequeathed most of his estate to 3 trusts. Trust 1 was a marital deduction trust for the benefit of his wife, Cynthia; Trust 2 was for the benefit of selected charities. Trust 3 was for Cynthias benefit during her life, and upon her death, the principal was to pour over to the charitable trust. 2. Cynthia and Lincoln Rochester Trust Co were appoint co-executors. By August 1973, the Trust Cos trust and estate officers had determined the estates expenses and how many shares of stock needed to be sold to cover the expenses. At that meeting the trust officers recommended holding the remaining shares until the trusts were funded. The memo did not otherwise discuss investment strategy. In Sept 1973, Cynthia consented to the sale of an additional 1,200 shares of Kodak stock. At that time, the stock was $139/share. The last time the trust officers discussed the retention of the stock or other investment issues w/ Cynthia. Afterwards, Kodak stock declined steadily every year, to $41/share by Mar 1978. 3. In 1980, the trust co filed its initial accounting covering most of the pd in question, and sought judicial settlement of it. Cynthia and the atty gen objected and sought to

1.

Chapter 10 Trust Administration: The Fiduciary Obligation 111 of 120

4.

5. 6.

surcharge the trust co for its imprudent retention of the high concentration of Kodak stock, in violation of the prudent investor rule. The surrogate court found the trust co had acted imprudently the trust co should have sold the stock by Aug 1973. The court imposed a $6.1 mil surcharge (including a lost profit on the money which would have been reinvested if there had been proper divestment. The Appellate Division upheld the finding of imprudence and the date used to calculate the damages, but not the inclusion of lost profits surcharge was reduced to $4.1 mill. On appeal, trust co argued there was no duty to diversify absent additional elements of hazard, and that a list of factors indicated that no additional elements of hazard existed in this case.

7. iii. I 1. Whether a fiduciarys duty of investment prudence may be limited to the opinion of investment bankers and analysts who follow the companys stock, and an overall determination of the investment quality determined by (1)the capital structure of the company, (2) the competency of its management, (3) whether the company is a seasoned issuer of stock with a history of profitability, (4) whether the company has a history of paying dividends, (5) whether the company is an industry leader, and (6) the expected future direction of the companys business? Whether, under all of the facts and circumstances of this case, the fiduciary violated the prudent person standard in maintaining a concentration of the Kodak stock? Whether August 9, 1973 was a reasonable time by which the petitioner should have divested the estate of the stock? Whether the proper measure of damages for breach of a duty to act prudently is lost profits or the amount that the proceeds of the stock would have yielded, up to the time of trial, had they been invested in petitioners own diversity equity fund on August 9, 1973. HELD 1: No. A fiduciarys duty of investment prudence may not be limited to the opinion of investment bankers and analysts who follow the companys stock and other factors offered by the petitioner. The prudent person rule dictates against any absolute rule that immunizes a fiduciary from its failure to diversify based upon such factors. Also, omits other factors to be considered under the prudent investor rule like, the amount of the trust estate, the situation of the beneficiaries, the trend of prices and of the cost of living, and the prospect of inflation and deflation. HELD 2: Yes. The petitioner here acted imprudently by failing to divest the estate of the Kodak stock by August 9, 1973 because the petitioner jeopardize the interests of the primary income beneficiary. The Kodak stock dropped in value, decreasing the amount of income that would be available to the primary income beneficiary, the testators 72 year old widow for whom the support testamentary trusts were created. Furthermore, the petitioner failed initially to undertake a formal analysis of the estate and establish an investment plan consistent with the testators primary objectives; (2) failed to follow petitioners own internal trustee review protocol during the administration of the estate, which advised special caution and attention in cases of portfolio concentration of as little as 2%; and (3) failed to conduct more than routine reviews of the Kodak holding in this estate, without considering alternative investment choices, over a sevenyear period of steady decline in the value of the stock. HELD 3: Yes. August 9, 1973 was a reasonable time by which the petitioner should have divested the estate of the stock The petitioners internal documents and correspondence, as well as the testimony of Patterson, Young and objectants experts establish that the petitioner had all the information a prudent investor would have needed to conclude that the percentage of Kodak stock in the estates stock portfolio

2. 3. 4.

iv. R/A/H 1.

2.

3.

Chapter 10 Trust Administration: The Fiduciary Obligation 112 of 120

was excessive, and should have been significantly reduced in light of the estates over-all investment portfolio and the financial requirements of Janes and the other charitable beneficiaries. 4. HELD 4: No. The proper measure of damages is the value of the capital that was lost, the difference between the value of the stock at the time it should have been sold and its value when ultimately sold. In this case which involves faithless transfers as oppose to deliberate self-dealing, the proper measure of damages is not lost profits. 5. RULE: A trustee must diversity assets unless the trustee reasonably determines that it is in the interests of the beneficiaries not to diversify, taking into account the purposes and terms and provision of the governing instrument. In imposing liability upon a fiduciary on the basis of the capital lost, the court should determine the value of the stock on the date it should have been sold, and subtract from that figure the proceeds from the sale of the stock, or, if the stock is still retained by the estate, the value of the stock at the time of the accounting. The court has discretion on whether interest should be awarded. Dividends and other income attributable to the retained assets should offset any interest awarded. 6. No precise formula exists for determining the prudent person standardeach case turns on its own facts and circumstances. The trustees investment decisions are to be measured in light of the business and economic circumstances at the time they were made 7. The high concentration of Kodak stock (with the other shares primarily in other stocks) failed to take into adequate consideration the needs of the testators 72 yr-old widow. 8. The trust co failed to exercise the due care and skill of a corporate fiduciary by (1) failing to establish an investment plan upon funding; (2) failing to follow its own internal policies of special caution and attention to cases of portfolio concentration exceeding 20%; and (3) failing to conduct more than routine reviews of the account in the face of declining values. v. Duty of Care Professional/Corporate Trustees vs Individual Trustees: Professional and corporate trustees are usually held to a higher standard of care in investing due to their presumed expertise. Individual trustees are usually held to a lower standard. vi. Exceptions to Duty to Diversify: 1. It may be prudent not to diversify or to delay diversification when the tax or other costs of reorganizing the portfolio are likely to outweigh the benefits of diversification (UPIA 3, cmt). 2. Or if a trust holds a family business, esp if the business is closely held and not readily marketable 3. Or if the trust is just one component of a larger scheme such that the beneficiarys financial interests are diversified overall 4. Investment in a mutual fund may constitute adequate diversification if the fund is diversified. vii. Inception Assets: Many jurisdictions permit a trustee to have a preference for retaining the trusts inception assetsthe assets used to fund the trust that the settlor recommends the trustee retain. 1. Requirement vs Authorization: Even if a trust authorizes the trustee to retain inception assets irrespective of diversification, courts may still find the trustee violated fiduciary duty (see First Ala. Bank of Huntsville, N.A. v. Spragins (CB p. 717)). (see also Emanuel p. 267) 2. The right is NOT absolute: It is subject to the trustees more general fiduciary duty of prudent administration a. Power vs Duty: Even if the trustee has the POWER to retain certain assets, the exercise of the power must accord w/ the trustees duties of prudence and loyalty (R.3d Trusts 91)

Chapter 10 Trust Administration: The Fiduciary Obligation 113 of 120

f.

viii. Authorization to Retain vs Duty to Sell: Even where the trust instrument authorizes the trustee to retain the trust assets in question, where failure to diversify is inconsistent with the modern portfolio approach, the trustee has a duty to sell the trust property in a timely manner (w/in a reasonable time period) ix. Direction to Retain vs Duty to Sell: Where the trust instrument directs the trustee to retain the trust assets in question, the issue is more complicated. 1. General Rule: Settlors intent controls some courts have ruled that the trustee must comply w/ the retention order 2. Modern Portfolio Approach: Some courts have approved diversification if there are changed circumstances, i.e. trustee doesnt have a duty to follow the settlors directions x. Calculating Damages: Where there is a breach of trust, the trust beneficiaries are entitled to be made whole. Three ways to do this: (1) Charge the trustee with any resulting loss; (2) charge the trustee with any profit made; or (3) charge the trustee with any profit that would have accrued but for the breach. (See UTC 1002) see CB 709 1. The Make-Whole Approach: The Total return/make whole damages approach holds the trustee liable for any losses incurred and gains forgone as a result of the breach. The trustee is also liable for any profit made by the trustee through the breach of trust. (UTC 1002). a. Restatement Approach: Where there are several plausible investment strategies, the R.3d Trusts favors application of the most favorable unless the trustee can justify why it should not apply 2. The Capital Lost plus Interest Approach: a. This approach takes the value of the trusts stock on the date when the trustee should have divested, plus compound interest through the [another date], minus the actual value of the trust. b. This approach does not punish the trustee for lost investment opportunities, awarding interest to the trust beneficiaries rather than possible profits from prudent investment of the lost capital. Under this approach, the rate of interest is critical. Different possible rates include the historic average rate of inflation; the annual return on long-term government bonds; and the legal rate applied to money judgments. 3. The Total Return Approach: The total return approach calculates damages by awarding the difference between how the particular, imprudently managed, portfolio actually performed vs how a hypothetical matching portfolio, prudently managed, would have performed (taking into account taxkes, expenses, and distributions). a. This approach is fact-intensive; requires expert testimony; is somewhat speculative. xi. Social Investing: Social Investing is choosing to invest (or not invest) based on social issues (e.g. choosing not to invest in a tobacco manufacturing companys stock 1. RULE: UPIA 5 says trustees should ALWAYS invest in a manner that best serves the trust beneficiaries i.e. no form of social investing is consistent with the duty of loyalty if the investment activity entails sacrificing the interests of trust beneficiaries in favor of the interests of the persons supposedly benefitted by pursuing the particular social cause. Delegation: At common law, a trustee was not allowed to delegate matters that the trustee could reasonably be required to perform (R.2d Trusts 171). i. Rationale for Common Law: Settlor put great trust in the trustee and assumed that the trustee would personally hold/manage the property. So, delegation violated the settlors intent ii. ExceptionMinisterial Responsibilities: Even at common law, there was an exception for ministerial responsibilitiesthose activities that do not require the exercise of discretion (e.g. cutting grass, making repairs, etc.)

Chapter 10 Trust Administration: The Fiduciary Obligation 114 of 120

5.

iii. Modern Trend (Duty to Delegate): Recognize that some trustees are unqualified to undertake certain responsibilities in holding/managing trust property, esp the duty to invest trust property properly. 1. Rules: UPIA 9, R.3d Trusts 171, and UTC 807 all agree trustee may have a duty to delegate responsibilities if a prudent person would delegate under similar circumstances (see CB p. 720). Trustee must act in the best interests of the beneficiaries in deciding whether to delegate discretionary responsibilities, including investment-making responsibilities, and to whom to delegate them. 2. Duty to Supervise: The trustee has an ongoing duty of care in a. (1) selecting agents to whom to delegate; b. (2) defining the agents role and giving proper instructions; and c. (3) monitoring/supervising the actions of the agent(s) to whom the trustee delegates the responsibilities to ensure that the agent(s) act(s) w/in the delegated authority and in the best interest of the beneficiaries d. The trustee cannot abdicate or delegate unreasonably e. (UPIA 9, See CB p. 722 for longer-winded versions of this :-D) iv. Directed vs Delegated Trusts: 1. Delegated Trust: The TRUSTEE decides which tasks should be delegated and to whom they should be delegated. The trustee is subject to the UPIA duties inherent in selecting, instructing, and supervising the agents 2. Directed Trust: The SETTLOR directs the trustee (through the terms of the trust) to follow the instructions of others. i.e. the settlor selects the agents to whom certain rd tasks are to be delegated, and the trustee must follow the 3 parties instructions. 3. Rules: a. R.3d Trusts 75 trustee has a DUTY to follow a direction of a person if the trust so provides, UNLESS the direction is contrary to the terms of the trust or the trustee knows or has reason to believe that the direction violates a fiduciary duty that the power holder owes to the beneficiaries. b. UTC 808 The trustee must follow a direction unless it is manifestly contrary to the terms of the trust or would constitutes a serious breach of fiduciary duty. c. Delaware Code Title 12 3313: The one who directs (called a trust adviser) is presumptively a fiduciary, but the governing instrument can provide otherwise, and the trustee has no duty to monitor or to notify the beneficiary if the truste has concerns about direction i. i.e. The trustee is protected from liability except in cases of the trustees own willful misconduct Impartiality Allocating Principal And Income a. Introduction: Perhaps the most important sub-rule in trust fiduciary law is the duty of impartiality. This duty is implicated when a trust has two or more beneficiaries. b. Rule: In investing, managing, and distributing the trust property, the trustee must strike a balance between the beneficiaries, giving due regard to their respective interests (e.g. strike a balance between investing the property to produce reasonable income, while preserving the principal for the remaindermen) (UTC 803, R.3d Trusts 79) i. Policy: Different beneficiaries have different interests (e.g. present interests life estate income interests vs future interests remainder in the principal). Because the beneficiaries have different property interests, their personal interests often conflict. ii. Note: Impartiality is not truly impartial the trustee must take into account any preferences that the settlor may have expressed in the governing instrument (or some other manner), and must also consider the various/conflicting beneficiaries interests. iii. Notewe skipped the case in the reading (Howard v. Howard). c. Accounting for Principal and Income i. 1962 Principal and Income Act:

Chapter 10 Trust Administration: The Fiduciary Obligation 115 of 120

Income: Money generated on a regular (or irregular) basis as a result of the trust property or trust investments constitutes income (eg interest, rent, cash dividends on stock 2. Principal: Money generated as part of a conveyance (voluntary or involuntary) of trust property is considered principal (eg sales proceeds, insurance proceeds) a. In Addition: Stock splits and stock dividends are considered principal because such property has to be retained as principal to maintain the trusts percentage interest in the company. Bond principal payments and part of royalty payments are also considered principal ii. 1997 Principle and Income Act the power of equitable adjustment: MAJORITY/MODERN Approach Focuses on the total return to the trust portfolio. As long as the trust achieves an acceptable rate of return on its investments, it irrelevant whether that return came from income or principal as traditionally defined. 1. Trustees Power: Trustee has power & discretion, the power of equitable adjustment, to reallocate the total return between the income and principal beneficiaries to ensure that the two groups are treated fairly while paying particular attention to the larger rate of return regardless s of how the return is classified 2. Settlors Intent: The settlor may expressly provide that the trustee does not have power to reallocate principal and income. iii. Unitrust: Under a unitrust, the life (income) beneficiaries are entitled to a specified percentage of the value of the trust principal each year, so there is no need to distinguish income from principal. 1. Rule: All property generated by the trust is assigned to principal, and at the appropriate intervals, the specified percentage of the trust principal is distributed to the appropriate beneficiaries 2. Purpose of Unitrust: Permit the trustee to focus on investing the trust portfolio to maximize total return, as opposed to worrying about investing to ensure an appropriate income stream for the income beneficiary. iv. Unitrust Election (Adjustment Powers) 1. Introduction: Many states have statutes authorizing a trustee to convert a traditional trust into a unitrust (adjustment powers). 2. Trustee-beneficiary: Some states deny this power to a trustee-beneficiary; others permit it but scrutinize the trustees actions more closely (See UPIA 104(c)(7)) 3. Settlors Intent: This statutory adjustment power is a default ruleit only is operational if the settlor does not provide otherwise (see also R.3d Trusts 113) 4. In re Matter of Heller (Unitrust) (p. 731) 5. F/P a. Jacob Heller created a testamentary trust for the benefit of his wife Bertha and his children from a previous marriage. Bertha was to receive income from the trust each year, and after her death, the principal was to be distributed to his children. Jacobs two daughters were to receive a 30% share each. His two sons, who were also trustees, were to get 20% each (they were trusteebeneficiaries). b. Following Jacobs death, the income to Bertha averaged $190,000/yr for several years. In 2001, to facilitate overall portfolio performance, NY enacted legislation that created an optional unitrust provision. It allows a trustee to calculate the income to be distributed according to a fixed formula and based on the net fair market value of the trust assets. c. In 2003, the brothers elected to apply this option retroactively, reducing Berthas annual income to $70,000. Jacobs sons were then able to make investments, although they produced low dividend yields, would outperform the alternative in the long-term, creating a better overall return. 6. I 1. Chapter 10 Trust Administration: The Fiduciary Obligation 116 of 120

a. 7. R/A/H a. b.

Does the trustee-beneficiarys election of unitrust violate fiduciary duties to beneficiaries?

6.

HELD: No RULE: When an interested trustee elects a unitrust, it does not per se violate fiduciary duties. c. Though the brothers were beneficiaries and trustees, they could elect unitrust treatment. d. NOTE: The court ruled that the lower courts should scrutinize the unitrust election with special care in such cases to ensure that the election is not a violation of the trustees fiduciary duties to any of the beneficiaries. Sub-Duties Relating to Care of Trust Property a. Introduction: At the macro level, a trustee has a duty to care for the property as a prudent person would care for the property of another. i. Duty to Collect and Protect: A trustee has a duty to collect and protect trust property w/o unnecessary delay (UTC 809; R.3d Trusts 76(2)(b)) 1. How long of a delay is reasonable: W/ testamentary trusts, trustee should collect the assets from the executor as soon as circumstances permit 2. Duty to Examine Property: W/ testamentary trusts, trustee owes a duty to the beneficiaries to examine the property tendered by the executor to make sure it is what the trustee ought to receive ii. Duty to Care For and Maintain: Trustee should treat the property as an ordinary owner would treat similarly situated property (e.g. insure the property, keep the property in good repair, etc) iii. Duty to Earmark: Where the trust property is personal property (esp fungible assets, such as money, stocks, etc.), the trustee has a duty to separate the trust property from all other assets and to properly designate the property as trust assets. This ensures that a trustee cannot switch trust assets and personal assets after the fact where the trust assets outperform the personal assets (UTC 810) 1. Fungible means a description applied to items of which each unit is identical to every other unit, such as in the case of grain, oil, or flour. 2. Fungible goods are those that can readily be estimated and replaced according to weight, measure, and amount. 3. ExceptionAssets Not Subject To Registration: Assets not subject to registration, such as bearer bonds, fall within an established exception to the earmarking requirement. In such cases, the trustee must keep records showing that the property belongs to the trust and should be kept separate from the trustees own property (R.3d Trusts 84, cmt d) 4. Common lawstrict liability: At common law, if a trustee breached the duty to earmark, the trustee was strictly liable for any damage the trust property sustained (even if the damage is not caused by the breach). 5. Modern TrendCausation: The modern trend is that a trustee is not liable for a breach unless the breach of the duty causes the damage to the trust property. iv. Duty not to Commingle Trust Funds with the Trustees Own: 1. RULE: A trustee is guilty of breach of trust if the trustee commingles the trust funds with his own, even if he does not use the trust funds for his own purposes. (UTC 810(b); R.3d Trusts 84, cmt b) 2. Policy: Commingled trust funds become difficult to trace, and are subject to the risk that personal creditors of the trustee can reach them (i.e. the trustee would be able to improperly pay off debts to his own creditors using trust property. This is a no-no). This defeats the purpose of trust law, which is that trustees creditors generally cannot reach the trust property. 3. Modern Trend: Keeps the general commingling rule, but permits commingling with other trust funds (but not the trustees own funds) to achieve economies of scale and to

Chapter 10 Trust Administration: The Fiduciary Obligation 117 of 120

7.

improve the efficiency of trust administration. The trustee must maintain records clearly indicating the respective interests. (UTC 810(d), see p. 737) 4. Standard of Liability: a. Common Law: Strict liability if the trustee breaches the duty not to commingle b. Modern Trend: Trust beneficiaries must prove the breach caused the damage to the trust property Duty to Inform and Account a. Duty to Inform/Disclose: The trustee has a duty to inform the beneficiaries of the existence of the trust and significant developments related to trust administration, and to respond promptly to requests by beneficiaries for information reasonably related to the beneficiaries interests in the trust. (UTC 813) (See CB p. 738) i. Settlor Authorizes Withholding Information: The law is not clear about what should happen when the settlor expressly provides in the trust that the terms of the trust or information about the trust property are to be withheld from the beneficiary. 1. UTC: Default rule is that the trustee must promptly provide a copy of the trust instrument to the beneficiary if the beneficiary requests a copy, UNLESS the settlor provides otherwise (UTC 813(b)(1)) 2. CaliforniaRight to Receive: Cali provides that upon the death of a settlor of a revocable trust, all beneficiaries AND HEIRS of the settlor have the right to request a complete copy of the trust instrument. ii. Fletcher v. Fletcher (Trustees Duty to Disclose/Inform) (p. 739) iii. F/P 1. Fletcher executed a revocable inter vivos Trust Agreement. The agreement contained specific provisions for the establishment of a number of trusts upon the grantors death, including three separate trusts for the respective benefit of certain children. The three separate trusts were to be in the amount of $50,000 each. The trustees were authorized in their discretion to expend of the benefit of James N. Fletcher, Jr., an adult child of the grantor, such amounts of the net income an principal, of the $50,000 trust as may be necessary to provide him adequate medical insurance and medical care during his lifetime, or until such time as the trust is depleted. They were also authorized to expend, in their discretion, for the benefit of Fletchers children such amounts of the income and principal for the benefit of Fletchers children. 2. The plaintiff alleged that one of the trust instruments recites that the Grantor transferred, assigned, and set over certain cash and securities which were described in a schedule entitled A attached to the trust agreement. The plaintiff also alleged that on his m others death, he was advised that the trust assets had been transferred to a new trust with the defendants as trustees. The plaintiff requested the details of both trusts from the trustees but the trustees refused to comply with his request. The plaintiff also alleged that the trustee, Henry L. Fletcher, justified his failure to disclose the requested information by stating that his mother requested that the trust terms and dealings be kept confidential, even from the beneficiaries. 3. The trial court issued an order that the plaintiff had an absolute right to complete copies of the Trust Agreement and all amendments referred to in the pleadings and associated documents. The court ordered the Trustees to provide the plaintiff with full and complete copes of the trust instruments that are referred to in the Bill of Complaint iv. I 1. Whether the trustees had a duty to disclose the terms of the trust agreement to the beneficiaries? v. R/A/H 1. HELD: Yes. The trustees had a duty to disclose the terms of the trust agreement to the beneficiaries, even though the terms of the trust itself did not require such a disclosure. 2. RULE: The terms of a trust may regulate the amount of information which the trustee must give and the frequency with which it must be given, but the beneficiary is always

Chapter 10 Trust Administration: The Fiduciary Obligation 118 of 120

3.

4. b.

entitled to such information as its reasonably necessary to enable him to enforce his rights under the trust or to prevent or redress a breach of trust. Without access to the agreement, the beneficiaries were not able to assure the Trustees were discharging their duty to deal impartially with all the beneficiaries in regards to the trust agreement, in regards to their duty to use reasonable care and skill to make the trust property productive, or in regard to whether the trustees investment decisions made with respect to the assets revealed on Schedule A. Though the claim that the grantor did not want the beneficiaries to see the trust agreements in full was unsubstantiated, the Grantor could not prevent the beneficiaries from having access to the trust agreements. Though the trustee does not have a duty in statute to turn over the trust documents, the beneficiaries did have a right to see the trust documents.

Duty to Account: i. Introduction: The law protects a trustee from liability to the beneficiary if (1) the facts underlying the beneficiarys claim are fairly disclosed in an accounting filed with the court, (2) notice of the accounting is properly served on the beneficiary, and (3) the beneficiary does not timely object to the accounting. 1. Problems arise when the facts underlying a subsequent claim by the beneficiary may not have been fairly disclosed in the accounting. ii. Testamentary Trusts: Trustees have a duty to account to the probate court so that the court can assess the trustees performance. 1. No Judicial Accounting Clauses (Settlors Intent): Some courts permit a provision in the trust releasing a trusting from his/her duty to account to the probate court (because doing so is slow and expensive) as long as the trustee accounts directly to the beneficiaries (typically the income beneficiaries). a. BUT: some courts hold that such no judicial accounting clauses violate public policy because they fail to adequately protect the interests of the remaindermen. iii. Inter Vivos Trusts: Inter vivos trusts are not created as part of the probate process, so they are not naturally subject to probate court supervision, though judicial accounting is still possible. 1. No Judicial Accounting Clauses: Usually held valid with inter vivos trusts, because the trusts dont pass through probate. iv. Uniform Trust Code: UTC drops the reference to accounting calls it reporting instead. 1. Settlor may waive the duty to report to the beneficiaries; (UTC 813(c)) 2. Beneficiaries may likewise waive their right to receive a report or other information (UTC 813(d)) 3. A beneficiary may not ex ante and irrevocably waive rights to all reports and information a. The term ex ante is a Latin word which means based on assumption and prediction. It also means beforehand or before the event b. This means that beneficiaries cannot pre-emptively waive ALL rights to receive reportsonly on a one-off basis c. Also, waiver does not relieve a trustee from liability for misconduct that a report would have disclosed. v. Beneficiaries Duty to Review Accounting: When a trustee makes an accounting (to court or directly to beneficiaries), the beneficiaries have a duty to check the accounting and to object in a timely manner. If the beneficiaries fail to object timely, they may be barred from complaining later. vi. Fraudulent Accounting: When a trustee files a fraudulent accounting, and the beneficiaries later discover the fraud, the beneficiaries MAY re-open the account vii. Constructive Fraud: Where an accounting makes factual representations that turn out to be false, if the trustee made the representations without undertaking reasonable efforts to ascertain the accuracy of the factual representations, the representations constitute

Chapter 10 Trust Administration: The Fiduciary Obligation 119 of 120

8.

constructive or technical fraud. Such fraud is grounds for reopening an otherwise properly allowed accounting 1. Investigation: The doctrine of technical fraud does not make trustees guarantors of all factual representations in an accounting. If the representations were made in good faith and follow reasonable efforts to ascertain the accuracy of the representations, the trustee has fulfilled his/her duty. 2. Constructive Fraud Only Applies Factual Representations: Not to statements of judgment or discretion 3. Discoverability: Constructive Fraud does not apply if the factual falsehood is discoverable from an inspection of all the trust accounts, the trust terms, and the law. viii. Improper Payments: Where an accounting reveals that a trustee has improperly distributed trust property to one who is NOT entitled to receive such property, the trustee is liable fro breach of trust UNLESS the court approves the accounting. Where the courts approval is based on fraudulent accounting, reopening such accounting voids the courts approval of the accounting. ix. Example: National Academy of Sciences v. Cambridge Trust Co. (Not assigned) See also IL Statutes a. 755 ILCS 5/14-1 to 3 (Inventory and Appraisal) b. 755 ILCS 5/19-1 to 14 (Administration of Personal Estate) c. 755 ILCS 5/20-1 to 24 (Administration of Real Estate) d. 755 ILCS 5/21-1 to 2 (Investments by Representative) e. 755 ILCS 5/24-1 to 22 (Accounts)

Chapter 10 Trust Administration: The Fiduciary Obligation 120 of 120

Das könnte Ihnen auch gefallen